You are on page 1of 97

Bridging Course MCQs

Block 1
Block 2
Block 3
Block 4
Block 5
Block 6
Block 7
Block 8
Block 9
Block 10

Block 1
Question 3

Question 1
Patients suffering from schizophrenia may experience
both positive and negative symptoms. All of the
following are negative symptoms EXCEPT:
a) Anhedonia
b) Hallucinations
c) Lack of motivation

Maud is a 70 year old who presents with acute periumbilical abdominal pain gradually increasing in
intensity. She is vomiting profusely and develops watery
diarrhoea with flecks of blood after an hour of pain.
Examination of the abdomen reveals localised
periumbilical tenderness with some rigidity. Rectal
examination is normal. An irregular pulse is noted and
an ECG is recorded (shown below).
The MOST LIKELY diagnosis is:

d) Emotional blunting

a) Acute appendicitis

e) Alogia

b) Acute pancreatis
c) Perforated peptic ulcer

Answer: B
Hallucinations are abnormal perceptions, e.g. hearing
voices speaking about the patient, and/or giving
instructions to do certain things. They represent
additional phenomena and, therefore, are classified as
positive symptoms. Delusions (false ideas) is the other
major group of positive symptoms. In contrast, each of
the other options represents a loss or diminution of a
mental function, e.g. anhedonia is a loss of capacity for
pleasure.

d) Biliary colic
e) Mesenteric artery occlusion

Answer: E
The clinical presentation is typical of mesenteric artery
occlusion. This occurs most commonly in patients with
atrial fibrillation leading to embolism. The ECG shows
atrial fibrillation. Arteriography will show the vascular
occlusion.

Question 2
Martha was born at 35 weeks gestation. She required
phototherapy for jaundice whilst in hospital. Martha is
being breast-fed and is beginning to gain weight. She is
now two months old. Regarding routine childhood
vaccination which of the following statements is
CORRECT?
a) Vaccinations due to be given at two months
of age should be postponed until Martha is at
least 13 weeks old to allow for her prematurity.
b) Martha's past history of jaundice is not a
contraindication to oral polio vaccine.
c) Vaccines for intramuscular injection should
be given into the buttock in a premature baby.
d) Breast-feeding
vaccination.

is

contraindication

to

e) It is not necessary to immunise Martha


against hepatitis B.

Answer: B
A history of jaundice after birth is not a contraindication
to any of the vaccines in the standard schedule.
Vaccination should not be postponed because of
prematurity. The fact that a child is breast-fed is not a
contraindication to vaccination. Vaccines should never
be given into the buttocks. The anterolateral thigh is the
preferred site for vaccination in infants under 12 months
of age. The deltoid region is the preferred site in older
children (those who have commenced walking) and in
adults. Martha would be due to receive vaccination
against hepatitis B at the age of two months according
to the current standard vaccination schedule.

Question 4
A 15 year old male has sudden onset of severe pain in
his right lower abdomen commencing 2 hours ago. He
has vomited several times in the last hour. He is rolling
on the bed, stating that the pain is going down into his
groin. T 37.1 degrees Celcius, P 110min, BP 135/ 80.
Abdomen - soft, no rebound. Tender right testicle. Your
immediate management is:
a) i/v fluids and antibiotics
b) arrange urgent ultrasound examination
c) i/v metoclopramide (maxolon)
d) refer for emergency surgery
e) arrange for intravenous pyelogram (IVP)

Answer: D
The sudden onset of severe pain in the lower abdomen,
groin or scrotum, in a young male under 25 years,
should be considered to be testicular torsion until proved
otherwise. This is a surgical emergency, as infarction of
the testis can occur quickly, and surgical exploration
should be undertaken urgently. This patient has no
fever,
nor tenderness of the epididymis to indicate epididymoorchitis. Antibiotic treatment will not help. Colour
doppler ultrasound may show increased blood flow in
infection and the absence of flow in advanced torsion.
However, these are not reliable findings, and the
investigation would waste valuable time. The vomiting is
related to the pain, and would be alleviated by
appropriate analgesia. Metoclopramide is not an

immediate priority. The clinical picture is highly


suggestive of testicular torsion rather than renal colic,
thus IVP is not the appropriate immediate management.

c) Descent is unlikely to occur after 1 year of


age.

Question 5

e) The undescended testis is at reduced risk of


malignancy.

In a 3 year old child with signs and symptoms


suggestive of bacterial meningitis, which of the following
is the BEST initial management?
a) Erythromycin IV

d) Orchidopexy should be delayed until late


childhood.

Answer: C
Testes which are undescended at birth may well descend
into the scrotum during the first two weeks of life,
however descent is unlikely to take place after the age
of one

b) Gentamicin IV
c) Ceftriaxone IV
d) Phenoxymethylpenicillin oral
e) Amoxycillin oral

Answer: C
If bacterial meningitis is suspected clinically it is vital to
immediately administer an appropriate antibiotic prior to
urgent transfer to hospital, as meningococcal meningitis
may be rapidly fatal. The drug of choice would be
benzylpenicillin 60mg/kg up to 3g IV or IM, or
ceftriaxone 50mg/kg up to 2g IV in patients
hypersensitive to penicillin or when further drug
treatment may be delayed.

year. 2% of boys born at full-term, and 20% of


premature males, have undescended testes. A testis
which was palpable in the scrotum in infancy may
ascend and become impalpable due to failure of the
spermatic cord to elongate at the same rate as body
growth. Orchidopexy is best performed by 12-18 months
of age as spermatogenesis in the undescended testis is
impaired after the age of two years. The undescended
testis is at 5-10 times greater risk of developing
malignancy (seminoma).

Question 8
An aspirate in an acutely painful, swollen knee shows
the following:

Question 6

white cell count 4100/uL (< 2000/uL)

Atypical antipsychotic drugs (eg. olanzapine) have


certain
advantages
compared
to
the
typical
antipsychotic drugs (eg. chlorpromazine) in the
treatment of schizophrenia. All of the following are
advantages of atypical drugs EXCEPT:

red blood cells ++

a) Improved therapeutic effect on positive


symptoms
b) Improved therapeutic effect on negative
symptoms
c) Reduced potential for acute extrapyramidal
symptoms
d)
Reduced
potential
extrapyramidal symptoms

for

e) Improved therapeutic
treatment-resistant patients

effect

longer-term
in

some

Answer: A
Both the typical and atypical antipsychotic drugs have a
beneficial effect on positive symptoms in schizophrenia.
However, they differ with regard to negative symptoms.
Atypical antipsychotics are much better at combating
these. The other options are true for the atypical drugs.

no organisms cultured
What is your interpretation of these results?
a) Inflammation is more likely
b) Traumatic tap - cannot be interpreted
c) Gonococcal arthritis
d) Tuberculosis arthritis
e) Viral arthritis

Answer: A
Normal synovial fluid contains less than 2000 white
blood cells per microlitre. Inflammation causes counts of
3000 or higher. Inflammation can also cause red blood
cells to migrate into the joint fluid.

Question 9
Concerning prostate cancer, which ONE of the following
is INCORRECT?
a) The majority of carcinomas arise in the
peripheral zone of the gland

Question 7
Which
of
the
following
statements
undescended testes is CORRECT?

no crystals

regarding

a) The testes are undescended at birth in 40%


of boys.
b) Once the testis is palpable in the scrotum it
will remain so.

b) The prostate specific antigen (PSA) is


elevated in all prostate cancers greater than
1.5 cm
c) Metastatic spread to pelvic lymph nodes
occurs early

d) Radionuclide bone scan is not a reliable


method of detecting bony metastases
e) Prostate cancers usually appear as
hypoechoic nodules on transrectal ultrasound

Answer: B
A normal PSA occurs in up to 20% of cancers, including
tumours greater than 1.5 cm. Prostate cancer is the
second most common cause of cancer deaths in men in
Australia. The majority of prostate cancers- usually
adenocarcinomas- arise in the peripheral zone of the
gland, and metastasise early to pelvic lymph nodes.
Bony metastases are also common. Bone scan is
unreliable at detecting bony metastases when the
prostate specific antigen (PSA) is <20ng/ml. The
commonest pattern for prostate cancer on transrectal
ultrasound is of hypoechoic nodules. This pattern may
also be seen in prostatic hypertrophy or prostatitis.

the metaphysis and a dense slightly flattened femoral


head. There is no need for testing white cell count or
CRP as these are normal. In most cases the prognosis is
excellent and the only requirement is to avoid prolonged
walking or exercise. Progress is assessed by clinical
examination and serial radiological examinations. Where
there are severe changes containment of the femoral
head is the treatment of choice and this usually requires
a plaster cast or orthosis. Osteotomy of the pelvis or
femur is only rarely required.

Question 12
Significant protein-energy malnutrition is likely to lead
to increased postoperative complications and prolonged
recovery times. In assessing a patient's nutritional
status, which ONE of the following findings would be of
MOST concern?
a) bony shoulder girdle sharply outlined
b) weak grip strength

Question 10
Which ONE of the following atypical antipsychotic drugs
is optimal as first line therapy in managing the first
episode of schizophrenia in a patient?
a) Risperidone

c) an elevated white cell count in a patient with


multiple trauma
d) at the biceps skin fold, the dermis can be felt
between finger and thumb
e) recent weight loss of >5kg

b) Olanzapine
c) Quetiapine

Answer: B

d) Amuisulpride

Assessment of physiological function


importance, as weight loss without

e) Clozapine

is

of

most

Question 11

evidence of physiological abnormality is probably of no


consequence. Physical weakness (as shown by weak
grip strength), and poor respiratory muscle function
increases the risk of post-operative pneumonia. When
the bony shoulder girdle is sharply outlined, and tendons
(eg of biceps and triceps) are readily palpable, the
patient is likely to have lost more than 30% of total
protein stores. Loss of body fat is often readily apparent
on looking at the patient, and a biceps skinfold where
the dermis can be felt between finger and thumb
indicates that the body mass has less than 10% fat.
Metabolic stress can occur from trauma, sepsis or
ongoing inflammation. The trauma patient may well
have near normal stores of protein and fat to call on in
the short term.

Ara, 5 years old, presents with a painless limp. You


suspect Perthes' disorder. Which of the following
statements is CORRECT?

Question 13

Answer: B
Olanzapine is the drug of choice for treatment of first
episode schizophrenia, since it does not cause
extrapyramidal symptoms at therapeutic doses, and is
useful for combating affective symptoms, including
depression. Its major disadvantage is weight gain. The
other drugs listed carry a greater risk of extrapyramidal
symptoms and other side effects.

a) Hip mobility is usually reduced, particularly


adduction and external rotation
b) Ultrasound is required to make a definitive
diagnosis
c) The white cell count and C reactive protein
(CRP) are usually raised
d) Progress is assessed with serial radiological
examination

In Australia, all of the following are risk factors for


suicide, EXCEPT:
a) Schizophrenia
b) Family history of suicide
c) Social disadvantage
d) Female
e) Youth

e) Osteotomy is the treatment of choice


Answer: D
Answer: D
Perthes' disorder is most common in 4-8 year olds. On
examination hip mobility is reduced, particularly
abduction in flexion. Xray confirms the diagnosis
showing widening of the joint space, decalcification of

Males, particularly young men and elderly single men,


are at greater risk of suicide than women who generally
have better social support structures than their male
contemporaries.

Question 14

Question 16

A 15kg child with a known food allergy to peanuts


suddenly develops anaphylaxis. The RECOMMENDED
immediate management is:

All of the following groups are at higher risk than


average of depression, EXCEPT:

a) 0.1ml of Adrenaline 1:1000 by deep intramuscular


injection
b) 0.1ml of Adrenaline 1:10,000 by deep subcutaneous
injection
c) 0.15ml of Adrenaline 1:1000 by deep intramuscular
injection
d) 0.15ml of
injection

Adrenaline

1:1000

by

a) Women
b) Postpartum women
c) Young rural males
d) Urban males
e) Adolescents

subcutaneous

e) 0.15ml of Adrenaline 1:10,000 by deep intramuscular


injection

Answer: C
Adrenaline 1:1000. is recommended as it is readily
available, and this concentration contains 1mg of
adrenaline per ml. The recommended dose of 1:1000
adrenaline is 0.01mg/kg body weight by deep
intramuscular injection, so a 15kg patient would require
0.01 X 15 = 0.15ml i.m.

Question 15
Which ONE of the following is a mass population
screening test which has been demonstrated to reduce
cancer mortality significantly?
a) annual faecal occult blood testing in the over
50 age group
b) annual colposcopy in sexually active women
c) annual plasma CA125 in post menopausal
women
d) annual colonoscopy in siblings of patients
with colon cancer
e) 2 yearly mammography in women in the 3545 age group

Answer: A
There have been at least 3 randomised control screening
trials conducted which show that faecal occult blood
testing every 1-2 years in the over 50 year population
reduces the mortality from colorectal cancer by around
20%. Colposcopy is a diagnostic, not a screening test.
The appropriate screening test for cancer of the cervix in
sexually active women is the Papanicolaou (Pap) smear.
Plasma CA125 levels may be raised in asymptomatic
women with ovarian cancer but there is not, as
yet, any evidence for its benefit as a mass screening
measure. Colonoscopy for siblings of patients with
colorectal cancer may reduce their mortality, but this is
'selective' screening, by targeting a high-risk group.
While mammography in the 50-70 age group has been
shown to reduce mortality from breast cancer by around
30%, the benefit for women in the 40-50 age range is
quite small, and there is little evidence for benefit in still
younger women.

Answer: D
Men living in urban areas are not especially at risk of
depression. However, young men living in rural areas
are at greater risk because of social isolation. Women,
particularly in the postpartum period, and adolescents
are also more vulnerable to depression.

Question 17
14 month old Mark is brought in to see you. He has had
high fever and a mild runny nose for three days but has
still been quite active. Mark's temperature has returned
to normal today but he has now developed a red
maculopapular rash on his trunk. The MOST LIKEY
diagnosis is:
a) Measles
b) Chicken pox
c) Erythema infectiosum
d) Rubella
e)Roseola infantum

Answer: E
Roseola infantum is a viral infection usually affecting
children between the ages of 6-18 months. The patient
typically develops high fever up to 40 degrees Celsius,
but is otherwise not particularly unwell. There may be
mild cervical lymphadenopathy and pharyngitis. After
three days the temperature usually returns suddenly to
normal and the patient develops a red macular or
maculopapular non-desquamating rash which is truncal,
usually sparing the face and limbs. The rash abates
within two days.

Question 18
A 50 year old woman has had major abdominal surgery
yesterday. You are called to see her urgently as she has
symptoms of shock. Which ONE of the following
examination findings is of MOST concern?
a) the patient is restless and confused
b) Temperature 39.2 degrees Celsius
c) pulse 130, sinus tachycardia
d) urine output over past 4 hours of 120ml
e) BP 80/45 mm Hg

Answer: E

Answer: A

The above signs taken together suggest a picture of


shock, probably septic shock. Hypotension (defined as
systolic BP <90mm Hg, or >40mm Hg fall from baseline
level) is a sinister development and requires urgent
attention. It is often a LATE manifestation of circulatory
failure. Thus it is the most alarming of these findings,
and the one most indicative of the urgency of this
situation.

The clinical picture suggests respiratory infection with


Bordetella pertussis. The history of cyanosis associated
with the coughing suggests the need for admission to
hospital but, in addition, children under 6 months of age
usually require admission for pertussis because of the
risk of complications. Complications include respiratory
arrest,
bacterial
pneumonia
and
encephalitis.
Salbutamol has not been shown to be helpful in a child
of this age and is of no benefit. Immunisation at a later
date should be encouraged even if the child has had
pertussis. Erythromycin is not curative but may reduce
infectivity.

Question 19
Arthur is a 74 year old man who presents reluctantly
because his wife feels 'he has not been himself since he
came home and is not sleeping properly.' His appetite is
poor and he has lost 6 kg in weight since discharge from
hospital eight weeks ago following treatment for
myocardial infarction. He has lost interest in his hobbies
and is not taking his medications. He has been a type 2
diabetic for 12 years. On examination he shows signs of
mild cardiac failure. The result of an HbA1C test is
10.5% (target <7%). Which ONE of the following is the
MOST likely cause of Arthur's condition.
a) Senile dementia

Question 21
Simon aged 4 months is diagnosed with pertussis. What
is the MOST APPROPRIATE management of Simon's
parents to reduce their risk of infection?
a) Immediate
pertussis

booster

immunisations

for

b) A 10 day course of erythromycin

b) Uncontrolled diabetes

c) Commence a 3 dose pertussis revaccination


schedule

c) Depression

d) Arrange nasopharyngeal swabs

d) Silent reinfarction

e) Immediate immunisation
immunoglobulin

e) Chronic fatigue syndrome

Answer: C
Arthur is showing several typical features of depression,
viz insomnia, loss of interest in pleasurable activities,
loss of appetite and loss of weight. Depression following
myocardial infarction is well recognised and increases
the risk of complications and death. Such patients are
less likely to observe instructions regarding medications,
making their situation worse. Arthur's diabetes is poorly
controlled, as shown by the elevated HbA1C level but
this is insufficient to explain his condition. The other
options could contribute to the clinical picture shown by
Arthur but are less likely to be the cause of his
condition.

Question 20
Aidan, a 3 month old boy, presents with paroxysms of
coughing associated with cyanosis, lethargy and poor
feeding for several days. On examination, between
episodes of coughing, he is afebrile and examination is
normal. What is the NEXT step in management?
a) Admission to hospital
b) Nasopharyngeal aspirate and review in 24
hours
c) Immunisation at this visit with DTP and
review in 24 hours
d) Erythromycin syrup and review in 24 hours
e) Trial of salbutamol by mask

with

pertussis

Answer: B
All household contacts of patients with pertussis should
receive erythromycin for 10 days to prevent further
spread of the disease. Cotrimoxazole is recommended
for those intolerant of erythromycin. An initial or booster
dose of pertussis vaccine should be considered for
children aged up to 8 years. This is not required for
adults nor is it necessary for them to recommence a full
vaccination schedule. Babies under 2 months of age who
have been exposed are best vaccinated early. Passive
immunisation with pertussis immunoglobulin is not
effective in the prevention of pertussis. Isolation is not
required for household contacts.

Question 22
Which of the following conditions is the COMMONEST
anxiety disorder encountered in general practice?
a) Generalised anxiety disorder
b) Phobic disorder
c) Obsessive compulsive disorder
d) Panic disorder
e) Post-traumatic stress disorder

Answer: A
Generalised anxiety disorder is the commonest anxiety
disorder seen in general practice. It has an insidious
onset, usually having been present 5 to 10 years before
diagnosis. It occurs more commonly in women and the
frequency increases with age.

Question 23

d) Amitriptyline

Which of the following is NOT a diagnostic feature


accompanying anxiety in generalised anxiety disorder
(GAD)?

e) Any of the above

a) Restlessness

Answer: E

b) Fatigue

There is a well defined role for short-term use of a


hypnotic medication in a situation like sudden
bereavement. None of the drugs listed stands out as the
best hypnotic with few side effects. Temazepam is the
most often prescribed hypnotic in Australia. However,
benzodiazepines have generally fallen from favour
because of their addictive properties if taken for more
than several weeks. Zopiclone and zolpidem share some
of the properties of benzodiazepines but have fewer
adverse
effects.
Amitriptyline
is
a
tricyclic
antidepressant which is a useful hypnotic in lower doses
than are used for depression.

c) Difficulty concentrating
d) Muscle tension
e) Loss of appetite

Answer: E
Loss of appetite is not a feature of GAD. The DSM-IV
criteria include 3 or more of the following in addition to
anxiety and worry: restlessness, fatigue, difficulty
concentrating, irritability, muscle tension, and sleep
disturbance.

Question 26
Insomnia is defined as inability to:

Question 24

a) Fall asleep

Martin, aged 50 years, complains of insomnia for two


weeks. He says this relates to anxiety about problems at
work which surround the end of the financial year. While
he is describing his problem, he is excited and agitated.
On examination, his pulse rate is 75/min and BP 135/95
mm Hg. Which of the following behavioural therapies
would be MOST helpful to Martin?

b) Maintain sleep

a) Cognitive therapy
b) Sleep restriction therapy
c) Stimulus control therapy
d) Relaxation therapy
e) Interpersonal therapy

c) Sleep at normal times


d) Obtain good quality sleep
e) Obtain enough sleep

Answer: D
Insomnia is the commonest sleep disorder and is
defined as poor quality sleep which often results in
daytime symptoms, including fatigue, irritability,
problems with concentration and memory, and feeling
unwell. The other options describe features of various
insomnia syndromes but do not define the overall
problem.

Answer: D
The successful treatment of insomnia depends on both
behavioural and pharmacological approaches. Relaxation
therapy would be the most useful behavioural therapy
for Martin, because he displays exaggerated arousal emotional, cognitive and physiological, shown by his
mental approach when describing the problem and his
physiological response in terms of pulse rate and BP.
Progressive muscle relaxation aims to reduce somatic
arousal and attention focussing techniques (e.g. on
tranquil situations) to reduce cognitive and emotional
arousal. (See reference for descriptions of other options,
except interpersonal therapy which is not an accepted
form of behavioural therapy.)

Question 27
Miriam, aged 67 years, presents with insomnia for two
weeks. She has tried several over-the-counter
medications but has not received any benefit. Further
questioning reveals that she is feeling unusually worried
about her health, is also irritable and restless and has
generalised muscle tension. You have known Miriam for
5 years since you joined this practice. She has
previously complained of 'arthritis' of the hips and
knees, back pain and gastro-oesophageal reflux on
various occasions. Which of the following drugs would be
MOST suitable for long-term treatment Miriam.
a) Amitriptyline

Question 25
Doris is a 74 year old woman whose husband has died
suddenly a week ago. She seeks your help in dealing
with insomnia which has been troubling her since her
husband's death. She has tried an over-the-counter
preparation which she obtained at the local pharmacy
but has not found it helpful. Which of the following drugs
would you offer to prescribe for Doris?
a) Temazepam
b) Zopiclone
c) Zolpidem

b) Diazepam
c) Venlafaxine
d) Oxazepam
e) Fluoxetine

Answer: C
Miriam is suffering from generalised anxiety disorder
(GAD). Fluoxetine and paroxetine are selective serotonin
reuptake inhibitors (SSRI) which have not been shown
to be effective in the treatment of GAD. Amitriptyline is

a tricyclic antidepressant; diazepam and oxazepam are


benzodiazepines. All three drugs can be used for
treating anxiety, but have major disadvantages in terms
of side affects and are unsuitable for long-term
treatment of GAD. Venlafaxine is a serotonin and
noradrenaline reuptake inhibitor (SNRI) which has been
shown to be effective in the long-term treatment of
GAD. However, it is important that patients also receive
appropriate psychotheraphy, viz-cognitive behavioural
therapy (CBT).

Question 28
The prevalence of insomnia in the Australian community
is about:
a) 10%
b) 20%
c) 30%
d) 40%
e) 50%

b) Inadvertently play down their anxiety


c) More often present with somatic complaints
d) Are misdiagnosed as having depression
e) Tend to keep away from doctors

Answer: C
Patients with GAD are frequently not diagnosed for
years because they tend to present with associated
somatic complaints, such as asthma, back pain,
migraine, allergies, and gastrointestinal disorders and
often consume a large amount of medical resources
before the diagnosis is made. They are not deliberately
concealing things from their doctors - they simply do not
appreciate the significance of their feelings. Patients
with GAD may also have clinical depression which can
further complicate the issue.

Question 31
All of the following are effective first line topical
treatments for severe dandruff EXCEPT:

Answer: C

a) Selenium sulphide

The prevalence of insomnia in Australia is about 30%.


The situation is similar in other industrialised nations.
Male sufferers outnumber females by 1.3 to 1 in the
40+ years age group. Other factors which increase the
prevalence of insomnia are old age, unemployment and
lower socio-economic status. The majority of patients
have a co-existing disorder, such as depression or
generalised anxiety, and often present with fatigue or
daytime sleepiness rather than insomnia.

b) Zinc pyrithione

Question 29
Which of the following is a cause of primary insomnia?
a) Obstructive sleep apnoea
b) Restless legs syndrome
c) Behavioural conditioning
d) Sleep phase disorder
e) Bereavement

c) Betamethasone
d) Miconazole
e) Ketoconazole

Answer: C
Dandruff, pityriasis capitis and seborrhoeic dermatitis
are related conditions on a continuum of severity.
Pityrosporidium ovale is a fungus which has been
implicated
in the development of these conditions. Eradication of
the fungus has been shown to relieve the symptoms of
the condition. Selenium sulphide, zinc pyrithione,
miconazole and ketoconazole are all first line treatments
which can be used once weekly to daily depending on
severity. Topical application of betamethasone lotion is
effective but only considered if there has been little
response to the first line treatments.

Answer: C

Question 32

In primary insomnia there is no pre-existing cause of


sleep disorder as there is, for example, in obstructive
sleep apnoea, restless legs syndrome, sleep phase
disorder
and
bereavement.
With
behavioural
conditioning or behaviours impairing sleep the patient
has developed a habit of doing things immediately
before bedtime which are not conducive to sleep, such
as sitting in bed watching television. With the other
options there is another established condition or
situation which disrupts sleep. Hence insomnia is
regarded as secondary in these cases.

Amy, a young girl, is worried about a rash on her face


which has been there for a few weeks. It is mildly itchy
and has slowly increased in size. On examination it is a
circular lesion which looks scaly around the edges with
the center appearing normal(see figure).
The MOST LIKELY diagnosis is:
a) Tinea corporis
b) Impetigo
c) Pityriasis versicolor

Question 30
Patients suffering from generalised anxiety disorder
(GAD) often go undiagnosed for years because they:
a) Deliberately conceal their anxiety

d) Psoriasis
e) Dermatitis

Answer: A
In children, tinea commonly involves the hair, face and
body. Tinea corporis is recognised by its scaly,
erythematous edge with clearing centre. Dermatitis and
psoriasis can mimic tinea and where the clinical picture
is uncertain a fungal scraping may confirm tinea.
Impetigo may present with blisters that leave a brown
crust, or erosions with yellow crusts but without
blistering. Pityriasis versicolor presents as well
demarcated pale or tan-coloured macules usually on the
upper trunk.

Question 33
After referral to a paediatric oncologist, Bronwyn is
diagnosed as having acute lymphoblastic leukaemia
(ALL). Both her parents are very distressed and visit you
to find out more about this condition. Which of the
following
statements
about
acute
lymphoblastic
leukaemia is INCORRECT?
a) 75% of children with ALL are cured
b) An initial intense course of treatment lasting
approximately 4-6 weeks is required
c) Intrathecal therapy is required only if the
disease has spread to the spine
d) Relapse rates are of the order of 30%
e) Remission rates are of the order of 95%

Answer: C
Intrathecal therapy is now given to all children with ALL.
Prior to blanket intrathecal therapy, up to 60% of
children had a relapse due to CNS disease. The use of
blanket CNS therapy has reduced the CNS relapse rate
to less than 10%.

Question 34
Mary is an attendant at a local accommodation centre
and has an intensely itchy rash on her wrists and arms
that has been present for the past few days (see figure).
She has recently bought a new watch and wonders if
this is the cause of the problem. The MOST LIKELY
diagnosis is:
a) Papular urticaria
b) Tinea
c) Contact dermatitis
d) Eczema
e) Scabies

Answer: E
Scabies is a skin infestation by the mite Sarcoptes
scabei. It is generally spread by skin to skin contact
such as in crowded areas, poverty, sexual contact and
casual contact. The mite can live for 2 days outside the
human body, so infection by contact with bed linen and
other infected material is possible. Intense itch is
characteristic of the condition - if it is not itchy, it is not
scabies. Distinct erythemato-papular itchy nodules are
due to an allergic reaction to the mite, its faeces and its
larvae. 0.5-1.0cm "burrows" can often be found on the

fingers and wrist. Contact dermatitis to her watch would


produce a local contact dermatitis.

Question 35
Which of the following statements
immunisation is CORRECT?

about

routine

a) If more than 3 months elapses between


triple antigen injections the series should be
restarted to obtain adequate immunisation
b) The first triple antigen injection should
always be half dose (1/2 cc) to reduce allergic
reactions
c) A convulsion following acellular pertussis
triple antigen vaccine (DTPa) does not
contraindicate its further use
d) Tetanus booster injections
repeated every 3 years

should

be

e) Measles vaccine should be given at 6 months


of age to protect the child as soon as possible

Answer: C
A convulsion, in the presence or absence of fever,
occurring
after
DTPa
vaccination,
does
not
contraindicate completion of the course with DTPacontaining vaccine. The only adverse events which
contraindicate further doses of pertussis containing
vaccine are:
encephalopathy within 7 days and
an immediate severe allergic reaction.
It is now recommended that, after a full primary course
of childhood vaccination, a single booster of tetanus
toxoid is administered at age 50 years only (unless
required sooner for managing a tetanus-prone wound).
Measles,
mumps
and
rubella
immunisation
is
recommended at 12 months of age. Triple antigen (DPT)
immunisation is recommended at 2, 4, 6 and 18 months
of age. The first dose should not be reduced.

Question 36
The MOST appropriate treatment
atrioventricular (AV) heart block is:

for

first

degree

a) An artificial pacemaker
b) Isoprenaline hydrochloride (Isuprel)
c) Atropine
d) Digoxin
e) Requires no treatment

Answer: E
First degree AV block often does not require any
treatment. Acute treatment of
extreme bradycardia or second degree AV block (Mobitz
type II) may require atropine or isoprenaline, but
temporary pacing is the preferred treatment. Permanent
pacing is recommended for distal block (Mobitz type 2)
because of frequent early progression to third-degree
atrioventricular block. Most patients with third degree
(complete) AV block will require permanent cardiac

pacing. Drugs such as digoxin may be the cause of an


AV block and should be ceased or the dose reduced.

Question 37
Melissa, a 49 year old woman, presents with loss of
libido which has been worsening over the past 12
months. She takes a combined oral contraceptive tablet,
and a selective serotonin reuptake inhibitor (SSRI) for
depression. She is a non smoker but drinks 1-2 glasses
of wine a night. Which of the following advice would you
offer Melissa to help her regain her libido?
a) Change to
contraceptive

b) Change to
contraceptive

an

progestagen

only

oral

oestrogen

only

oral

Question 39
A 61 year old farmer with a history of hypertension
presents with an amnesic episode of sudden onset.
When assessed he is perplexed and bewildered, and
repeatedly asks where he is and how he comes to be
there. He has short term memory loss but knows his
identity and is fully conscious. He recovers from his
memory loss after 6 hours. Neurological examination is
normal. His blood pressure is 165/96 sitting. The MOST
LIKELY diagnosis is:
a) Hypertensive encephalopathy
b) Richardson-Steele syndrome
c) Alzheimer's disease
d) Transient global amnesia
e) Complex partial seizure

c) Reduce her alcohol intake


d) Change antidepressant medication

Answer: D

e) Undertake cognitive behavioural therapy

Answer: D
Various medications may cause loss of libido, including
antidepressants like the SSRIs. The remedy is to change
to a different class of antidepressant. Excess alcohol
consumption can have a deleterious effect on libido,
however Melissa's intake is within recommended limits
for adult females. Changing the oral contraceptive would
be unlikely to help. Cognitive behavioural therapy would
be the next step if suitable pharmacotherapy cannot be
established.

Question 38
In assessing a patient for adult-onset asthma, which of
the following is FALSE?
a) There is usually a history of past or present
cigarette smoking

The most likely diagnosis is transient global amnesia.


This is a syndrome in which a previously well person
suddenly becomes confused and amnesic. The attacks
are usually spontaneous. The patient appears
bewildered and repeatedly asks questions about present
and recent events. Orientation for person and
sometimes place is preserved but recent memory is
impaired and the patient cannot recall new information
after a few minutes delay. Attacks usually last 24 to 48
hours. Recovery is complete and recurrence occurs in
about 20% of patients. The cause is a mystery. Complex
partial seizures are brief and the individual loses
conscious contact with the environment. Post-ictal
amnesia is common if the seizure becomes generalised.
Richardson
Steele
Syndrome
(or
progressive
supranuclear palsy) resembles Parkinson's disease in
that there is a disturbance of balance and gait, with
rigidity of the trunk and neck muscles. Alzheimer's
disease is a progressive form of dementia which does
not resolve. Hypertensive encephalopathy is an acute
syndrome where severe hypertension is associated with
headache, vomiting, convulsions, confusion, stupor and
coma.

b) There is usually a long history of atopic


disease
c) Attacks can be triggered by chemical and
physical agents
d) The asthma tends to be more chronic than
episodic
e) The associated cough
suffusing and non-productive

can

be

harsh,

Question 40
Which of the following factors would NOT be a poor
prognostic indicator for joint replacement in hip
osteoarthritis in a 65 year old man?
a) Age
b) A BMI of 33
c) Associated diabetes

Answer: B

d) Presence of osteoporosis

Adult onset asthma is non-atopic, although patients can


have attacks precipitated by aspirin, viruses, cold air
and coughing itself. A history of smoking is nearly
ubiquitous, but the connection to the development of
adult-onset asthma is not yet understood. Patients tend
not to have long spells of being 'puffer free', but regular
inhaled steroids are very effective in controlling the
condition. The cough can keep the patient awake all
night, along with the rest of the family.

e) A previous joint replacement

Answer: A
Poor prognostic indicators for joint replacement include:
age less than 50 years, diabetes, obesity, high demand,
poor bone quality, previous joint surgery and poor
general health. Since Michael is 65 years old his age
would not be a poor prognostic indicator.

Block 2
Question 1

Question 2

All of the following tests should be part of a routine


initial (first trimester) antenatal screen EXCEPT:

Martin is 5 years old. His mother reports he has been


waking her at night screaming in apparent fear. She is
extremely worried. Although Martin is obviously
frightened during these episodes, he does not seem fully
awake, and she is unable to comfort him or wake him
fully. He is sweaty, tachypnoeic and tachycardic. Martin
does not recall these episodes in the morning. He is
MOST LIKELY suffering from:

a) Full blood examination


b) Blood group and antibodies
c) Dating ultrasound
d) Syphilis serology
e) Hepatitis B surface antigen

a) Nightmares
b) Thyrotoxicosis
c) Night terrors

Answer: C
Although there are no official national antenatal
screening guidelines in Australia, there are a set of
recommendations by the Royal Australian and New
Zealand College of Obstetricians and Gynaecologists.
Tests to be offered at the first trimester initial antenatal
visit include the following:
1. Blood group and antibody screen
2. Full blood examination
3. Rubella antibody status
4. Syphilis serology
5. Midstream urine culture
6. Hepatitis B serology
7. HIV Antibody
8. Hepatitis C serology

d) Panic disorder
e) Seizure disorder

Answer: C
Night terrors are a disorder of arousal from NREM sleep,
usually occurring in 3 to 8 year olds. The child wakes
only partially, cannot be fully roused or comforted, and
will have no recall of the episode. Autonomic symptoms
as described are common. With nightmares, the child
wakes fully, and frequently has full recall of dreams.
Night terrors may be precipitated by anxiety or an
experience which has frightened the child prior to sleep.

Question 3
All of the following are associated with the timing of
secondary sexual maturation EXCEPT:

9. Others to consider:

a) body weight

10. Cervical cytology

b) adequate sleep

11. Haemoglobinopathy screen

c) phenotype

12. Varicella antibody

d) optic exposure to sunlight

Where the blood group has already been performed it


does not need to be repeated. However, the antibody
screen should be repeated at the beginning of each
pregnancy, and all Rhesus negative women and all those
who have had isoimmunisation of any type in the past
should be re-screened. Although a previous high rubella
antibody titre is generally used to justify exclusion of
this investigation from first visit testing, there is some
evidence that antibody levels may decline after rubella
immunisation, especially since antibody levels are rarely
boosted by exposure to wild viruses in the community.
All pregnant women should be offered hepatitis C and
HIV screening at the first antenatal visit and if necessary
after exposure to risk of infection. It is imperative that
the woman is provided with appropriate counselling as
to the limitations of the testing and the implications of
both positive and negative findings. Documented normal
cervical cytology within the preceding 18 months may
be used to delay repeat screening if there is no clinical
indication for another Papanicolaou smear. Routine
vaginal ultrasound during the first trimester is not
recommended except where the viability of the
pregnancy is in question such as when the woman has
had first trimester bleeding.

e) latitude of habitation

Answer: E
Girls must attain a critical body weight (irrespective of
height) before sexual maturation will begin. A body
weight of 38 - 48 kilograms must be achieved before
menses begins and a proportion of body fat of 16-24%
is required to sustain ovulatory cycles. Other critical
elements to the timing of secondary sexual maturation
include adequate sleep and vision. Blind girls have
delayed menarche and blind boys have delayed
spermatogenesis
and
ejaculation.
Chromosomal
abnormalities such as Turner's syndrome result in
premature ovarian failure and lack of secondary sexual
maturation. Latitude has no impact.

Question 4
Eve is 9 weeks pregnant. She presents with a history of
24 hours of intermittent dark blood loss staining her
underwear. On vaginal examination, the uterus is the
expected size and the cervical os is closed and nontender. You should advise her that:
a) she is likely to have miscarried
b) approximately half of all pregnant women
suffer bleeding during the first trimester
c) she requires admission to hospital for a D&C
d) there is a higher incidence of congenital
malformations in fetuses where bleeding
occurred in the first trimester
e) she should have an ultrasound examination

Answer: E
Eve has a threatened abortion as the cervical os is
closed and there is no history of passage of products of
conception. This occurs in 25% of pregnancies and half
of these progress to spontaneous abortion. For those
who carry the pregnancy to term, there is no increased
risk of congenital malformation. An ultrasound
examination would be helpful, as it could demonstrate
the presence (or absence) of a foetus appropriately
sized for dates. The demonstration of a foetal heart is
very reassuring. At 9 weeks, when a foetal heartbeat is
present and not slow, 90% of pregnancies will continue
despite bleeding.

Question 5
Samantha is 18 months old. She is usually well, with no
significant past history, and is up to date with her
childhood vaccinations. Samantha has been miserable
and unwell for 24 hours with a clear runny nose, and a
fever of 39 to 40 degrees Celsius. She has no cough, no
diarrhoea or vomiting, and is drinking well, even though
her appetite for solids is decreased. Suddenly Samantha
has a generalised tonic-clonic seizure, lasting 3 minutes.
Which of the following statements regarding febrile
convulsions in children is TRUE?
a) most febrile convulsions are focal or partial
seizures
b) There is a 10% chance of developing
epilepsy
c) the most common cause is CNS infection
such as meningitis
d) 30-50% of children have recurrent episodes
e) Peak incidence is 2-3 years of age

Answer: D
Febrile convulsions may be recurrent in 30 - 50% of
cases, but this does not usually worsen long term
prognosis. Most febrile convulsions occur in children
aged 3 months to 5 years, the peak incidence being
between 6 and 20 months of age. More than 90% of
seizures are generalised. Acute upper respiratory tract
infections are the most common cause of febrile
convulsions. In children under the age of two years if
the cause of the febrile convulsion is not obvious, a
lumbar puncture must be performed to exclude CNS
infections. Only rarely do febrile seizures lead to

epilepsy, the incidence being about 2 - 3%. The chance


of developing epilepsy is increased if there is a family
history of epilepsy, the child has a pre-existing
neurological
abnormality, the child is under one year of age, the
seizure is prolonged (greater than 15 minutes) and/or
has complex or focal features, or if the child fits more
than once in one day.

Question 6
Vera, aged 61 years, has noticed a swelling in the front
of her neck, which is enlarging quite rapidly. She is
otherwise well, without symptoms of hypothyroidism or
hyperthyroidism. On examination she has a multinodular
goitre, with a dominant nodule of 4 cm diameter in the
right upper pole of her thyroid gland. Of the following
investigations, which is the MOST important in assisting
diagnosis?
a) Thyroid function tests
b) Thyroid ultrasound
c) Fine needle biopsy
d) Thyroid isotope scan
e) CT scan

Answer: C
The incidence of malignancy in a dominant thyroid
nodule is approximately 7%, and fine needle biopsy is
the appropriate investigation to exclude malignancy.
Vera has several features consistent with a thyroid
malignancy, including a rapidly growing solitary lump,
her age (over 60 years) and the fact that the dominant
nodule is >3cm in diameter. Detection or exclusion of
malignancy is the MOST important issue initially. Thyroid
function tests must also be performed to assess
glandular activity, but they
do not help in determining malignancy. Ultrasound scan
adds little to clinical examination, but may be used to
guide the biopsy needle. CT scan may be useful to
assess retrosternal extension of the thyroid, but
currently Vera has no symptoms of this problem.
Nuclear medicine scan is useful to determine the
functional status of a nodule but carcinoma cannot be
excluded on the basis of a radionuclide scan.

Question 7
All of the following are true of ectopic pregnancy
EXCEPT:
a) an ectopic pregnancy is one where there is
implantation outside the uterine cavity
b) the primary risk factor for ectopic pregnancy
is a history of salpingitis
c) the prevalence of ectopic pregnancies has
decreased over the last 20-30 years
d) ectopic pregnancy is one of the leading
causes of maternal mortality
e) less than 50% of women who have an
ectopic will subsequently have a successful
normal pregnancy

Answer: C

Answer: D

The prevalence of ectopic pregnancies has increased by


up to six-fold since 1970. The reason behind this
increase may be an increase in the prevalence of
sexually transmitted diseases and tubal sterilizations, or
that women are conceiving later in life with a
corresponding increase in the risk of tubal problems.
Ectopic pregnancy remains one of the leading causes of
maternal mortality. Probably because of underlying tubal
problems, <50% of women who have an ectopic will go
on to have a successful intrauterine pregnancy.

The rapid growth of a painful swelling in the region of


the parotid gland, suggests a malignant process. 1520% of parotid tumours are malignant, and in Australia
the most common parotid malignancy is metastatic
melanoma or squamous cell carcinoma from a skin
primary in the head and neck region. Sjorgrens
syndrome produces persistent and painful parotid
swelling, but growth is usually slow; associated
symptoms include dryness of the mouth and eyes.

Question 8
Regarding urinary tract infection (UTI) in children, which
of the following statements is INCORRECT?
a) Diagnosis is based on a colony count of 108
organisms/litre from a bag urine specimen
b) Approximately 5% of children will have had
a UTI by the age of 10 years, girls more often
than boys

Pleomorphic adenoma is the commonest parotid


tumour; it is benign grows slowly and rarely causes
pain. Salivary duct calculi produce intermittent swelling
of the salivary gland, often during eating; they are more
common in the submandibular gland. Acute sialadenitis
is salivary gland inflammation presenting as rapid
development of erythema, pain, tenderness and swelling
often due to infection, most commonly with Staph
aureus.

Question 10

d) Even one UTI in a child <4 years of age


needs a renal ultrasound and micturating
cystourethrogram (MCU)

A 26-year-old woman (G3P2), who has reached 30


weeks gestation, telephones you to say that she has had
an episode of bright red vaginal bleeding. It occurred
several hours ago and she has had no associated
symptoms. She estimates she lost about a teaspoonful
of blood. What would be the MOST APPROPRIATE
IMMEDIATE management?

e) Renal scarring on an isotope scan increases


the risk of early onset hypertension

a) Advise rest at home and to call back if


bleeding recurs

c) Infants often present with non-specific fever


and
failure
to
thrive,
irritability
and
hypothermia

Answer: A
By 10 years of age, 2% of boys and 8% of girls will have
had at least one UTI. About 25% of these will have
vesicoureteric reflux (VUR). Escherichia coli is
responsible for more than 80% of childhood UTIs. A bag
specimen of urine is only of value excluding a UTI as the
sample is easily contaminated. However a clean catch or
midstream sample showing >108 colony-forming units
per litre is highly suggestive of a UTI. Most children,
especially those under 4 years of age with a confirmed
UTI require further investigation. This will always involve
an ultrasound and for younger patients, an MCU which is
necessary to define the grade of VUR. In the case of
high grade VUR or a febrile UTI, radioisotope scan will
be done 3 months after the UTI has resolved to assess
renal scarring. If the scan is abnormal the child is at risk
of hypertension and should have blood pressure and
urinalysis monitored yearly.

Question 9
Betty, aged 62 years, first noticed a swelling just below
and in front of her left ear, a month ago. The overlying
skin is normal and the swelling has enlarged rapidly and
become painful. She has some "pins and needles" in the
skin of her left cheek but is otherwise well. Which of the
following is the MOST LIKELY diagnosis?
a) Sjogren's syndrome
b) Pleiomorphic adenoma
c) Salivary duct calculus
d) Metastatic skin cancer
e) Acute sialadenitis

b) Advise her to attend immediately for further


evaluation
c) Reassure her that spotting without other
symptoms is unlikely to be significant at this
stage
d) Send her directly for urgent ultrasound
examination
e) Book an ambulance as this may be a
warning of an impending massive bleed

Answer: B
Although this bleeding may have a benign cause, it is
important to recognise that she may have a placenta
praevia, or have had a placental abruption. Both of
these conditions require urgent attention because of the
risk of maternal and fetal morbidity and mortality. The
patient should be examined (vital signs, examination of
the uterus and foetus and a sterile speculum
examination) to assess the nature of the bleeding and
whether or not the cervical os is closed. An ultrasound
examination will be necessary to assist in confirmation
of the diagnosis. It should be done only after the patient
has been assessed and is stable.

Question 11
Brian is 9 years old. He has been complaining of itchy
hands for over a week and is constantly scratching. On
inspection, you note a lumpy excoriated rash between
several of his fingers on both hands. You suspect Brian
may have scabies. Which of the following statements
regarding scabies is CORRECT?

a) History taking must include recent travel and


contacts with animals including pets
b) Family and close contacts only require
treatment if they are symptomatic
c) Persistent itch 5 days
indicates treatment failure

after

treatment

d) Permethrin 5% cream may be used to treat


scabies in a child of Brian's age
e) Brian should be excluded from school for
seven days, or until the itch has subsided

completely ignored. While clinical examination is always


important, detailed neurological assessment is not
appropriate initially.

Question 13
In women of reproductive age the most common causes
of vaginal irritation and discharge in DESCENDING order
of occurrence are:
a) Bacterial vaginosis, candidiasis, trichomonas
b) Bacterial
simplex

vaginosis,

candidiasis,

herpes

Answer: D

c) Candidiasis, bacterial vaginosis, Chlamydia

Permethrin 5% cream is a suitable treatment option in


children over the age of 6 months. Sarcoptes scabiei,
the mite which causes scabies, is spread by close human
contact and is not from animals. All family members and
close contacts should be treated, regardless of
symptoms. Patients and their families should be advised
that the itch does not resolve immediately and may take
three weeks to subside. It is unnecessary to repeat
treatment for scabies during this time based on
persistence of itch. Brian needs to be kept away from
school only until he has begun appropriate treatment.

d) Candidiasis, Chlamydia, herpes simplex

Question 12
Margaret, aged 46 years, is distressed and agitated. She
is holding her head, and complaining of 'the worst
headache I have ever had'. This headache began
suddenly 2 hours ago. She has vomited twice, and is
photophobic. Her temperature is 37.8 degrees Celsius,
her pulse 110/min and her blood pressure 140/80 mm
Hg. Which of the following is the MOST appropriate
action?
a) give intravenous fluids, metoclopramide
(maxolon) then soluble aspirin
b) request an emergency CT scan of her head
c) perform or refer for urgent lumbar puncture
d) do not leave the patient unattended in your
rooms
e)
perform
assessment

full

clinical

e) Chlamydia, bacterial vaginosis, trichomonas

Answer: A
Bacterial vaginosis is the most common cause of vaginal
irritation and discharge. It is often misdiagnosed by
women and their doctors as "thrush". Proven candidal
infections are the second most common, accounting for
around 35% of infections. Trichomoniasis is a sexually
transmitted disease that causes vaginitis. Chlamydia
and gonorrhoea infect the cervix and urethra and do not
cause "vaginitis" per se. Herpes may cause vaginal pain
and irritation if vesicles occur in the vagina but is not
commonly associated with discharge.

Question 14
All of the following may be extra-intestinal features
associated with Crohn's Disease EXCEPT:
a) Episcleritis
b) Ankylosing spondylitis
c) Hepatic steatosis
d) Erythema nodosum
e) Dermatitis herpetiformis

neurological

Answer: B
The sudden onset of 'the worst headache' the patient
can ever recall, accompanied by vomiting, must raise
the suspicion of subarachnoid haemorrhage.(SAH).
While this is rare (and only 1 patient in 8 who presents
with sudden extreme headache will have a SAH) it is a
diagnosis which must not be missed. SAH occurs most
frequently in patients under 65 years of age, the
majority being in the fourth decade. Women are more
frequently affected than men. All patients with this
clinical picture require investigation commencing with
emergency head CT scan to exclude SAH. Lumbar
puncture may be undertaken as the next investigation if
the CT scan does not confirm SAH. Blood or
xanthochromia in CSF may be detected in a smaller
SAH, and meningitis may be excluded. Migraine may be
considered after exclusion of SAH and meningitis.
Suspected drug seekers should not be left alone in your
rooms. However, this is not a typical presentation from
a drug-seeker, but that possibility should not be

Answer: E
Up to 35% of patients with inflammatory bowel disease
have an extra intestinal manifestation. The more
common
are
erythema
nodosum,
pyoderma
gangrenosum, psoriasis, arthritis, ankylosing spondylitis,
episcleritis, iritis, conjunctivitis, fatty liver (hepatic
steatosis), primary sclerosing cholangitis, cholelithiasis,
nephrolithiasis
and
thromboembolism.
Dermatitis
herpetiformis is associated with coeliac disease.

Question 15
Michael, aged 12 years, sustained a blow to his left
temple when he fell while climbing a tree. He was
dazed, but able to recount what had happened. An hour
later he complained of an increasingly severe headache,
vomited once, and then was brought to hospital. His
pulse is now 54 bpm, BP 130/90 mm Hg and he is
drowsy and confused. His left pupil is larger than his
right. Which is the MOST appropriate advice to give
Michael's parents? Michael:

a) will need an urgent CT scan of his head to


confirm the diagnosis
b) has a severe concussion and must be
observed closely overnight
c) requires urgent neurosurgery to reduce the
chance of disability or death

children a tonsillectomy would not be recommended. In


all of the other cases, tonsillectomy is likely to be
recommended. Claire's four episodes of (proven)
tonsillitis have been accompanied by a worrying if not
sinister symptom. Steven's quinsy is a clear indication
for tonsillectomy, and Olive may have a malignancy.
Peter's snoring and sleep apnoea also justifies
tonsillectomy.

d) will be monitored in ICU until stabilized for


surgery in 24 hours
e) is gravely ill and has a 50% chance of
survival

Answer: C
The history of the injury is highly suggestive of an
extradural (epidural) haematoma. Although Michael did
not lose consciousness initially, his condition has
deteriorated rapidly and significantly. He is now
bradycardic, hypertensive and his pupil is dilated on the
side of the injury. His level of consciousness is also
deteriorating. An extradural haematoma will result in
death if not evacuated promptly. There is approximately
75% chance he will have a fracture overlying the
haematoma, but skull x ray is not indicated. There may
be 'concussive' injury to the underlying brain but the
life-threatening factor is the raised intracranial pressure
from the extradural haematoma. While it would be ideal
to have a CT scan, the rapidly deteriorating condition
means that emergency surgery should not be delayed.
Michael is clearly gravely ill, but the mortality from
epidural haematoma is 9% for obtunded patients and
20% for those who are comatose prior to surgery.
Prognosis is better for young patients, but deteriorates
with other associated intracranial injuries and with delay
between injury and surgical intervention.

Question 16
Tonsillectomy would be recommended for each of the
following patients EXCEPT:
a) Claire, aged 3, has had four episodes of
tonsillitis in the last 12 months, each time
associated with a febrile convulsion.
b) Sarah aged 4 years, has had six episodes of
a sore throat in the last 12 months and now
has bilateral large tonsils and enlarged cervical
lymph nodes
c) Steven aged 25 years, has recently required
incision and drainage of a quinsy (peritonsillar
abscess)
d) Olive aged 63 years, has noticed that her
right tonsil has enlarged recently. She also has
an enlarged cervical lymph node on the right
e) Peter, aged 10 years, snores loudly,
sometimes "stops breathing" at night and is
sleepy at school. He has large tonsils.

Answer: B
It is not unusual for young children to have several
episodes of 'sore throat' each year. These could be due
to a number of conditions including pharyngitis,
laryngitis, tonsillitis (either viral or bacterial), or
exposure to irritants like cigarette smoke. Children such
as Sarah in the preschool age group frequently have
large tonsils, adenoids and cervical glands as part of the
normal growth pattern of lymphoid tissue. In these

Question 17
Cindy is 13 months old. She presents with two days of
fever, runny nose and cough. Her cough is becoming
increasingly distressing, and she is quite tachypnoeic.
On auscultation of her chest you hear some expiratory
wheeze and scattered crepitations. Cindy's breathing is
obviously laboured and there is rib retraction. You
diagnose bronchiolitis. Which of the following statements
is CORRECT?
a) Parenteral penicillin is the treatment of
choice
b) Chest Xray may show hyperinflation
c) Parainfluenza virus is the most common
pathogen
d) Corticosteroids are contraindicated
e) Inpatient treatment is often necessary

Answer: B
Chest x-ray findings in bronchiolitis typically include
hyperinflation with depression of the diaphragm and
horizontal ribs. There may also be streaky hilar
shadows, mild interstitial infiltrates and some segmental
atelectasis. RSV (respiratory syncitial virus) is by far the
most common pathogen, although some cases may be
due to the parainfluenza, influenza and adenoviruses.
Antibiotics are not indicated. Corticosteroids are not
contraindicated, but their use has not been shown to
modify the course of the disease. While some very
young or very unwell children may require supportive
inpatient management, most children with bronchiolitis
can be confidently managed as outpatients.

Question 18
Six weeks ago you excised a skin lesion from the cheek
of 65 year old Bill. The wound had healed well when you
removed the sutures five days later, but the pathology
report was not available. You told Bill you would contact
him if anything further needed to be done, as he was
leaving on an extended holiday. In doing some
paperwork today, you discover to your horror, Bill's
pathology report, which states: There is a squamous cell
carcinoma (SCC) measuring 5 mm in diameter, with a
depth of 4 mm. The lesion has been completely excised
but extends to within 1mm of one lateral margin, and to
within 2 mm of the base of the excision. What is your
MOST appropriate course of action?
a) contact Bill urgently, apologising for the
delay and explain that he should have further
excision as a clearance margin of 5mm in depth
and laterally is required to minimise local
recurrence or metastasis
b) contact Bill, apologising for your delay, and
advise it was a skin cancer (SCC) but it has
been completely excised, and there is minimal
risk of local recurrence or metastasis

c) do not attempt to contact Bill during his


holiday as the matter is not urgent. Clearance
of 1mm in all directions is adequate to
minimise local recurrence
d) do not disturb Bill as the holiday is important
to him, but ensure that you recall him when he
returns to discuss options for further surgery
e) contact Bill, saying you have just received
the report which the laboratory must have lost.
Advise him he had a skin cancer (SCC) and that
he should have further excision on his return,
as there is a very small risk this cancer could
spread

Answer: A
Squamous cell carcinomas (SCC) can recur locally, and
unlike basal cell carcinomas, they can also metastasise.
Local recurrence due to incomplete primary excision is
associated with a high rate of subsequent recurrence
(23%) and then metastasis to regional lymph nodes
(30%). Metastatic SCC has a 33% mortality rate.
Lesions less than 2cm in diameter require a 4mm
margin for adequate excision with depth through normal
underlying fat. Larger lesions may need up to 10mm
margins. Clearance for BCC excision is usually adequate
at 3mm margins. It is important to accept responsibility
for your failure to follow up as planned, and then to
recommend further excision. The re-excision should be
done quickly. It is not appropriate to blame the
pathology laboratory. It is wise to contact your medical
defence organisation.

Question 19
Jarrah is a 2 month old Aboriginal boy. He was born at
term via an uncomplicated vaginal delivery, and he has
been well. He lives with his family in Darwin. His mother
has brought him in for his immunisations. Which ONE of
the following statements is CORRECT?
a) Jarrah will need to receive the first of his
hepatitis A vaccines at 18 months of age
b) Limiting information to his mother about
risks will improve the chance of completing the
schedule

hepatitis A infection so from 1999 a programme for


HepA vaccination was commenced from 18mths of age.
Otherwise, the Standard Vaccination Schedule now
applies to all Australian children. All parents and
guardians must be given appropriate information
regarding vaccination so as to be able to give informed
consent to vaccination.

Question 20
Patricia, aged 25 years, has been diagnosed with
polycystic ovarian syndrome. In counselling this patient
regarding
long-term
consequences
the
MOST
APPROPRIATE advice would be:
a)
There
are
consequences

no

serious

b) There is an increased risk of endometrial


hyperplasia
c) There is an increased risk of cervical cancer
d) There is an increased risk of osteoporosis
e) There is an increased risk of premature
menopause

Answer: B
Polycystic ovarian syndrome is a metabolic condition
whose long term consequences include an increased risk
of cerebrovascular disease, non insulin dependent
diabetes and endometrial hyperplasia .The latter is due
to unopposed action of oestrogen in anovulatory cycles
.There is no effect on the timing of the onset of
menopause and no evidence for an effect on bone
density.

Question 21
Sally, aged 14 years, has mitral valve prolapse with
regurgitation. For which of the following procedures
should she be given antibiotic prophylaxis?
a) urethral catheterisation
b) flexible bronchoscopy

c) Jarrah should have had the BCG vaccine


soon after delivery before leaving hospital

c) dental examination

d) The oral polio vaccine (OPV) provides better


immunity than the inactivated type (IPV) in
Aboriginal children

e) local anaesthetic injection

e) Jarrah will need his first MMR vaccine at 6


months of age due to his higher susceptibility

Answer: C
Indigenous Australians are at increased risk of acquiring
tuberculosis. BCG is recommended for indigenous
neonates in 'regions of high incidence' of pulmonary TB,
particularly Darwin and the Northern Territory. It is
usually given to eligible infants soon after delivery.
Aboriginal children in north Australia have a suboptimal
response to OPV so there is likely to be improved
immunity to polio with the introduction of IPV as part of
the funded schedule. Although measles is very common
in indigenous communities, the first dose of MMR is
administered as usual at 12 months of age. Indigenous
children in Far North Queensland are at higher risk of

long-term

d) orthodontic bands placement

Answer: D
In patients with mitral valve prolapse with valvular
regurgitation antibiotic prohylaxis is required for dental
procedures which produce bleeding from the gingiva,
mucosa or bone. This includes the placement of
orthodontic bands. Prophylaxis is not required for
routine dental examination (except where scaling of the
teeth is to be performed), nor for the other options
listed.

Question 22
Rodney, aged 30 years, a diesel mechanic, presents
with a painful right eye. He was using a metal lathe at
work last night, and states that he was wearing his
safety goggles as required. He was aware that the eye

was a little irritable at the end of his shift, but it was not
until this morning that it became painful. On
examination, Rodney's visual acuity is 6/6 in his left eye
and 6/7.5 in the right. No obvious foreign body is seen,
but the pupil is slightly distorted. You do not have a slit
lamp. Of the following, which is the MOST appropriate
initial action?
a) Instill antibiotic drops, apply an eye pad and
check visual acuity in 24 hours
b) Irrigate the eye copiously with sterile saline
until the pH is neutral
c) Apply loose fitting eye shields to both eyes,
commence antibiotic and antiemetic
d) Order antero-posterior and lateral facial Xrays
e) Reassure Rodney that his safety goggles
have prevented serious eye injury

Answer: C
The history of eye pain after working with metal and
high-speed machinery should raise the suspicion of
penetrating eye injury. This may still occur despite
safety goggles being worn. Not all goggles conform to
standards, and not all workers wear them correctly!
Visual acuity is not always greatly reduced, but the pupil
commonly shows a 'tear-drop' deformity. There may be
hyphaema. Facial X ray and CT may be useful in locating
an intra-ocular pressure. Always check tetanus
immunisation status and give a booster if necessary.

Question 24
Which of the following statements about colonisation of
pregnant women by group B Strep is CORRECT?
a) most pregnant women are colonised
b) colonisation poses no risk to the woman or
foetus
c) colonisation is associated with premature
labour
d) colonised women are usually symptomatic
e) diagnosis requires serological testing

Answer: C
Group B Strep infection is the leading cause of neonatal
infection, and is implicated in premature labour and
other maternal morbidity. 10-30% of women may be
colonised with vaginal Group B Strep during pregnancy,
but the infection may be transient or intermittent. It
usually causes no symptoms. Transmission to the infant
almost always occurs after the onset of labour, or
membrane rupture. There is little evidence that
treatment earlier in pregnancy is beneficial. However,
treatment of women who, test positive at 35-37 weeks
gestation, with intrapartum antibiotics has been shown
to reduce the incidence of neonatal sepsis significantly.
Testing for Group B Strep colonisation of pregnant
women by vaginal swab is therefore offered by many
doctors in the third trimester.

Question 25
Question 23
Warren, aged 40 years, had a malignant melanoma
removed from his thigh last week. Which of the
following factors is MOST important in determining his
overall prognosis?
a) The depth of invasion of the skin and
subcutaneous tissue
b) The fact that he has multiple dysplastic
naevi

Felicia is a 15 month old girl. She has symptoms and


signs consistent with a viral upper respiratory tract
infection (URTI), including a fever of 38.8 degrees
Celsius. Her weight is 11kg. Felicia's mother has decided
to use paracetamol to help relieve her symptoms. Which
of the following is CORRECT regarding the dose of oral
paracetamol in this case? Daily dose should not exceed:
a) 90 mg/kg/day
b) 75 mg/kg/day

c) The diameter of the lesion removed

c) 60 mg/kg/day

d) A family history of malignant melanoma

d) 45 mg/kg/day

e) The site of the lesion- on his lower limb

e) 30 mg/kg/day

Answer: A

Answer: C

The single most important feature in determining the


ultimate prognosis in melanoma is the depth of invasion
of skin and subcutaneous tissue. Melanoma classification
systems are based on either the vertical thickness of the
lesion in millimetres as in Breslow's classification, or the
anatomic level of invasion of the layers of skin as in
Clark's classification. The diameter of the lesion is not as
important. Family history, multiple freckles and
melanocytic naevi are risk factors for melanoma, not
indicators of prognosis. The site of the lesion is of some
prognostic importance; those on the extremities have a
better outlook than those on the trunk or face.

The recommended oral or rectal paracetamol dose is


15mg/kg every 4 to 6 hours. In an unsupervised
community setting, dosage is limited to 60 mg/kg/day
for up to 48 hours. Up to 90 mg/kg/day can be used
under medical supervision with review after 48 hours. It
was believed that doses >150mg/kg/day were
necessary for hepatotoxicity but sick children <2 years
of age having > 90mg/kg/day for more than one day
are at higher risk

Question 26
All of the following
amenorrhoea EXCEPT:

Answer: E
are

causes

of

secondary

a) Diabetes

Studies indicate that maternal transmission of HIV to


the infant is most likely in the perinatal period. The
other options are correct. Advanced maternal disease is
also a risk factor for increased transmission risk.

b) Drug abuse
c) Eating disorders

Question 29

d) Asherman's syndrome

Which of the following statements concerning a retained


placenta is CORRECT?

e) Autoimmune disorders

a) antibiotic cover is rarely necessary after a


manual removal of placenta
Answer: A
Secondary causes of amenorrhoea and oligomenorrhoea
include eating disorders, drug abuse, Asherman's
syndrome (intrauterine adhesions), thyroid or adrenal
dysfunction, uterine or vaginal obstruction, familial early
menopause,
chromosomal
abnormalities
and
autoimmune disorders. Diabetes is not a cause.

Question 27
Muriel, aged 80 years, fell onto her outstretched right
hand sustaining a Colles' fracture. Which of the following
is the most frequent LATE complication of this fracture
for a patient of her age?
a) delayed union of the fracture
b) ischaemic necrosis of the distal fragments
c) neuralgic pain in the arm and hand
d) stiffness of the wrist and fingers
e) ulnar nerve weakness

Answer: D
Joint stiffness is common following a Colles' fracture,
especially after prolonged immobilisation. All of the
other complications are relatively uncommon. Union is
usually not delayed but there may be a degree of malunion. Appropriate management of the elderly patient
with a Colles' fracture includes early mobilisation aimed
at restoring function.

Question 28
Amanda, aged 24 years, is HIV positive. She has just
given birth to twin sons, George and Harry, by normal
vaginal delivery. Regarding vertical transmission of HIV
from mother to baby, which ONE of the following
statements is INCORRECT?
a) the first born twin is more likely to be
infected than is the second born twin
b) higher rates of transmission are likely with
low maternal CD4 lymphocyte count
c) breast feeding is a possible route of vertical
transmission
d) high maternal plasma HIV RNA increases
transmission risk
e) transmission to the foetus is most likely in
the antenatal period

b) the third stage of labour takes on average


two hours to complete
c) when delivering the placenta the doctor or
midwife should pull the cord upwards
d) there is no association between placenta
praevia and placenta accreta
e) oxytocics and continuous cord traction
enable delivery of most placentas within 10
minutes

Answer: E
The third stage of labour (when the placenta is
delivered) is normally completed within 30 minutes. If
not, it is unlikely to occur spontaneously. With the use
of oxytocics and gentle continuous downward cord
traction, 97% of third stages are complete within 10
minutes. Antibiotic cover is necessary for manual
removal of the placenta, because of passage of the hand
from a non-sterile (vagina) to a sterile (uterus)
environment. Placenta accreta (where placental villi
penetrate the uterine wall preventing separation) occurs
more commonly in cases of placenta praevia and after
caesarean section.

Question 30
Hazel, aged 40 years, has a month-long history of a
burning pain in the middle of her right foot. She says it's
"as though I'm standing on a sharp stone". The pain
radiates into the tips of her toes. Pressure applied to the
dorsum of the head of the third metatarsal reproduces
her pain, but there is no other abnormality noted. What
is the MOST likely diagnosis?
a) Interdigital (Morton's) neuroma
b) Plantar fasciitis
c) Gout
d) Peripheral neuropathy
e) Stress fracture of the third metatarsal

Answer: A
Interdigital neuroma represents a gradual and persistent
thickening of the perineurium of one, or less commonly
two or more interdigital nerves. Hazel's history is
typical, as is the examination finding of reproduction of
the pain by direct pressure over over the head of the
metatarsal. Plantar fasciitis causes primarily heel pain.

The absence of heat, redness or swelling over the


metatarsal heads makes gout and rheumatoid arthritis
unlikely. Peripheral neuropathy is usually associated
with nocturnal paraesthesia, dysaesthesia and burning
pain across the sole of the foot but is unlikely to be
affected by dorsal pressure on a metatarsal head. Stress
fractures of the metatarsals cause pain that is felt along
the shafts of the metatarsals, and is unlikely to radiate
to the tips of the toes.

Question 31

C. The problem is not an exacerbation of the symptoms


of a chronic condition (e.g., major depressive disorder).
D. The above criteria must be confirmed by prospective
daily ratings during at least three consecutive
symptomatic cycles to confirm a provisional diagnosis.

Question 32

With regard to symptoms of the severe form of the


premenstrual
syndrome
(premenstrual
dysphoric
disorder) which of the following is CORRECT? The
symptoms:
a) are confined to the late luteal phase of the
menstrual cycle
b) may eventually extend throughout the whole
menstrual cycle
c) disappear within hours of the onset of
menstruation
d) may fluctuate and disappear altogether in
some cycles
e) may present as
endogenous depression

B. Symptoms cause marked interference with work,


school, usual social activities, or relationships with
others.

an

exacerbation

of

Classic symptoms of endometriosis include all of the


following EXCEPT:
a) pelvic pain
b) dysmenorrhoea
c) dyspareunia
d) oligomenorrhoea
e) infertility

Answer: D
Endometriosis is the presence of endometrial tissue
outside the uterine cavity. It causes pelvic pain,
dysmenorrhoea,
dyspareunia
and
infertility.
Oligomenorrhoea is not associated with endometriosis.

Answer: A
While premenstrual syndrome is the term commonly
used to describe the constellation of symptoms prior to
periods, Premenstrual Dysphoric Disorder is a much
more specific condition listed by the American
Psychiatric Association in the Diagnostic and Statistical
Manual of Mental Disorders (DSM-IV). The diagnostic
criteria for PMDD are:
A. At least five of the following symptoms (one of which
must be 1, 2, 3 or 4, below) must be present in the
majority of menstrual cycles in the last year. Symptoms
should be isolated to the late luteal phase of the
menstrual cycle and remit within days of onset of
menses.
1. Markedly depressed mood, feelings
hopelessness, self-deprecating thoughts

of

2. Marked anxiety, tension, feelings of being


"keyed up" or "on edge"
3. Marked affective lability
4. Persistent and marked anger or irritability or
increased interpersonal conflicts
5. Decreased interest in usual activities
6. Subjective sense of difficulty concentrating
7. Lethargy, easy fatigability, marked lack of
energy
8. Marked change in appetite
9. Marked change in sleep pattern
10. Subjective sense of being overwhelmed or
out of control
11. Physical symptoms (e.g., breast tenderness
or swelling, headaches, joint or muscle pain,
sensation of bloating, weight gain)

Question 33
For a perimenopausal woman who has irregular cycles,
severe hot flushes and no contraindications to hormone
replacement therapy the MOST APPROPRIATE therapy
is:
a) continuous oestrogen therapy
b) continuous combined
progestogen) therapy

(oestrogen

and

c) sequential oestrogen therapy


d) sequential combined
progestogen) therapy

(oestrogen

and

e) continuous progestogen therapy


Answer: D
Hormone replacement therapy (HRT) is indicated in
women who are suffering from severe menopausal
symptoms, provided they have no contraindications to
its use. Sequential combined HRT is the best option for
perimenopausal
women
who
do
not
require
contraception and for women in the first 2 years after
menopause. It can alleviate symptoms and control
irregular cycles. The use of continuous or sequential
unopposed oestrogen is associated with endometrial
hyperplasia and the development of endometrial cancer
and is contraindicated in women who have not had a
hysterectomy.
Continuous
combined
HRT
is
recommended for symptomatic women more than 2
years post menopause; 50% will have irregular bleeding
for the first 6 months but 90% are amenorrhoeic after
12 months. Progestogen alone is not always effective at
treating menopausal symptoms. The issues surrounding
potential complications of HRT must always be fully
discussed before commencing therapy.

Question 34

Question 36

Sherri, aged 56 years, complains that she loses control


of her "waterworks" when she puts her key in the front
door. Given this history, which form of incontinence is
she MOST LIKELY to have?

Cheryl, aged 28 years, (G1P0), presents at 8 weeks


gestation complaining of constant nausea and vomiting.
Which of the following statements about nausea and
vomiting in pregnancy is CORRECT?

a) stress incontinence
b) urge incontinence
c) overflow incontinence
d) incontinence from a urinary fistula
e) incontinence secondary to a neuropathic
bladder

Answer: B
Urge incontinence occurs when there is an inability to
delay micturition and may be precipitated by various
triggers including the sound of running water or placing
a key in the door when arriving home. Stress
incontinence occurs when the intra-abdominal pressure
is raised as with coughing or sneezing. Overflow
incontinence is due to obstruction and may be
secondary to uterovaginal prolapse or a hypotonic
bladder as in a neuropathic bladder. A urinary fistula is
associated with continuous dribble of urine or leakage of
small amounts on effort.

Question 35
Teresa, aged 25 years, presents having experienced an
episode of postcoital bleeding two days ago. What is the
MOST APPROPRIATE management?
a) Reassure her and ask her to return if
bleeding recurs
b) Undertake cauterisation of the cervix to
prevent further bleeding
c) Treat her with metronidazole gel to eradicate
infection
d) Send her to the emergency department for
immediate assessment
e) Do a Pap smear and screen for sexually
transmitted infections

a) Less than 30% of women suffer nausea and


vomiting in pregnancy
b)
Metoclopramide
(antiemetic)
contraindicated in pregnancy

is

c) Women should try to have frequent small


feeds to control nausea
d) Nausea and vomiting should subside by 9
weeks gestation
e) Nausea and vomiting in the evening points
to a more sinister cause

Answer: C
At least two thirds of women experience nausea during
the first trimester, and 50% experience vomiting.
Symptoms can occur at any time of the day, although
classically they predominate in the morning. These
symptoms usually subside by 12-16 weeks gestation.
The best advice for women is to take small frequent
meals but, if the symptoms persist antiemetics such as
metoclopramide can be used safely.

Question 37
Which of the following elements on an antenatal
cardiotocograph
(CTG)
at
term
is
considered
"abnormal"?
a) Accelerations of 15 beats per minute lasting
15 seconds
b) One reactive movement in a 20 minute CTG
recording
c)
Decelerations
contractions

during

Braxton

Hicks

d) Variability in foetal heart rate beat-to-beat


e) Baseline heart rate of 120-160 beats per
minute

Answer: E

Answer: C

Postcoital bleeding is a serious symptom that could be


indicative of cervical pathology. It is not an emergency
requiring assessment in hospital. Common causes of
postcoital bleeding include a cervical erosion, an
infection such as chlamydia and other less common
pathologies, such as a cervical polyp. Medical
practitioners must however ensure that they exclude
precancerous or cancerous lesions of the cervix by
making sure that cervical cytology (Pap smear) is
performed as well as appropriate STI (sexually
transmitted infection) screening. If the bleeding is
recurrent, or the cervix looks abnormal, colposcopy is
recommended. Cauterisation of the cervix is sometimes
performed if a friable cervical erosion is present,
bleeding is recurrent and other cervical pathology has
been excluded.

Decelerations occurring after contractions are ominous,


particularly if they are prolonged. A healthy CTG shows
a baby that is moving and having acceleration of
heartbeat after movement. The beat to beat variability
is indicative of an intact central nervous system.

Question 38
Melinda has just started to menstruate (experience
periods). She is age 16 and has Down syndrome
(Trisomy 21). She wants to know all about her periods
and why she has to bleed. Which of the following
statements should you NOT tell Melinda regarding
menstruation (periods)?

a) Most females start having their periods


between the ages of 9 and 16
b) Other body changes may be happening when
periods start. Breasts get bigger and hair starts
to grow under arms and around the vagina
c) A period will occur about once a month and
will last for several days (about 3 to 7 days)
d) The blood that comes out with a period is
clean and healthy and it is normal
e) Tampons must be used during periods to
avoid bleeding onto underwear and clothes

malpresentations,
polyhydramnios,
during
breech
deliveries and with premature rupture of the membranes
It is an obstetric emergency, as the umbilical vessels
constrict,
once
exposed
to
the
extrauterine
environment. Unless the cervix is fully dilated and an
immediate operative vaginal delivery can be conducted,
an emergency caesarean section is required. During the
transfer to theatre the woman should be positioned so
that gravity can assist in keeping the presenting part off
the cord, i.e the knee - chest position (kneeling with
head down). The presenting part should also be pushed
digitally up and away from the cord which should be
placed wholly within the vagina.

Question 40

Answer: E
All the other statements are true and helpful when a
female with a potential learning disability starts learning
about her menstruation. Tampons can be particularly
convenient for certain activities, such as swimming, but
they are not essential and are not recommended for
women who cannot comfortably manage their own
menstrual self-care. If tampons are to be used it is
worth checking instructions are understood before
encouraging practice.

In counselling a woman regarding use of the mini pill


(progestogen only contraceptive pill, POP) which of the
following is CORRECT?
a) The menstrual cycle will be unaffected by
the POP
b) The main mode of action is to thicken
cervical mucous
c) Active pills are taken for 21 of 28 days to
allow a withdrawal bleed

Question 39

d) A delay of 12 hours in taking the POP does


not affect efficacy

Angelina, aged 27 years (G3 P2), has a transverse lie at


36 weeks gestation. Should her membranes rupture,
what would be the MOST APPROPRIATE MANAGEMENT?

e) Contraceptive efficacy of the POP


equivalent to combined pill in all women

a) Advise attendance at the Delivery Suite


when contractions are 5 minutes apart
b) Attempt an external cephalic version to allow
safer vaginal delivery
c) Wait for contractions to establish as this will
probably correct the lie
d) Augment the labour with
oxytocin to facilitate swift delivery

intravenous

e) Place in the knee-chest position and organise


emergency Caesarian section

Answer: E
Cord prolapse occurs when the umbilical cord lies beside
or in front of the presenting part. It is more common in

is

Answer: B
The principal mode of action of the progestogen only pill
is thickening of cervical mucus. In about one third of
women the minipill will also inhibit ovulation but in the
majority this is not the case. As a result of the variability
of the effect of the POP on ovulation and the effects of
progesterone on the endometrium, menstrual cycles
may be regular, irregular or spotting can occur
throughout the cycle in POP users. Women take the
minipill everyday without a break (28 active pills with no
inactive pills) in the same three-hour period each day in
order to maintain maximal efficacy. In general, it is less
efficacious than combined oral contraception because it
does not uniformly inhibit ovulation. However, in older
women who are less fertile and who use the POP
correctly, the efficacy of the POP can approximate that
of combined oral contraception.

Block 3
Question 1

Question 3

Tom, aged 18 years, presents with a typical first episode


of schizophrenia. Which of the following antipsychotic
medications is the treatment of choice for him?

Sarah, a 40 year old woman whose husband has a


plasma cholesterol of 6.9 mmol/l (normal <5.5mmol/l)
wants to know which oil she should use in meal
preparation for her spouse. Which of the following would
you advise?

a) chlorpromazine
b) haloperidol
c) thioridazine
d) olanzapine
e) clozapine

a) It makes little difference which cooking oil


she uses
b) She should use either canola or sunflower oil
c) Any margarine is suitable
d) Choose an oil rich in saturated fat over one
rich in unsaturated fats

Answer: D
Olanzapine, risperidone and quetiapine are three of the
new so-called atypical antipsychotic medications. These
are now preferred (over traditional antipsychotic drugs
like chlorpromazine, haloperidol and thioridazine) as first
line therapy in first episode schizophrenia, because of
improved efficacy and reduced incidence of psychomotor
retardation, a well-known and troubling side effect of
these medications. Clozapine is an atypical antipsychotic
drug recommended for schizophrenic patients who prove
resistant to other antipsychotic medications.

Question 2
Pete, aged 54 years, has been a heavy smoker for most
of his adult life. He complains about coughing up blood
first thing in the morning. The MOST IMPORTANT
condition to exclude is:
a) Bronchiectasis
b) Recurrent pulmonary emboli

e) None of the above

Answer: B
Epidemiological studies have shown reduced mortality
from cardiovascular causes associated with diets
containing
increased
levels
of
monoand
polyunsaturated fatty acids. Canola oil, like olive oil, has
a high concentration of monounsaturated fatty acids,
while sunflower oil is rich in n-6 polyunsaturated fatty
acids. For lowering of plasma cholesterol levels, the
National Heart Foundation of Australia recommends that
saturated fat in the diet be replaced with a combination
of mono- and
polyunsaturated fats.

Question 4
The diagnosis of acute gonorrhoea in a male is ideally
made by:

c) Leukaemia

a) Gonococcal complement fixation test

d) Bronchogenic carcinoma

b) VDRL reaction

e) Laryngeal cancer

c) Dark ground illumination of urethral pus


d) Gram stain and culture of urethral pus

Answer: D
Smokers are prone to develop a range of diseases
including:
atherosclerotic cardiovascular disease,
various cancers (lung, larynx, oral, oesophagus,
bladder, kidney, pancreas, stomach, cervix)
chronic obstructive pulmonary disease
peptic ulcer.
As bronchogenic carcinoma has a much more aggressive
course than the other options, a bronchoscopy should be
performed as soon as possible.

e) Prostatic massage

Answer: D
The gonococcal organism is rapidly identified by gram
stain and culture of urethral pus. Dark ground
illumination is a method for demonstrating the presence
of Treponema pallidum. VDRL is used for the diagnosis
of syphilis. Prostatic massage is an unreliable method of
obtaining a test sample for acute gonorrhoea.

Question 5

Answer: D

A COMMON side effect of the atypical antipsychotic drug,


olanzapine, is:

Only about 10% of human breast cancers are due to a


germline mutation (of genes p53, BRCA-1, BRCA-2). The
other 90% are due to somatic mutations, often of the
same genes as are involved in the familial varieties. A
family history of one 1 family member diagnosed with
breast cancer at 60 years of age (ie. over the age of 50
years) places June at average or only slightly increased
risk (1.5 times higher than population average). All the
other options place June at a moderate to high risk.

a) Neutropenia
b) Hypotension
c) Sexual dysfunction
d) Weight gain
e) Parkinsonism

Question 8
Answer: D
Weight gain of between 4kg and 9kg is a common side
effect of treatment with olanzapine. Neutropaenia is a
rare side effect. Sexual dysfunction can occur through
medication with major tranquillisers and selective
serotonin reuptake inhibitors (SSRI) antidepressants.
Parkinsonism tends to occur with long-term use of major
tranquillisers,
especially
phenothiazines
and
butyrophenones.

A 40 year old female undergoing treatment for


schizophrenia, is admitted repeatedly for not taking her
prescribed medication. She has delusional ideas,
claiming she communicates with angels and, as she
does not consider herself to be ill, believes that she
should not have to take any medication. The CORRECT
term for the latter phenomenon is:
a) Therapeutic delusions
b) Side effect of drug
c) Impaired insight

Question 6
A patient with a past history of rheumatic fever requires
oral amoxycillin cover for a tooth extraction. The
optimum time for commencing this is:
a) One week before the extraction

d) Transference
e) Hallucinations

Answer: C

b) Two days before the extraction

Impaired insight is one of the cardinal signs of psychotic


illness. It describes the situation where the patient lacks
a realistic awareness of self and the relationship of self
to others. Delusions are beliefs held, despite proof to the
contrary.
Hallucinations
are
abnormal
sensory
perceptions and are usually auditory in schizophrenia.
Transference is a psychoanalytical term referring to
transfer by a patient of subconscious or conscious
feelings onto the therapist.

c) The day before the extraction


d) One hour before the extraction
e) Immediately after the extraction

Answer: D
The risk of endocarditis in patients with valvular heart
disease stems from the bacteremia introduced by the
extraction. Antibiotics need only cover this period of
time and therefore are given orally one hour before the
extraction.

Question 9
The MOST COMMON cause of a blood-stained discharge
from the nipple of a 45 year old woman is:
a) Gynaecomastia

Question 7

b) Duct papilloma

June, aged 38 years, has a family history of breast


cancer and seeks advice about her risk of developing the
disease. All of the following are indicative of moderate to
high risk EXCEPT:

c) Paget's disease of the nipple

a) Two 2 individuals on the one side of the


family affected with breast cancer
b) One 1 family member with ovarian cancer
diagnosed before the age of 50 years
c) One 1 or 2 family member with bilateral
breast cancer
d) One 1 family member diagnosed with
breast cancer at 60 years of age
e) One 1 and one 2 family
diagnosed with ovarian cancer

member

d) Fibroadenoma
e) None of the above

Answer: B
A blood stained discharge from the nipple is commonly
caused by an intraductal
papilloma. Less common causes are an intraductal
carcinoma and mammary dysplasia. Gynaecomastia is
breast enlargement in the male and may be associated
with discharge, depending on the underlying cause.
Paget's disease of the nipple usually presents with a dry,
eczematous rash of the nipple. Fibroadenoma tends to
present with an asymptomatic discrete, mobile breast
lump.

Question 10

Answer: A

Which of the following drugs, causes stimulation of


cardiac contraction with LEAST vasoconstrictor effect?

A traumatic perforation of the tympanic membrane (ear


drum) that occurs in wet conditions eg. swimming or
waterskiing will generally become infected and be
associated with purulent discharge. Pseudomonas is
more likely to be the offending organism than
staphylococcus. A short course of topical antibiotics is
indicated, as is analgesia. Most traumatic perforations
heal spontaneously, but may take up to 9 months.
Surgical repair is indicated for the rare failure to heal.

a) Adrenaline
b) Isoprenaline
c) Pitressin
d) Ephedrine
e) Noradrenaline

Question 13
Answer: B
Isoprenaline works almost exclusively on beta receptors
causing increased rate and strength of cardiac
contractions (B1) and vasodilatation (B2). All the other
drugs listed cause significant vasoconstriction.

Question 11
While counselling a patient, a therapist becomes aware
that the patient is avoiding discussion of certain topics,
and is steering away from topics he finds uncomfortable.
Which of the following types of behaviour is the patient
exhibiting?
a) Resistance
b) Suppression
c) Regression
d) Repression
e) Projection

Answer: B
Suppression refers to the conscious or 'semi-conscious'
decision of an emotionally mature, healthy adult to
postpone dealing with conflict. Resistance refers to the
conscious and informed decision of a patient not to
change behaviour or comply with treatment. Regression
refers to return to an earlier stage of developmental
function. Repression refers to the mechanism by which
ideas, impulses or emotions which the person finds
painful or unacceptable are forced out of consciousness
and forgotten. Projection refers to the unconscious
attribution to others of one's own unacknowledged
feelings, thoughts or characteristics.

Which of the following statements about patent ductus


arteriosus is INCORRECT?
a) It occurs
phenomenon

frequently

as

an

isolated

b) Cyanosis is usually present


c) It causes a pansystolic 'machinery' murmur
at the upper left sternal edge
d) There is a wide pulse pressure
e) Treatment is by surgical closure

Answer: B
Cyanosis is not usually present unless a right to left
shunt develops. Patent ductus arteriosus is usually an
isolated problem occurring most commonly in females.
There are often no symptoms until later in life, when
heart failure or infectious endocarditis
develops. Clinical signs include a continuous murmur
and a bounding peripheral pulse with wide pulse
pressure due to shunting of blood from the aorta to the
pulmonary artery.

Question 14
Where both parents have schizophrenia, what is the
probability of their child developing schizophrenia?
a) More than 90%
b) About 70%
c) About 40%
d) About 10%
e) Less than 1%

Question 12
A traumatic perforation of the ear that has occurred in
wet conditions such as swimming or waterskiing will
often:
a) Be associated with a purulent discharge
b) Be complicated by a staphylococcal infection
c) Require a short course of oral antibiotics
d) Not heal spontaneously
e) Require surgical repair

Answer: C
Pooled data from a number of family studies show that
the risk of schizophrenia is about 40% for each child of
two schizophrenic parents. The incidence in the general
community is about 1%.

Question 15

Answer: E

Which of the following tympanic membrane perforations,


if left untreated, is NOT likely to progress to significant
complications?

Pre-contemplation is the first stage in a model of


behavioural change that helps the clinician assess the
likelihood that a patient will be receptive to an
intervention. At the pre-contemplation stage the patient
has not yet considered change as an option. The other
options all indicate that the patient is aware of the
process of change.

a) Continuously discharging central perforation


b) Large dry central perforation
c) Marginal perforation with discharge
d) Perforation associated with a cholesteatoma
e) Perforation surrounded by granulation tissue

Question 18
Epistaxis is UNLIKELY to arise from:
a) Injury to the turbinates

Answer: B
A dry central perforation will not progress to
complications, even if it does not heal. Surgical repair is
therefore elective and not mandatory. The other types
of perforation are not 'safe' and require specialist
attention. A continuously discharging central perforation
indicates granulation and a risk of osteitis and bone
destruction. Marginal perforation carries the same risk.
A cholesteatoma is not a neoplasm but a cystic lesion
containing amorphous debris (and sometimes spicules of
cholesterol). It is formed through chronic infection and
perforation of the eardrum with ingrowth of squamous
epithelium, forming a nest which becomes cystic. By
progressive enlargement a cholesteatoma can erode the
ossicles, labyrinth and adjacent bone and carries the risk
of cerebral abscess formation and meningitis.

Question 16
Which one of the following features is UNLIKELY to be
due to arterial ischaemia?
a) Pain along the buttock and thigh after
exertion
b) Weakness of the buttock and thigh
c) Shooting pain from buttock along the back of
the leg to calf
d) Weakness of the leg
e) Smooth shiny skin on the leg below the
knees

b) Spontaneous bleeding from Little's area


c) Anticoagulation therapy
d) Enlarged adenoids
e) Nasal fracture

Answer: D
Enlarged adenoids do not usually cause epistaxis. In
90% of cases, epistaxis arises from Little's area, the
lower anterior portion of the nasal septum, and
responds to first aid. A fracture commonly causes
epistaxis. Rarely, it can be associated with medical
conditions.

Question 19
Harold, aged 24 years, presents with fatigue, shortness
of breath on exercise and orthopnoea. On examination
there are signs of moderate left-sided heart failure. A
grade III pansystolic murmur is heard most prominently
at the apex and radiating into the left axilla. Which of
the following conditions is the MOST LIKELY diagnosis?
a) Mitral stenosis
b) Mitral regurgitation
c) Aortic stenosis
d) Aortic regurgitation
e) Tricuspid stenosis

Answer: C
Diffuse pain, weakness and paralysis are all signs of
arterial ischaemia. Characteristically the pain is a
cramp-like ache due to the release of pain-inducing
metabolites in muscle. Due to the aetiology, the pain is
diffuse and cannot be localised, as can the shooting pain
of nerve irritation.

Answer: B

Question 17

Mitral regurgitation presents as fatigue, exertional


dyspnoea and orthopnoea. It is associated with a
pansystolic murmur loudest at the apex but radiating
over the praecordium and into the axilla. It may also be
associated with a short mid-diastolic flow murmur
following a third heart sound, due to the rapid flow of
blood into the dilated left ventricle. The second heart
sound is normal.

When someone is referred to as being in the precontemplation stage with regard to a change in
behaviour, this means s/he is:

Question 20

a) Resistant to change
b) Preparing for change
c) Receptive to change
d) Looking forward to specific advice
e) Has not yet considered change

Stephen, aged 18 years, presents with spontaneous


epistaxis. What is the FIRST STEP in managing Stephen?
a) Direct pressure to the lower nose for two
minute interval
b) Position Stephen so that he is sitting and
leaning forward

c) Application of topical local anaesthetic

Question 23

d) Cautery of bleeding vessel

Sarah is a 28 year old diabetic patient who presents


with a recent history of fever and increased urinary
frequency. Urine culture shows E. coli sensitive to
ampicillin and gentamycin with which she is treated
intravenously. However, a week later she is still having
fever and the same urinary symptoms. Blood culture
reveals motile E.coli. All of the following are likely to be
causes for her symptoms EXCEPT:

e) Nasal packing with gauze

Answer: B
The initial steps are to position the patient sitting
forward to prevent blood dripping down the throat; and
to compress the cartilaginous portion of nose (Little's
area) for 5-10 minutes without interruption (constant
checking is likely to interfere with haemostasis and
restart bleeding). If this is ineffective, application of
local
anaesthetic
(traditionally
cocaine
for
its
vasoconstrictive properties) will facilitate packing of the
nose with ribbon gauze, Foley's catheter or similar
devices. Only in extreme cases would surgery be
needed.

a) In vivo resistance of the organism


b) Autonomic dysregulation due to her diabetes
c) Papillary necrosis
d) Inadequate dosage of antibiotic
e) Perinephric abscess

Answer: B
Question 21
What is the cause of the GREATER life expectancy at
birth of females than males in Australia'?
a) Males exercise more than females

Clinical failure of antimicrobial therapy may be due to


inadequate dose, accelerated drug inactivation, poor
penetration to a site of infection, undrained abscess,
poor host defences, dead tissue, superinfection by
another pathogen, or development of drug resistance.

b) Genetic and biological differences


c) Females seek health care facilities more than
males
d) Males die more in accidents and violence
than the females
e) Employment stress is more for males

Question 24
The aim of surgery in patients with perforation and
infection of the tympanic membrane is to:
a) Restore hearing
b) Produce a dry, safe, waterproof ear
c) Improve the appearance of the drum

Answer: B
Genetic and biological factors play a role together with
environmental factors in causing the greater life
expectancy of females at birth.

d) Prevent further perforation


e) Restore Eustachian tube function

Answer: B
Question 22

The aim of myringoplasty is to produce a dry, 'safe',


waterproof ear to which a hearing aid may be fitted.
'Safe' implies free of risk of cholesteatoma.
Myringoplasty may not restore hearing, as this also
depends on Eustachian tube function. The appearance of
the drum is obviously unimportant, nor will surgery
prevent recurrence of perforation.

Which of the following is CORRECT? Epistaxis is often:


a) A sign of underlying nasal disease
b) Influenced by environmental conditions
c) Attributable to posterior nasal causes
d) Associated
bleeding

with

congenital

causes

of

e) Copious unless promptly treated

Answer: B
Epistaxis is very common and is not usually a sign of
underlying disease. Environmental factors predisposing
to epistaxis include pollens causing allergic rhinitis, the
irritant effects of some nasal sprays, nose picking and
hot, dry, air dessicating the nasal mucosa and rendering
it more friable. In 90% of cases bleeding is from Little's
area, i.e. anterior nasal septum. While systemic causes,
such as bleeding diatheses and hypertension are
important causes of severe haemorrhage, they are rare,
as is life threatening bleeding.

Question 25
Victor, a 36 year old man, has known ischaemic heart
disease. He complains of a recent increase in frequency
of chest pain and presents with a prolonged episode of
chest pain. There are no ECG changes on your initial
assessment. Management includes all of the following
EXCEPT:
a) Admission to hospital
b) Plasma troponin measurement
c) Continuous ECG monitoring
d) Commencement of a statin drug
e) Begin thrombolytic therapy

Answer: E

a) An elderly debilitated patient

Clinically this patient has unstable angina pectoris


(UAP). Management should include continuous ECG
monitoring,-admission to hospital and plasma troponin
measurement to exclude myocardial infarction. The
Heart Foundation guidelines 2000-2002 state that
immediate commencement of a statin reduces risk in
UAP, as does aspirin and antithrombotic agents such as
heparin. Use of thrombolytics in UAP is not indicated
since they are ineffective and may be harmful. If there
is
no
improvement
in
24-48
hours,
cardiac
catheterisation and angioplasty are indicated.

b) The wound is more than 8 hours old

Answer: E

Question 26

Wounds treated appropriately and early do not need


antibiotics. Antibiotics may be needed if the following
risk factors are present:

Moira, a 21 year old woman, presents with an enlarged


lower cervical lymph node. Biopsy shows thyroid
glandular tissue. The MOST LIKELY diagnosis is:

c) The patient has alcoholic liver disease


d) A deep wound to the hand
e) A large superficial abrasion on the thigh of a
22 year old

delayed presentation
contamination

a) Ectopic thyroid

compromised patient, debilitated or general ill health

b) Adenoma of lateral thyroid

wounds in areas where infection may have serious


consequences, eg hands

c) Thyroglossal cyst

patients in whom presence of bacteremia may have


serious consequences, eg prosthetic heart valves or
orthopaedic appliances

d) Metastasis from thyroid carcinoma


e) Lymphadenoid goitre

Answer: D

Question 29

Thyroid cancer typically presents as a nodule in the


gland but can present as an enlarged cervical node, as
in this case. Surgical removal of involved nodes is
usually curative, especially in young patients. They are
sensitive to TSH and so thyroxine can be used to
suppress them. The cervical nodes lie more lateral in the
neck than is usual for ectopic thyroid tissue or an
adenoma. Similarly, thyroglossal cysts lie in the midline
between the thyroid gland and the base of the tongue,
and move when the tongue is protruded. Lymphadenoid
goitre is due to Hashimoto's thyroiditis.

John, a 58 year old overweight plumber, complains of


lack of energy towards the middle of the day. He has to
get up twice at night to pass urine and wants a check for
prostate cancer. The MOST LIKELY cause of his
symptoms is:

b) Diabetes insipidus
c) Diabetes mellitus
d) Hypercalcaemia
e) Chronic renal failure

Question 27
Which of the following usually
homonymous hemianopia?

a) Psychogenic polydipsia

results

in

left

a) Damage to the left optic nerve


b) A pituitary tumour
c) Damage to the right optic nerve
d) A lesion of the left optic radiation

Answer: C
The classical symptoms of type 1 diabetes mellitus are
polydipsia, polyuria, polyphagia, fatigue and loss of
weight. In type 2 diabetes mellitus, patients have insulin
resistance related to obesity, rather than loss of weight.
The other options listed also cause polyuria, but are less
likely in the circumstances described.

e) A lesion of the right optic tract


Question 30
Answer: E
Lesions of the optic tracts cause homonymous
hemianopic visual field defects. A lesion of the right
optic tract causes a left homonymous hemianopia, i.e.
left half of visual field lost in both eyes. Damage to the
optic nerve results in a unilateral blindness while
damage to the optic radiation results in a quadrantic
field defect. A pituitary tumour causes a bitemporal
hemianopia.

Question 28
Which one of the following situations will NOT require
prophylactic antibiotics to manage a wound:

Which of the following statements about post-traumatic


stress disorder, is correct?
a) It develops in every person who is affected
with the same severe stressful event
b) Signs and symptoms usually occur between
1 and 6 months after the stressful event
c) Alcohol abuse reduces the likelihood of its
development
d) It seldom occurs in assaulted spouses in
domestic violence
e) Debriefing and counselling are insufficient as
initial treatment

Answer: B

Answer: D

According to DSM-IV, the person must have experienced


an event outside the range of usual human experience
that would be markedly distressing to anyone. Signs and
symptoms usually occur between 1 and 6 months of the
stressful event. Not every person who suffers the same
stressful event will develop post-traumatic stress
disorder, and there are some known protective factors,
such as being part of a group involved in a traumatic
event. Risk factors include alcohol and drug abuse,
previous history of depression and previous history of
sexual abuse. Victims of domestic violence can develop
post-traumatic stress disorder. Short term counselling
and debriefing are effective treatments. Some sufferers
will also require drug therapy.

This ECG shows ventricular tachycardia with a rate of


150 b.p.m. There is a rapid ventricular rhythm with
broad, abnormal QRS complexes. Since his blood
pressure is well maintained, medical treatment is
indicated as first line approach. Lignocaine IV or sotalol
IV or amiodarone IV can be used. DC cardioversion is
required if medical therapy is unsuccessful. If the
cardiac output and blood pressure are very depressed,
emergency DC cardioversion must be considered.
Carotid sinus massage is not indicated in this setting
because there is a high likelihood of carotid artery
disease which makes the procedure dangerous. Also the
arrhythmia is unlikely to be a supraventricular
tachycardia with bundle branch block. If untreated the
ventricular tachycardia may rapidly progress to a
ventricular fibrillation.

Question 31
Malcolm, a 55 year old man, presented with worsening
symptoms of gastro-oesophageal reflux disorder (GORD)
despite following your lifestyle advice. You referred him
for a gastroscopy which has not revealed any
abnormality. He still complains of bloating and
heartburn. Which of the following is the MOST
APPROPRIATE advice?

Question 33
Which of the following is INCORRECT with regard to the
management of syphilis?
a) The diagnosis should be confirmed with
treponemal tests

a) Reflux has been excluded as a cause of his


symptoms

b) Intramuscular injection (IMI) of penicillin is


the treatment of choice

b) He should see a dietician to review possible


food allergies

c) Serological and clinical review should be


done at six and 12 months

c) Endoscopy detects the presence of reflux in


only 60-80% of patients

d) The patient should be tested for other STDs

d) He should begin a trial of a proton pump


inhibitor (PPI)
e) He should have a repeat endoscopy in 6
months

Answer: D
About 50% of patients with significant symptoms of
GORD have no abnormality on endoscopy. A good
response to a PPI is as good as 24 hour pH monitoring
to confirm the diagnosis. True food allergies are
uncommon (1-2% of adults) and typically cause skin
reactions, nausea, vomiting, diarrhoea or anaphylaxis.
Other reactions are non-immune and are called food
intolerance or idiosyncrasy.

e) Sexual contacts should be


examined for features of syphilis

clinically

Answer: E
Syphilis should be confirmed on diagnosis with specific
tests such as FTA-Abs. IMI benzathine penicillin as a
single dose followed by procaine penicillin IMI daily with
probenecid for 10 days is the first line treatment for
syphilis. Follow up with serological and clinical review is
recommended, as is the need to test for other STD's.
Recent sexual contacts may not have clinical features of
syphilis but should be treated.

Question 34

Question 32

Daryl is a 3 year old boy who presents with lesions at


the corner of his mouth and ulcers in the mouth (see
image below).

Herman is a 57 year old man who is recovering from a


hitherto uncomplicated myocardial infarction. On the
fourth day he complains of sudden onset of

These have developed rapidly over a few days and he is


febrile and not eating. Daryl's management should
include:

palpitations. Initial examination confirms a tachycardia


with blood pressure of 140/80. The ECG shows the
following rhythm (see figure).
The first line treatment for this patient is:
a) Carotid sinus massage
b) Digoxin IV

a) Commencement of oral metronidazole


b) Commencement of oral flucloxacillin
c) Commencement of famciclovir
d) Application of topical mupirocin
e) Application of topical corticosteroid

c) Verapamil IV
d) Lignocaine IV
e) DC cardioversion

Answer: C
This is an example of herpes simplex infection (primary
herpetic gingivostomatitis). Regional lymphadenopathy,
fever, headache and malaise may also be present.

Where there is difficulty eating or swallowing oral


famciclovir,
valaciclovir
or
aciclovir
should
be
commenced.
Systemic
analgesics
and
topical
anaesthetic agents, eg lignocaine gel can be used and
chlorhexidine mouthwashes may prevent secondary
infection. Topical corticosteroids are contraindicated.

Question 37
Edith is a 70 year old woman who presents with
palpitations. Her ECG is shown below.
What is the diagnosis?
a) Atrial flutter
b) Atrial fibrillation

Question 35
Maria brings Amy, her 6 month old daughter, who has
never been immunised, to see you. Maria states that
she is using homoeopathic drops. Which of the following
concepts is it important to ensure that Maria
understands?
a) Vaccine
prevalent

preventable

diseases

are

still

b) Side effects of the disease are greater than


side effects of the vaccine

c) Atrial premature beats


d) Sinus arrhythmia
e) 1st degree AV block

Answer: B
This ECG shows atrial fibrillation. There are no p waves
and the rhythm is irregularly irregular which causes the
patient to perceive palpitations.

c) The acellular form of the pertussis vaccine


reduces the incidence of side effects
d) Amy could still be fully immunised with
conventional vaccines
e) All the above

Question 38
The clinical features of classical migraine include all of
the following EXCEPT:
a) Unilateral temporofrontal distribution

Answer: E

b) Retro-orbital and occipital radiation

Maria needs to appreciate all the concepts listed in the


options. Moreover, she needs to be made aware thathomeopathic 'immunisation' has not been proven to give
protection against infectious diseases-only conventional
immunisation produces a measurable immune response.
The Australian Immunisation Handbook has a table
which clearly shows a comparison of the effects of
vaccines versus the much greater morbidity of the
diseases against which they protect.

c) Intense throbbing character


d) Duration 4 hours to a week
e) Associated with nausea and vomiting

Answer: D
Migraine attacks last 4-72 hours (average 6-8 hours),
but never as long as a week.

Question 36
Which of the following statements regarding dementia is
CORRECT?
a) One in nine Australians in the group aged
>85 suffers from dementia
b) Dementia affects one in four people aged
80-85

Question 39
Malcolm, aged 25 years, presents complaining of feeling
unwell with a painful ulcer on his penis. He has tender
inguinal lymphadenopathy on examination. What is the
MOST LIKELY diagnosis?
a) Primary genital herpes

c) Family history is a major risk factor for


Alzheimer disease

b) Primary syphilis

d) Vascular disease is the most common cause


of dementia

d) Recurrent genital herpes

e) Dementia is no more common in the


indigenous population than in the general
community

Answer: C
Age and family history are the two most common risk
factors for dementia. One in 15 Australians aged 65 and
over has dementia. In people aged 80-85 years, it
affects 1 in 9 people. In those over 85 years, it affects 1
in 4. Alzheimer's disease is the most common cause of
dementia. In the most recent assessment of indigenous
Australians, 10% of those aged 65 and over were found
to have dementia and another 10% were suspected of
having it.

c) Secondary syphilis

e) Chancroid

Answer: A
Primary genital herpes is the most likely cause of a
painful ulcerative lesion on his penis. It begins as
multiple vesicles which ulcerate and can become
secondarily infected. Recurrent genital herpes episodes
tend to become milder and less frequent over time. The
primary lesion of a syphilitic ulcer is painless and usually
persists for 4-6 weeks and heals spontaneously.
Chancroid
produces
multiple
painful
exudative
nonindurated ulcers.

Question 40

Answer: C

A 24 year old married woman presents with patches of a


scaly coppery pink macular rash over the trunk. The
rash has been present for one week. The patches are
oval, of different sizes, and appear to be spreading. The
patches are arranged along the skin creases. She feels
well. There are no other abnormal findings. Which of the
following is the MOST APPROPRIATE management?

The description is typical of Pityriasis rosea and


management is usually reassurance only as this is a
self-limiting condition and disappears in 4-10 weeks.
Calamine lotion can be used if there is an associated itch
and topical steroids are only rarely used in the presence
of moderately severe itch.

a) Application of benzyl benzoate lotion


b) Prescription of antihistamines
c) Reassurance as it is a self limiting condition
d) Pathology test for rubella antibody titre
e) Application of topical steroids

Block 4
Question 1
Which of the following statements about Dupuytren's
contracture is CORRECT?
a) It is a thickening of the tendon sheath
b) It usually causes flexion contracture of the
2nd and 3rd fingers of the hand
c) It is more common in women than men
d) The mode of inheritance
recessive

is autosomal

e) It is more common in diabetics

breathe, heavy in the chest, clammy and her heart


races. These symptoms also occur at other times quite
unexpectedly. These 'turns' last about 10 mins and then
she feels better, but Mary is really scared about when
they might occur next. What is the MOST LIKELY
diagnosis?
a) Mary has anxiety
b) Mary has a phobia
c) Mary has anxiety with panic attacks
d) Mary is depressed and has a phobia for
shopping centres
e) Mary has anxiety, and ongoing angina

Answer: E
Dupuytren's contracture is a thickening of the palmar
fascia resulting in flexion contracture of the fingers of
the hand particularly the ring and fifth fingers. Its
aetiology is unknown, but it is thought to be familial. It
is more common in men than women. It is more
common in alcoholics, diabetics and epileptics treated
with phenytoin.

Answer: C
Mary has the classical symptoms of panic attacks with
some pre-existing anxiety. With panic attacks symptoms
must peak within 10 min and usually dissipate within
minutes, leaving little to observe, except the person's
fear of another terrifying panic attack. A distinguishing
feature of panic disorder is that some of the panic
attacks are unexpected or spontaneous.

Question 2
Which of the following drugs is contraindicated in a child
with known glucose-6-phosphate dehydrogenase (G6PD)
deficiency?
a) Paracetamol
b) Salbutamol
c) Metronidazole
d) Sulphamethoxazole
e) Prednisolone

Question 4
Clarice, 26 years, presents to you concerned because
she has noticed that a dark mole on her thigh has
become enlarged, slightly lumpy and itchy over the last
two
months.
The
MOST
APPROPRIATE
initial
management would be to:
a) Ask Clarice to return for review in three
months
b) Take an incisional biopsy of the lesion for
histopathology
c) Treat the lesion using liquid nitrogen

Answer: D

d) Remove the lesion using laser

Glucose-6-phosphate dehydrogenase deficiency is a


genetically inherited enzyme deficiency which results in
acute haemolysis upon exposure to an environmental
stress (viral or bacterial infections), drugs or toxins.
Particular drugs may cause haemolysis in patients
deficient
in
G6PD
including
Sulphamethoxazole,
nitrofurantoin, primaquine, aspirin and others. Fava
beans may also precipitate haemolysis in a minority of
patients.

e) Undertake an elliptical excision clear of the


margin for histopathology

Question 3
It has been 18 months since Mary had her heart attack
and stroke. She is 81 years old, slightly anxious, but
very independent & mobile, even though she gets a little
short of breath going up stairs. Increasingly, Mary is
fearful of leaving the house, because sometimes as she
is due to leave, she feels dizzy, unsteady, unable to

Answer: E
If a malignant melanoma is suspected then an accurate
pathological report is required to guide further
management. For this reason it is important that the
initial
management involves complete removal of the lesion
without destruction of the tissue. Early detection and
removal of melanomas leads to better outcomes (Clark's
level one and two melanomas have a five year prognosis
of >90%). If a melanoma is diagnosed then referral to a
plastic surgeon is necessary for a wide local excision
involving a margin of 1-3 cm and to a depth of the deep
fascia.

Question 5
All of the following may be features of Down syndrome
(Trisomy 21) EXCEPT:

deviation. As the apparent right ventricular hypertrophy


disappears the ECG takes on a more adult appearance,
and should have an adult pattern by the age of ten
years.

a) Hypotonia
b) Webbing of the neck

Question 8

c) Congenital heart defects

Benny has always loved to go clubbing, and often after a


few drinks at the end of a night of dancing, he ends up
having casual sex with someone he meets at the
nightclub.

d) Abnormalities of the dermal ridge pattern


e) Epicanthic folds

Answer: B
Webbing of the neck is a feature seen in patients with
Turner syndrome. All other listed features may be
present in a patient with Down syndrome.

Benny had his first hepatitis B serology testing done last


week. These are his test results:
HBsAg = positive
HBsAb = negative
IgM HBcAb = positive
HBeAg = positive.

Question 6
The clinical features associated with raised intracranial
pressure include all of the following EXCEPT:
a) morning headache
b) vomiting
c) presence of papilloedema
d) decrease in conscious state
e) falling blood pressure with a falling pulse

What is the MOST LIKELY cause of these results?


a) Benny has been vaccinated in the past for
hepatitis B and is now immune
b) Benny has had hepatitis B infection
sometime in the past and it has resolved,
leaving him with life-long immunity
c) Benny is a hepatitis B carrier
d) Benny has acute or current hepatitis B
infection
e) Benny has early liver cirrhosis

Answer: E
Rising blood pressure (not falling) in combination with a
falling pulse rate is a classical feature of rising
intracranial pressure known as the Cushing response.
Headache occurs as a result of the deformation of
intracranial blood vessels and dural membranes which
arises from conditions which give rise to raised
intracranial pressure. The headache is worst in the
morning (as is vomiting) and is aggravated by coughing,
sneezing or stooping. When present papilloedema
(swelling of the nerve fibres of the optic disc) is highly
suggestive of raised intracranial pressure. A decrease in
conscious state commencing with confusion and
progressing through various grades of coma is also seen
with increasing intracranial pressure.

Answer: D
Benny is HBsAg positive which occurs 1-6 months after
exposure to the hepatitis B virus and indicates acute
infection. If HBsAg persists after 6 months, it defines
carrirer status. HBsAb is not present (it would be
positive following vaccination). IgM HBcAb is present in
acute infection only (IgG HBcAb is present in highly
infective carriers and in acute infection). HBeAg is
present and implies high infectivity in recent infection
and carriers. Benny needs education about hepatitis B,
safe sex & drug use.

Question 9
Question 7
In the first year of life which of the following ECG
features may be considered normal?
a) Right axis deviation
b) Sinus bradycardia
c) First degree heart block
d) Left bundle branch block
e) ST segment depression

Answer: A
At birth the right ventricular muscle is as thick as the
left. This results in an ECG pattern which would indicate
right ventricular hypertrophy, including right axis

Which of the following is FALSE regarding neural tube


defects and folate before and during pregnancy?
a) Folate intake should be increased at least
one month before and three months after
conception
b) Most women before and during pregnancy
need 0.5mg folate daily
c) Women on anti-epileptic medication may
require 5mg folate daily before and during
pregnancy
d) Folate reduces the incidence of neural tube
defects which occur at the rate or 1:5000
pregnancies
e) Women with a family history of neural tube
defects need more folate before and during
pregnancy

Answer: D

Question 12

Pregnant women are at increased risk of folate


deficiency due to the high demand of the developing
foetus. Deficiency in the first few weeks of pregnancy
can cause neural tube defects in the newborns. Neural
tube defects occur at a rate of 1:500 pregnancies. The
other options are true.

Kylie is pregnant. She has smoked 25 cigarettes per day


for the past 10 years and continues to smoke even now
she is pregnant. Early in her pregnancy you outline to
her, that, compared with infants born to non-smoking
mothers, her infant is more likely to experience a
number of disadvantages. These include all of the
following EXCEPT:
a) Higher perinatal mortality

Question 10
Pamela aged 45 years, attends having found a lump in
the upper outer quadrant of her right breast two days
ago. She is concerned about the likelihood of cancer. In
order to diagnose the nature of the lump you invoke the
use of the "triple test" or "triple assessment". The triple
test consists of:
a)
Clinical
examination,
magnetic resonance imaging
b) Mammography,
biopsy

mammography,

ultrasound,

fine

needle

c) Clinical examination, mammography, fine


needle biopsy
d) Clinical examination, ultrasound, magnetic
resonance imaging
e) Ultrasound, fine needle biopsy, magnetic
resonance imaging

Answer: C
Management of breast lumps is now based on the triple
test, which combines the results of clinical examination,
mammography (+/- ultrasound) and fine needle
aspiration biopsy. When combined, these tests give a
sensitivity of 95-99% in the diagnosis of breast lumps.

b) Small for gestational age


c) Greater likelihood of sudden infant death
syndrome
d) Developmental lag at least in early years
e) Greater likelihood of developing small teeth
with faulty enamel

Answer: E
Small teeth with faulty enamel is a disorder resulting
from excess alcohol intake in pregnancy and the foetal
alcohol syndrome. The other options are all
disadvantages experienced by infants of women who
smoke.

Question 13
Alison, aged 18 years presents with a mobile, smooth,
solid lump of 2 cm diameter in her left breast. The MOST
LIKELY diagnosis is:
a) malignancy
b) fibroadenoma
c) breast cyst
d) intraductal carcinoma

Question 11
On examining Fatima (aged 18 months), whom you are
seeing for the first time, you hear a heart murmur.
Which of the following clinical findings would suggest
that this is an innocent heart murmur?
a) The murmur is diastolic
b) The murmur is associated with a thrill
c) The murmur is pansystolic
d) The murmur disappears when the child lies
down
e) The murmur is associated with reduced
exercise tolerance

e) breast abscess

Answer: B
A fibroadenoma is a benign breast condition that arises
as an aberration of normal development and involution.
The diagnosis is best made using the triple test.
Indications for removal include patient preference or
discomfort, size > 3 cm, continued growth or lump
presenting for the first time > 40 years. Fibroadenomas
may spontaneously disappear or calcify.

Question 14
Which of the following statements concerning nappy
rash is CORRECT?

Answer: D
The disappearance of the murmur when the child lies
down suggests it is the innocent murmur known as
'venous hum'. This is a murmur produced by blood flow
through the great veins and is heard at the base of the
heart, often just below the clavicles. It is blowing and
continuous in nature. The murmur varies with
respiration and the position of the head, and disappears
when the child lies down. The other features listed are
not those of an innocent murmur.

a) Seborrhoeic dermatitis is the most common


cause
b) Management should include liberal use of
talcum powder to help keep the nappy area dry
c) The napkin area should be washed
frequently with soap to avoid recurrences
d) Superinfection with Candida tends to involve
the flexures
e) Topical
treatment

steroids

are

the

mainstay

of

Answer: D

a) The usual age of symptom onset is 2-6 years

Irritant dermatitis is the most common cause of nappy


rash and tends to spare the flexures. Candidiasis will
involve the flexures and may extend beyond the napkin
area as 'satellite lesions'. In managing nappy rash, the
area should be kept dry, but powders should be avoided
as should soaps and excessive bathing or scrubbing.
Topical corticosteroids should be used with caution to
treat specific causes of nappy rash only, including atopic
dermatitis and seborrhoeic dermatitis.

b) Genetic
recessive

inheritance

is

usually

X-linked

c) Approximately 25% of patients die by the


age of 20 years
d) A lordotic, waddling gait is a feature
e) Female carriers are usually asymptomatic

Answer: C

Question 15
The classical signs of congenital rubella
measles) include all of the following EXCEPT:

(German

75% of patients with Duchenne muscular dystrophy die


by age 20, usually from cardiac or respiratory failure.
The other options given are correct for this condition.

a) Cataract
b) Heart disease

Question 18

c) Deafness

Mel always loved playing girls games as a child and had


fantasies about being female. When puberty arrived he
was distressed at the physical changes that occurred in
his body, and as soon as he left home he adopted a
complete female appearance and female role in public
and private. He obtained a driver's license and was able
to work and live in society as a woman. Mel takes
ethinyl estradiol 0.10 mg/day and has nearly completed
2-years of living completely as a woman. Mel has
requested sex reassignment surgery. What is Mel's
diagnosis?

d) Low birth weight


e) Koplik's spots

Answer: E
Koplik's spots are typically associated with measles
(rubeola) only and not any other infectious diseases.
The other options are features of congenital rubella.

a) Transvestite
b) Cross-dressing homosexual

Question 16
Which of the following statements regarding carcinoma
of the lung is CORRECT?
a) Lung cancer is the most common registrable
cancer in women
b) Approximately 10% of lung cancers are
derived from squamous cells
c) Approximately 60% of lung cancers are
adenocarcinomas
d) The majority of lung
asymptomatic at diagnosis

cancers

are

e) Exposure to asbestos increases risk of lung


cancer

Answer: E
Smoking and exposure to asbestos are associated with
the development of squamous cell and adenocarcinoma
of the lung. Prostate and breast cancer are the most
common registrable cancers in men and women
respectively. Lung cancer is the most common
malignancy causing death in men and second most
common in women after breast cancer although
incidence in women is rising. Of lung cancers, 32% are
adenocarcinoma, 29% are squamous cell, 9%
undifferentiated large cell and 18% small cell type. Up
to 15% of people are asymptomatic of their lung cancer
at diagnosis.

Question 17
Which of the following statements regarding Duchenne
muscular dystrophy is INCORRECT?

c) Schizophrenia with gender issues


d) Borderline personality disorder
e) Male transsexual

Answer: E
Male transsexualism is a gender identity disorder in
which the male believes he is the victim of a biologic
accident, cruelly imprisoned in a body incompatible with
his subjective gender identity. Transvestism occurs
when heterosexual males dress in women's clothing,
and at least initially this is associated with sexual
arousal. Transvestism is a psychiatric disorder only if the
fantasies, urges, or cross-dressing behaviours are
associated with clinically significant distress or
recognizable dysfunction. Cross-dressing per se is not a
disorder. Homosexuality is not a psychosexual disorder
but a preference of a sexual partner. Schizophrenia is
not a gender disorder.

Question 19
Regarding Sudden Infant Death Syndrome (SIDS),
which of the following statements is CORRECT?
a) Positioning a baby prone (on its front) to
sleep may reduce the risk
b) The incidence is greater in female infants
c) Maternal smoking has not been shown to be
a risk factor
d) Breastfed infants are at greater risk
e) Infant overheating may be a risk factor

Answer: E
Regarding modifiable risk factors for SIDS, positioning
the infant supine (on its back) to sleep, breastfeeding,
avoidance of overheating, and maternal smoking
cessation may reduce risk. Male infants are more at risk
from SIDS (the male: female ratio is approximately
3:2).

trauma or spontaneously. The physical signs of a


pneumothorax are a hyper-resonant percussion note
and absent breath sounds. In cases of tension
pneumothorax, there is mediastinal displacement away
from the side of the defect. Treatment is necessary
when there is a large enough pneumothorax to inhibit
respiratory activity and involves the insertion of an
intercostal drain in the fifth intercostal space in the
midaxillary line or alternatively in the second intercostal
space anteriorly in the midclavicular line.

Question 20
With regards to cryptorchidism (undescended testes),
which of the following statements is CORRECT?
a) It is essential that the testes are returned to
their normal position in the scrotum by the
time the boy is five years old
b) Bilateral undescended testes is a more
common
occurrence
than
unilateral
undescended testis
c) The lower the arrest in the line of descent of
the testis the more hypoplastic it is
d) Malignancy in the undescended testis is 2030 times more common than usual
e)
Inguinal
hernia
is
associated
with
undescended testes in approximately 50% of
cases

Question 22
Beth, aged 6 months, is brought to see you by her
mother who has noticed her eyes are not always lined
up. You are concerned Beth may have a squint
(strabismus). Which of the following statements
regarding strabismus is CORRECT?
a) Investigation is unnecessary in this age
group as strabismus improves with time
b) By the age of 6 months Beth's eyes should
be constantly well aligned
c) Strabismus is rarely a marker of other ocular
disease
d) Strabismus is not associated with amblyopia
e) The corneal light reflex is a reliable test to
diagnose strabismus

Answer: D
Complications of undescended testes include defective
spermatogenesis, torsion, trauma, and malignant
degeneration 20-30 times more common even after
surgical placement in the scrotum. Ninety five percent
are associated with a patent processus vaginalis but
only 25% develop a clinical hernia. Twenty five percent
have bilateral undescended testes. The testis/es may be
intrabdominal, inguinal or high in the scrotum. The
higher the arrest along the line of descent the more
hypoplastic the testis. In order to minimise these
complications the testis/es should be placed in their
normal position before the second year of life.

Question 21
Regarding pneumothorax,
statements is CORRECT?

Answer: B
A baby's eyes should be constantly well aligned by the
age of 5 to 6 months. Intermittent ocular deviation
should be investigated if present at six months, as it
may be a marker of severe underlying ocular or
neurologic disease. It should never be assumed that the
strabismus will be outgrown. The corneal light reflex test
should not be relied upon to diagnose or exclude
strabismus. The cover test is a more accurate diagnostic
test. Strabismus may lead to amblyopia, which in turn
may result in permanent loss of vision if it is not
corrected by 4 to 6 years of age.

Question 23
which

of

the

following

a) It is due to the presence of air outside the


parietal pleura
b) It may occur spontaneously with or without
underlying lung disease
c) Clinical examination reveals a dull percussion
note and absent breath sounds
d) There may be a mediastinal shift towards
the side of the pneumothorax
e) Treatment consists of an intercostal drain in
the second intercostal space in the mid axillary
line

Answer: B
Pneumothorax is the presence of air between the
visceral and parietal pleura. It can occur as result of

Sam and Mary have two daughters. Their second


daughter has Cystic Fibrosis, but their elder daughter
does not. They are considering having another baby.
The likelihood of Sam and Mary having another child
with Cystic Fibrosis is?
a) 1 in 2
b) 1 in 4
c) 1 in 10
d) 1 in 16
e) 1 in 25

Answer: B
Cystic Fibrosis is an autosomal recessive disorder. If a
husband and wife are both carriers of the autosomal
recessive gene then each pregnancy has a 25% chance
of resulting in a child who will be homozygous for and
thus affected by the disease.

Question 24
Sean, aged 65 years, presents with a history of painless
haematuria over the last week. Possible causes include
all of the following EXCEPT:
a) Cancer within the kidney
b) Use of anticoagulants c) Glomerulonephritis
d) Benign prostatic hypertrophy
e) Use of cyclophosphamide

Answer: D
Benign prostatic hypertrophy is associated with difficulty
micturating but not haematuria. Common causes of
painless haematuria include malignancy of the renal
pelvis, drugs such as anticoagulants, cyclophosphamide
and D- penicillamine, and glomerulonephritis. Rarely it
can result from a bleeding tendency due to inherited
disorders,
bleeding
secondary
to
idiopathic
thrombocytopenic purpura or Henoch Schonlein disease,
malaria, "jogger's haematuria", schistosomiasis.

Question 25
Robyn, aged 43 years, is known to have gallstones. On
this occasion she presents with the acute onset of
severe pain which was at first central in location but has
now moved to the right costal margin and radiates to
the back. She is pyrexic, slightly tachycardic and has
tenderness over the area of the gall bladder but no
rigidity of the abdomen. The MOST APPROPRIATE
MANAGEMENT would be to:
a) Observe her at home for 2 to 3 days to allow
this attack to settle
b) Admit her to hospital for treatment with IV
fluids and antibiotics
c) Avoid opioid analgesia due to the risk of
worsening biliary spasm
d) Admit her to hospital for urgent surgery
e) Treat this episode with the goal of preparing
for surgery in 2 to 3 months

Answer: B
Robyn has acute cholecystitis. Initial management
includes IV fluids and nil by mouth, pain relief with
parenteral opiate administration and a short, intensive
course of antibiotics. Although opiates may increase
biliary spasm this is not a contra-indication in view of
their excellent analgesic effect. The patient is monitored
and immediate operation is ONLY indicated if the fever
does not settle or symptoms worsen, indicating
perforation of the gall bladder or peritonitis. Immediate
operation is not warranted, as there is no indication of
perforation of the gall bladder or peritonitis. However,
early operation for acute cholecystitis is now
recommended compared to delaying surgery.

Question 26
Kari is 7 months old and has not received any
immunisations. She presents with two weeks of
paroxysmal coughing and vomiting, but is relatively
happy between paroxysms. You suspect she may have
whooping cough (pertussis). Kari lives at home with her

mother, father and older brothers, aged 2 and 4 years.


Neither of her brothers have been immunised against
pertussis. Choose the BEST INITIAL MANAGEMENT
option from the list below.
a) Arrange to have Kari admitted to hospital
and isolated immediately
b) Report the family to the child protection
agency in your state for failing to immunise
their children
c) Vaccinate Kari immediately with DTPa-hepB
or DTPa
d) Prescribe oral erythromycin for Kari and the
whole family
e) Take a nasopharangeal aspirate for
diagnosis, and await confirmation of diagnosis
prior to starting any other treatment measures

Answer: D
Whilst it is important to obtain a laboratory diagnosis of
pertussis, this should not delay treatment, which should
be commenced after appropriate nasopharangeal
aspirate or serological samples are collected. Kari should
be treated with erythromycin 10mg/kg/dose up to
250mg orally 6 hourly for 10 days, as should all
household and other close contacts. This will not shorten
the course of the illness in Kari but will reduce infectivity
and eliminate carriage of the Bordatella pertussis
organism in family members. Hospitalisation and
isolation are unnecessary unless the clinical condition of
the patient warrants inpatient management or in infants
less than 6 months of age. Catch-up vaccination should
be addressed, but is not the most immediate concern
here. There is no requirement to report the family to
authorities if they are conscientious objectors to
immunisation.

Question 27
Which
of
the
following
statements
regarding
nephroblastoma (Wilms tumour) is CORRECT?
a) It usually presents as an asymptomatic
abdominal mass
b) It is frequently associated with congenital
abnormalities
c) Most patients
diagnosis

will

be

hypertensive

at

d) Prognosis is worse for stage 1 than for stage


4 tumours
e)
The
treatment
of
choice
chemotherapy prior to surgery

involves

Answer: A
Nephroblastoma is a tumour arising from the kidney and
is a common tumour in children. Most nephroblastomas
present as an asymptomatic abdominal mass. Fever and
haematuria and hypertension are present in 20 to 25%
of patients. The majority of Wilms tumours occur
sporadically, although rarely there may be associated
malformations and syndromes. Prognosis is worse for
stage 4 than for stage 1 tumours. Wilms tumours are
first resected, then chemotherapy or radiotherapy may
be administered depending on tumour histology and
stage.

Question 28
Bobby presents with a
pathological in nature.
osteosarcoma. Which
regarding osteosarcoma

fracture which is found to be


Further investigation confirms
of the following statements
is CORRECT?

sequelae of this common condition. Bathing the lesions


to remove the crusts may be helpful.

Question 29

a) The peak occurrence of osteosarcoma is in


early childhood

Which of the following statements is CORRECT? Type 1


Diabetes Mellitus:

b) The tumour usually arises in the midshaft of


the tibia

a) Is less common than Type 2 diabetes in


adolescents

c) The X-ray appearance of osteosarcoma is


characteristic

b) Is due to immunological
pancreatic Alpha cells

d) Osteosarcoma accounts for 10% of primary


malignant bone tumours of childhood

c) Occurs in 6% of siblings of an affected


person

e) Osteosarcoma rarely metastasizes to lung

d) Presents with polyuria, polydipsia and weight


gain
e) Can be managed initially
hypoglycaemic agents and exercise

Answer: C
The x-ray appearance of osteosarcoma is quite
characteristic with destruction of the normal bony
trabecular pattern and periostial new bone formation
with lifting of the bony cortex to create a Codman
triangle. However, a tissue sample is required for
diagnosis. Osteosarcoma accounts for 60% of primary
malignant bone tumours in childhood, and occurs mostly
in adolescents and young adults. More than 40% of
tumours arise in the distal femur. At diagnosis a chest
CT is essential to look for lung metastases which may be
present in 20% of cases, worsening prognosis.

damage

with

to

oral

Answer: C
Type 1 diabetes mellitus is the most common type of
diabetes in people under 40 years of age, including
adolescents. Type 1A diabetes mellitus, or immunemediated diabetes, results from immunologic damage to
the insulin-producing Beta cells of the pancreatic islets.
About 6% of siblings of an affected person also develop
Type 1 diabetes. The classic presentation is with
symptoms of polyuria, polydipsia and weight loss.
Insulin is the required treatment for Type 1 diabetes.

Question 29
Esther is 7 years old. She presents with a large yellow
crusted lesion on her left cheek and similar yellow
crusted lesions along her left lower jawline. She has no
lesions or rash elsewhere and is otherwise well. Which is
the MOST ACCURATE statement regarding this
condition?
a) Herpes
organism

simplex

is

the

likely

causative

b) It is important not to disturb the crusts


c) Esther should be screened for immune
deficiency
d) Topical
treatment

mupirocin

is

an

Question 31
This pure tone audiogram is recorded from a 12 year old
Maori girl complaining of deafness in her right ear. The
MOST likely explanation for this problem is:
a) Debris in the external auditory meatus
b) Cholesteatoma
c) Middle ear effusion
d) Toxin-induced nerve damage
e) Necrosis of the ossicular chain

appropriate

e) Oral antibiotics should be commenced as


early as possible to prevent septicaemia
developing

Answer: D
The most likely diagnosis is impetigo, with the ruptured
vesicles that form yellow crusts and weeping erosions
being quite typical of the lesions. Herpes simplex has a
different clinical presentation. In childhood, primary HSV
infection
usually
presents
as
severe
acute
gingivostomatitis. Impetigo is a very common, highly
contagious infection, and does not suggest an
underlying immune deficiency. The usual pathogen is
Staphylococcus aureus, or Streptococcus pyogenes. For
mild or localised impetigo, topical mupirocin 2%
ointment or cream 3 times daily for 7 days is
appropriate treatment. The lesions must be covered.
Whilst oral antibiotics may be indicated for more
widespread infection, septicaemia is not a usual

Answer: D
The pure tone audiogram provides measurement of the
threshold of hearing at a variety of frequencies, by air
and bone conduction. The pattern of hearing loss shown
on this audiogram is of significant hearing deficit in the
higher frequencies, in the right ear. Air and bone
conduction are equally affected. This is the pattern of
sensorineural deafness. The left ear shows a normal
pattern.
Toxin-induced nerve damage is the only option which
would produce sensorineural hearing loss. All of the
others would give rise to a conductive deafness, where
the loss would be in the lower frequencies during air
conduction. The bone conduction curve would be
normal.

Question 32

Answer: C

One minute after birth an infant shows deep cyanosis of


the trunk and limbs, makes no reaction to a catheter
inserted into the nose, is limp but takes an occasional
gasp. What is the Apgar score?

Heberden's nodes are due to primary osteoarthritis.


They are firm swellings composed of bone and cartilage
on the dorsomedial and dorsolateral aspects of the distal
interphalangeal joints in the hand joints. Rheumatoid
arthritis
is
characterised
by
symmetrical
joint
involvement (usually proximal interphalangeal joints and
metacarpophalangeal joints), morning stiffness greater
than an hour, synovial inflammation. Rheumatoid
nodules can occur on extensor surfaces of joints in 2030% of people with rheumatoid arthritis.

a) 0
b) 1
c) 2
d) 3
e) Insufficient data

Question 35

Answer: E
The table below shows the data required to determine
an Apgar score. The scenario given lacks information
about the heart rate. Other data given are compatible
with a score of 0.

Herman, a 58 year old businessman, finds it difficult to


travel by air or train. When he has no option but to
travel, he finds the journey very difficult and gets out of
the aircraft or train as soon as possible. Because of this
he mostly avoids travel and this liability is interfering
with his work. What is the MOST LIKELY diagnosis?
a) Social phobia
b) Agoraphobia
c) Depression
d) Generalised anxiety disorder
e) Panic disorder

Answer: B
Agoraphobia is anxiety about being placed in crowded
situations from which escape might be difficult or
embarrassing, e.g. on aircraft or trains. Generalised
anxiety disorder, panic disorder and social phobia are
other varieties of anxiety disorders. Anxiety is frequently
a symptom of clinical depression.

Question 33
In Huntington's disease the mode of inheritance is:
a) X (or sex) linked
b) Recessive

Question 36

c) Dominant

In which of the following conditions does acute arthritis


commonly occur?

d) Isolated genetic mutation

a) Rubella

e) None of the above

b) Influenza
c) Measles

Answer: C
Huntington's disease is an autosomal dominant condition
with full penetrance. Therefore, the child of an affected
parent has a 50% chance of developing the disease.
Onset is usually in middle age.

d) Infectious mononucleosis
e) Varicella

Answer: A
Question 34
Each of the following is
rheumatoid arthritis EXCEPT:

characteristic

of

adult

a) Morning stiffness of joints


b) Soft tissue swellings
c) Herberden's nodes
d) Subcutaneous nodules at pressure points
e) Symmetrical joint involvement

Acute polyarthritis may occur in rubella, especially in


young women. The pain and swelling involve wrists,
fingers and knees. It is most marked during the period
of the rash, but can persist for up to 14 days after other
manifestations have disappeared. Recurrent joint
symptoms up to a year have been recorded. Acute
polyarthritis is not one of the usual manifestations or
complications of the other diseases listed.

Question 37

Question 39

All of the following characteristics describe the 'ideal'


vaccine, EXCEPT?

Sanjay, a 45-year-old man, presents with a one-year


history of progressive ankle swelling, difficulty with
speaking due to an enlarged tongue, pain and
paraesthesiae in the forearm and hand, particularly at
night, and easy bruising. What is the MOST LIKELY
diagnosis?

a) It is heat stable
b) It provides lifelong immunity with a single
dose
c) It has minimal adverse reactions
d) It has a good antibody response in the
presence of other antigens
e) It is administered orally

a) Membranous nephritis
b) Allergic reaction
c) Chronic leukaemia
d) Amyloidosis
e) Hypothyroidism

Answer: E
All the characteristics listed in the options are desirable
in an 'ideal' vaccine, except (e). Each vaccine has an
appropriate route of administration which determines its
efficacy and probability of side effects. For compliance
and ease of administration, delivery without a
hypodermic syringe would be ideal but most vaccines
are ineffective via the oral route. Other desirable
features of the 'ideal' include: being able to combine
readily with other antigens, ease of administration and
low cost. Although characteristics of the ideal vaccine
are well established, developing and producing them is
often difficult.

Question 38
Lulu is a three year old child who has swallowed
kerosene and is brought immediately to the hospital
casualty department. Which of the following measures
should be undertaken in the immediate management of
Lulu's problem?

Answer: D
Amyloidosis involves the deposition of excess amounts
of insoluble, fibrous amyloid protein in the extracellular
spaces of organs and tissues. The causes are multiple,
including neoplastic disease, inflammatory disease and
advancing age. There is a rare hereditary form. All
organ systems can be involved, with cardiomyopathy,
macroglossia and vessel disease. The other conditions
can produce oedema, bruising and abnormal hormone
production, but not the complete clinical picture as
described above. Amyloidosis is usually well advanced
by the time it is diagnosed.

Question 40
Amelie is a severely depressed 29-year-old woman with
suicidal thoughts who is two months pregnant. Which of
the following approaches to treatment would be MOST
appropriate?

a) Gastric lavage

a) Counselling and reassurance

b) An emetic

b) Termination of pregnancy

c) Chest x-ray

c) Treatment in hospital
oxidase inhibitor (MAOI)

d) Intravenous saline
e) Methicillin

d)
Treatment
antidepressant

at

home

with

monoamine

with

tricyclic

e) Electroconvulsive therapy (E.C.T) in hospital


Answer: C
Kerosene is an aliphatic, highly volatile hydrocarbon
which is poorly absorbed from the gastrointestinal tract.
Pneumonitis through aspiration of fumes is the
predominant toxic mechanism in children and
respiratory distress can be severe and occur rapidly.
While a chest xray is not useful for the prediction of lung
involvement, serial chest xrays are important to monitor
progression. In children who present with lethargy, fever
or respiratory signs in the first 4 hours 80% develop
pneumonitis. Gastrointestinal irritation is common with
nausea and vomiting. There may also be a high fever
within 30 minutes of ingestion. Management should be
conservative and decontamination (emesis or gastric
lavage) should not be attempted - it merely increases
the risk of aspiration, and development of pneumonitis.
IV saline may be required if haemolysis from the
kerosene occurs and hypotension develops.

Answer: E
In severely depressed pregnant women ECT has been
shown to be safe and effective both antenatally and post
partum. Tricyclic antidepressants have been used for
over 40 years and are a good choice in a supervised
setting but not if the woman is suicidal because of the
risk of overdose. MAOIs have not been shown to be safe
in pregnancy.

Block 5
Question 1
Audrey is a 61 year old widow who lives alone. She has
become very anxious about leaving her house to go
shopping, or to attend appointments, like visits to the
doctor, since viewing a TV new story about the rise in
daytime home burglary. She finds that she has to check
and recheck that she has closed and locked all windows
and doors over and over again, before she can reduce
her anxiety enough to leave her house. This usually
takes more than an hour. Some weeks, she does not go
out at all, because she still feels anxious after this
extensive checking procedure. In such cases her
daughter does her shopping for her. Audrey does not
have anxiety about other things. What is the most likely
diagnosis?
a) Generalised anxiety disorder
b) Obsessive compulsive disorder
c) Posttraumatic stress disorder

(50% of cases) and a fluctuating level of consciousness


occurs in 35% of cases. Headaches, localised
neurological symptoms and a change of personality may
also occur.

Question 3
Jane (age 28) and her husband, Mike, have been trying
to have a child for 18 months. Mike has one child with a
previous partner. Jane's menses started at age 12 and
they have always been infrequent, irregular and
sometimes very heavy. Jane used the combined oral
contraceptive pill (Diane 35) for 10 years but stopped all
contraceptives 2 years ago. Recently Jane has been
trying to lose weight. At a height of 165cms, she weighs
85kg. On examination Jane appears normal but she
relies heavily on waxing to remove embarrassing facial
and lower abdominal hair. What is the MOST LIKELY
diagnosis?

d) Agoraphobia

a) Endometriosis

e) Panic disorder

b) Post-pill infertility
c) Polycystic ovarian syndrome

Answer: B

d) Pituitary prolactinoma

Audrey displays the features of obsessive compulsive


disorder. This is characterised by (a) obsessive thoughts
and/or compulsive behaviour that impair everyday
functioning, e.g. fears of contamination by germs,
repeated handwashing, checking windows and doors
etc; (b) the disruptive behaviours are undertaken to
relieve the anxiety, and (c) they take up more than one
hour per day. The other conditions listed are also forms
of anxiety disorders.

e) Hypothyroidism

Question 2
Mavis is 82 years old and recently she fell, landing face
down on the floor. She was very shaken and had
bruising about her lower face. Two weeks later her
family started to notice that Mavis seemed very
withdrawn and was sleeping a lot more than usual.
Mavis would spend the whole day in bed and she was
not really herself. When her family visited, she was
increasingly abrupt and moody. Mavis was usually very
gentle and quietly spoken. What is the MOST LIKELY
diagnosis?
a) Subdural haemorrhage
b) Extradural haemorrhage
c) Dementia
d) Stroke
e) TIA

Answer: C
Polycystic ovarian syndrome (PCOS) is characterized by
oligoamenorrhoea, hirsutism, acne, infertility, obesity
and insulin resistance. Menarche occurs at the usual
time and androgen excess becomes apparent during
puberty with development and persistence of hirsutism
and/or acne. Diagnosis is largely based on clinical
evaluation. Endometriosis is more associated with
dysmenorrhoea than irregular cycles. Prolactinoma and
hypothyroidism may cause oligoamenorrhoea but not
androgen excess. Prolonged use of the combined oral
contraceptive pill is not associated with infertility after
the cessation of its use.

Question 4
Mr Davy has had severe intermittent pain in the right
side of his back, radiating into his right groin and to the
tip of his penis. It has been present for the last 6 hours.
He feels continuously nauseous, and with every spasm
of pain, he feels he cannot lie still but must move
around. Sometimes, curling himself into a tight ball
helps. He has had one similar, but less severe episode of
pain one year ago that resolved spontaneously. On
examination he is afebrile and his urine has only a trace
of red blood cells. What is the MOST LIKELY diagnosis?
a) Appendicitis

Answer: A
Subdural haemorrhage may be insidious in onset, and
the elderly are particularly susceptible due to brain
shrinkage. A history of trauma may not be recalled

b) Urinary tract infection


c) Pyelonephritis
d) Ureteric calculi
e) Diverticular disease

Answer: D
Renal calculi (stones) may be asymptomatic. However
calculi in the ureters commonly cause pain from the loin,
into the groin and/or pain in the tip of the penis. There
is usually no penile redness or discharge and few other
abdominal signs are present, unless urinary obstruction
is occurring with urethral calculi. Haematuria and loin
tenderness are common.
Question 5
John is a 28 year old unemployed man with multiple
complaints, including headache, low backache, upper
abdominal pain, pain in both feet, nausea, bloating,
impotence and weakness in both forearms and left leg.
Physical examination shows no abnormal clinical signs.
Previous investigations including chest X-ray, full blood
count, biochemical profile and abdominal ultrasound
show no abnormality. What is the MOST LIKELY
diagnosis?
a) Factitious illness
b) Munchausen's syndrome
c) Conversion disorder
d) Hypochondriasis
e) Somatisation disorder

Answer: E
In somatisation disorder the patient has multiple
physical complaints referable to different organ systems,
including at least four pain, two gastrointestinal, one
sexual and one pseudoneurological symptom(s) which
are not consistent with any specific diagnosis. There is
significant impairment of social, occupational or other
important area of functioning. Treatment involves
behaviour modification and limitation of further
investigations.

Question 6
Myra, a 38 year old bank teller, presents with a painful
right lower leg. On examination, she has some dilated,
tortuous veins mostly on the posterior and lateral
aspects of her calf. There is an area of redness and heat
over one of these veins, and a firm cord like lump in the
vein, 3 cm long, which is tender to touch. The MOST
correct statement is:
a) Myra requires antibiotic treatment with
flucloxacillin
b) There is a small risk of extension into deep
veins
c) Myra should have subcutaneous low
molecular weight heparin while awaiting a
venous Doppler scan

extension is suspected it would be preferable to confirm


this by doppler ultrasound before commencing
anticoagulants. Varicose veins have many risk factors,
one of which is prolonged standing. This increases
hydrostatic pressure leading to chronic venous
distension and secondary valvular incompetence.
Women are particularly susceptible as the vein walls
become more distensible under the cyclic influence of
progesterone. The condition is likely to resolve
spontaneously over a few days. Non-steroidal antiinflammatory agents may be used to reduce pain and
local
inflammation,
and
graduated
compression
stockings may be helpful if the condition does not
resolve quickly. Thrombophlebitis is not usually infective
however antibiotics may be used in the case of
persistent or severe symptoms.

Question 7
Little Andrew, aged 18months, was 'helping' Dad in the
shed, when he began screaming and rubbing his eyes.
He had climbed onto the workshop bench, on which was
kept a variety of potential ocular hazards. Which of the
following substances is potentially MOST harmful to
Andrew's eyes?
a) Methylated spirits
b) Superglue
c) Acetic acid
d) Dog shampoo
e) Powdered cement

Answer: E
Cement is alkaline, and alkaline burns are more
dangerous than those from other chemicals. Alkali has
the potential to penetrate the cornea and gain access to
the anterior chamber, causing uveitis, secondary
glaucoma and cataract. Alcohols and solvents cause
severe pain initially but although the epithelium is burnt,
it tends to regenerate quickly. Superglue, while it may
cause distress in gluing eyelids together, is actually not
harmful to the eye- in fact it is sometimes used in
treatment of corneal wounds. The weak acid, and the
dog shampoo, will both cause more irritation than actual
damage. First aid treatment for any substance splashed
into an eye is profuse irrigation.

Question 8
Fred is a 74 year old hypertensive man who has been
found to have a 55mm fusiform abdominal aortic
aneurysm, discovered when he had an abdominal
ultrasound for right flank pain two days ago. Of the
following, which is NOT a risk factor for rupture of Fred's
aneurysm?

d) This condition is unrelated to her occupation

a) Persistently elevated mean arterial pressure

e) The condition is unlikely to resolve without


specific treatment

b) The fact
cigarettes/day

that

he

still

smokes

c) His chronic obstructive pulmonary disease


Answer: B

d) The fact that Fred is male

Myra has superficial thrombophlebitis, a relatively


common problem. In this particular site, it is likely to be
in the short saphenous vein system and the risk of
extension to the deep system via perforating veins is
small. However, it is not negligible. If deep vein

e) The size of the aneurysm

15

Answer: D
Risk factors for the development of an abdominal aortic
aneurysm (AAA) include smoking, increasing age,
hypertension, family history, chronic obstructive
pulmonary disease (COPD) and being male. Risk factors
for AAA rupture are an elevated mean arterial pressure,
continuing to smoke, more severe COPD and having an
aneurysm that is either rapidly enlarging or is measured
at >50mm diameter. Although women have a lower
incidence of AAAs which tend to be smaller, they have a
much higher risk of rupture. In this scenario, the flank
pain may well be an indicator of expansion of the
aneurysm.

Question 9
Colin is 22 years old. His right arm was amputated
above the elbow when it became caught in the industrial
mulcher he was using. His mate tied his own T shirt
firmly around the stump and brought him to hospital. On
arrival, 15 minutes later, the T shirt is soaked, and
blood is trickling out. Colin is pale, his skin is cool and
clammy, and he looks anxious. His pulse is 110
beats/min and his BP 130/95 mmHg. His respiratory
rate is 20 breaths/min. Capillary refill time is 5 seconds.
You are able to insert an intravenous cannula in his left
arm. Which fluid orders are MOST appropriate in this
circumstance?
a) 1 litre Normal saline as a bolus, then 1 Litre
4% dextrose in 1/5N saline
b) 2 units O negative blood
c) 500ml normal saline

105/70mm Hg. There appears to be diminished


excursion of his right chest wall, and the breath sounds
are hard to hear on the right. There is hyperresonance
to percussion on of the right chest. Your IMMEDIATE
response should be?
a) Arrange an urgent chest X ray
b) Perform
intubate

rapid

sequence

induction

and

c) Insert a thoracostomy tube in the right fifth


intercostal space in the anterior axillary line
d) Insert a wide bore needle in the right second
intercostal space
e) Insert a wide bore needle in the left second
intercostal space

Answer: D
Brendon has almost certainly developed a right tension
pneumothorax, as indicated by his increasing dyspnoea ,
and the physical signs described above. This is a lifethreatening
condition
which
requires
urgent
management. Decompression with a wide-bore needle in
the second intercostal space, in the midclavicular line of
the affected side is potentially life-saving, and allows
time for the more complex procedure of the tube
thoracostomy to follow. Tension pneumothorax is a
clinical
diagnosis, and emergency treatment should not be
delayed for X ray confirmation. Intubation and
ventilation may turn a simple pneumothorax into one
under tension. It is not indicated in this situation.

d) 500ml colloid
e) 1.5 L Normal saline

Answer: E
Colin is a young adult, apparently fit. His signs indicate
that he has suffered a class 2 haemorrhage, and has
lost approximately 15-30% of his total blood volume or
750-1500ml. So far his body has compensated well, but
this may not be sustained. He requires replacement of
volume and the most commonly recommended fluid is
an isotonic crystalloid such as normal saline. O negative
blood is not required in this
situation. There would normally be time to obtain cross
matched blood if bleeding could not be controlled.
Hypotonic saline/dextrose solutions are not appropriate.
These fluids are used to maintain fluid balance in a
normovolaemic, normonatraemic patient and do not
restore intravascular volume in the volume-depleted
patient.500ml of normal saline is not sufficient. While
there are some theoretical advantages to using colloid
as the replacement fluid, there is little evidence of
improved outcome from using this instead of crystalloid.
500ml of colloid is not sufficient on its own. 1-2 litres as
the initial bolus, for an adult of average build is
appropriate in this circumstance, then the patient's
response should be assessed.

Question 10
Brendon is a 35 year old man who has been involved in
a motor vehicle accident. He was wearing his seat belt,
but it did not hold and he was thrown against the
steering wheel. He is anxious and increasingly
dyspnoeic. His pulse is 126 beats/minute and his BP

Question 11
Mary, aged 65, had a laparotomy for resection of a
bowel cancer seven days ago. She has been progressing
well, but has just noticed some pinkish fluid leaking
from her wound. Which of the following is TRUE
regarding this situation?
a) This complication occurs in 10% of older
patients undergoing abdominal surgery
b) The wound will require urgent surgical repair
c) There is a mortality rate of 1% associated
with this complication
d) If the wound breaks down, it must heal by
secondary intention
e) The appropriate management is intravenous
antibiotics

Answer: B
The serosanguinous discharge heralds dehiscence of the
wound,
and
after
undertaking
any
necessary
resuscitation and preparations for theatre, Mary should
return to theatre as soon as possible. Early wound
dehiscence is a serious complication, usually occurring
around the 7th to 10th post-operative day. It occurs in
fewer than 1% of laparotomy wounds but can have a
mortality of around 30%. Risk factors include poor
nutritional state, malignancy, obesity, prolonged
surgery, infection or coughing. The wound cannot be left
to heal by secondary intention. Intravenous antibiotics
may form part of the management but will not suffice
alone.

Question 12

Answer: A

Kevin, a 45 year old labourer, had a laparotomy five


years ago when he suffered a ruptured appendix. He has
recently noticed a dragging sensation in the region of his
scar, especially when lifting heavy objects at work, and
now presents with a swelling of 1.5 cm diameter in the
medial end of his scar. Concerning Kevin's problem,
which of the following is TRUE?

As increasing age is a risk for colorectal cancer, a


patient over the age of 40 who presents with PR
bleeding should have a digital rectal examination and be
investigated by colonoscopy. If this is not available a
flexible sigmoidoscopy and double contrast barium
enema would be satisfactory. Rectal bleeding is a
common symptom of haemorrhoids, but a rectal
neoplasm may also cause PR bleeding. Even in the
presence of obvious haemorrhoids patients at increased
risk for colorectal cancer should be investigated. FOBT is
a screening test , not a diagnostic investigation.

a)
Kevin's
lean,
muscular
predisposes him to this problem

body

type

b) The fact that the scar is paramedian and in


the lower abdomen predisposes to this problem
c) The problem is of nuisance value only, as
only fatty tissue protrudes into the swelling
d) Kevin should have surgical repair as soon as
convenient
e) Kevin should wear an abdominal support
garment to prevent complications

Question 14
Florence, aged 50, has decided to have a
haemorrhoidectomy after months of unsuccessful
conservative management of her haemorrhoids. In
obtaining informed consent, you discuss with her the
potential complications of haemorrhoidectomy. Which of
the following is the LEAST likely complication?
a) Urinary retention

Answer: D
Kevin has an incisional hernia, which is a protrusion of
abdominal contents into the subcutaneous plane through
a defect at the site of a previous incision. Incisional
herniae should be repaired as soon as convenient
because they can increase in size over time and may
become very difficult to repair. More particularly, as with
most herniae, they may become irreducible, with
possible obstruction and strangulation of abdominal
contents including bowel. Incisional herniae are more
common in obese patients in whom there is fatty
infiltration of the tissues, increased intra- abdominal
pressure and reduced muscle tone. They are more
common in midline and upper abdominal scars. There is
no evidence that any supportive garment will prevent
complications in an incisional hernia although it may
relieve discomfort.

b) Post-operative bleeding
c) Sepsis
d) Faecal incontinence
e) Pain
Answer: C
Sepsis is fortunately a very rare complication of
hemorrhoidectomy.
Urinary
retention
occurs
in
approximately 5-10% of cases and may be due to spinal
anaesthesia and/or the use of IV fluids and urinary
catheter intraoperatively. Bleeding is uncommon but
may be severe. It can occur in the first 24 hours or 7 to
10 days later due to local infection. Pain is fairly
common and may be severe. It is associated with faecal
impaction and incontinence. Later rare complications
include fissures, fistulae and anal stenosis.

Question 13
Peter is 47 years of age and presents with a single
episode of bright red bleeding per rectum (PR). which he
noticed after passing a bowel motion this morning. He is
unaware of any significant family history of colorectal
problems. On examination Peter has some obvious
haemorrhoids but nothing else of note on rectal or
proctoscope
examinations.
What
is
the
MOST
appropriate advice for Peter?
a) In view of his age he should have a
colonoscopy to investigate this bleeding
b) As there is an obvious cause for his
bleeding, no further investigation is needed at
present
c) As he has no significant family history of
colorectal disease, he only needs reassurance
d) Monitoring with 6 monthly faecal occult
blood testing (FOBT) is required
e) He should have a trial of increased fibre in
his diet and review the haemorrhoids in 3
months

Question 15
Brian, a 52 year old man, walks awkwardly into your
rooms. He complains of severe pain, which he indicates
as being quite deep in his rectum. He says the pain
began earlier in the day but has become much worse in
the last hour and he it feels like 'something coming
down' in his back passage. Which of the following
statements MOST accurately describes Brian's condition?
a)
Brian
has
haemorrhoid

thrombosed

external

b) Brian's deep pain is due to prolapsing


internal haemorrhoids
c) Brian has a rectal prolapse
d) Brian has grade three haemorrhoids
e)
Brian
haemorrhoids

has

strangulated

internal

Answer: E

Answer: D

The pain from strangulated internal haemorrhoids is


typically felt as a deep pain. Prolapsing internal
haemorrhoids can cause perianal pain by causing a
spasm of the anal sphincter complex. If the
haemorrhoids become trapped by the spasm, they
become engorged with secondary venous and later
arterial thrombosis, and become irreducible. This is
known as 'strangulation' and results in deep seated
pain, especially if necrosis and ulceration occur. The
pain of thrombosed external haemorrhoids is felt
perianally. Rectal prolapse is rarely painful. Brian's
haemorrhoids are now irreducible, so are no longer
grade 3 (require manual reduction).

Unilateral renal agenesis is not uncommon and the


solitary kidney compensates by hypertrophy and
maintains normal renal function. It is usually
accompanied by ureteral agenesis. Potter's syndrome is
bilateral renal agenesis and it is fatal.

Question 16
Jason is a 30 year old mature age medical student. He
has been hospitalised following a haematemesis due to a
Mallory-Weiss tear. Jason asks for an explanation about
Mallory-Weiss tears. Which of the following statements
is FALSE?
a) Mallory-Weiss tears are tears in the mucosa
of the lower oesophagus
b) Haematemesis in Mallory-Weiss tears is
always preceded by retching or vomiting
c) Bleeding from Mallory-Weiss tears stops
spontaneously in 80-90% of patients

Question 19
Jane is 45 years of age and she has noticed the
following changes in herself over the last 4 months. She
has lost weight, her eyes feel dry, but they are
constantly watering and she feels irritable and 'on edge'
and occasionally experiences palpitations. Her periods
have become irregular, her hair is thinning and her
fingernails seem very brittle. Her father and older sister
experienced the same symptoms when they were 40
years of age. What is the MOST LIKELY diagnosis?
a) Graves' disease
b) Toxic adenoma
c) Simple diffuse goitre
d) Multi-nodular goitre
e) Hashimoto's thyroiditis

Answer: A

The classical presentation of Mallory-Weiss syndrome is


haematemesis from a tear in the oesophagus, brought
on by prolonged vomiting of any cause. It is often
associated with alcoholic excess but this is NOT always
the case. Haematemesis may occur without prior
retching or vomiting. The tear is typically a longitudinal
one in the mucosa of the lower oesophagus close to the
gastro-oesophageal junction. The bleeding settles
spontaneously in 80-90% of cases of Mallory-Weiss
tears. Not all MW tears present with haematemesis. In a
small proportion, melaena, haematochezia, syncope or
abdominal pain are the presenting symptoms.

Graves' disease is characterized by hyperthyroidism and


one or more of the following: goitre, exophthalmos, and
pretibial myxoedema. It is an auto-immune disorder
that has a genetic component and commonly presents in
women aged 40 -50 years. Toxic adenoma can occur at
any age.It usually presents as a single thyroid nodule
not a goitre, and hyperthyroidism. Simple diffuse goitre
occurs mostly in younger women aged 15-25 years. The
thyroid gland is enlarged but the person is euthyroid.
Multi-nodular goitre is often a simple diffuse goitre that
has progressed as the person has become 'middle-aged'
or elderly. The goitre is 'lumpy', not diffusely enlarged
and initially the person is euthyroid but may become
hyperthyroid in the long-term. Sometimes it causes
difficulty with swallowing and breathing if large.
Hashimoto's thyroiditis is a chronic inflammation of the
thyroid caused by autoimmune factors. It causes
painless enlargement of the thyroid gland or fullness in
the throat and many patients have hypothyroidism when
first seen. Other forms of autoimmune disease are
common.

Question 18

Question 20

Hugo did not realise until he volunteered to be a kidney


donor that he had been born with only one kidney.
Which ONE of the following statements is TRUE?

Amanda, 47 years, has noticed her right eyelid is higher


than her left and her right eye seems more prominent.
She first noticed she can apply her mascara to her left
eyelashes easier if she tilts her head back and looks
upward. Her contact lenses still fit perfectly. Amanda is
otherwise well with no other symptoms or signs. What is
the MOST LIKELY diagnosis?

d) Alcoholic binge drinking may be associated


with Mallory-Weiss tears
e) Haematemesis is not a universal symptom of
a Mallory Weiss tear

Answer: B

a) Hugo's
syndrome

condition

is

known

as

Potter's

b) Unilateral renal agenesis is uncommon


c) Usually in unilateral renal agenesis there are
still two ureters
d) In unilateral renal agenesis the solitary
kidney maintains normal renal function
e) Hugo needs an annual ultrasound scan of his
solitary kidney

a) Bell's palsy
b) Hyperthyroidism
c) Myasthenia gravis
d) Horner's syndrome
e) Optic nerve glioma

Answer: D
Ptosis is drooping of the upper eyelid associated with an
inability to elevate the lid completely. Nerves from the
sympathetic chain innervate the superior tarsal muscle
causing unilateral partial ptosis that can be overcome by
looking upward. Horner's syndrome includes unilateral
partial ptosis, ipsilateral constricted pupil and ipsilateral
lack of sweating of the face. Myasthenia gravis usually
causes bilateral partial ptosis. Hyperthyroidism causes
protruding eyes (proptosis/ exophthalmos) which may
be unilateral. Bell's palsy (VII nerve paralysis) prevents
the patient from forcefully closing their eyes and they
have bilateral wide palpebral fissures. Optic nerve
glioma causes painless progressive proptosis.

Question 21
Michelle needs a transfusion after a major motor vehicle
accident. In the accident her pelvis was fractured, both
femurs have mid-shaft fractures and she sustained a
hemothorax requiring a chest drain. Michelle was
trapped for an hour before the fire rescue could cut her
out of her vehicle. Michele has blood group O Rh
positive. Which of the following statements is TRUE?
a) Michelle has type A antigens on her red
blood cells
b) Naturally occurring A and B antigens are
called isoagglutinins

shallow respiration, no abnormal physical findings. Of


the following, which is the MOST LIKELY diagnosis?
a) Agoraphobia
b) Posttraumatic stress disorder
c) Generalised anxiety disorder
d) Panic disorder
e) Acute psychosis

Answer: D
Karen's story displays the features of a panic attack
which is the cardinal manifestation of panic disorder.
Patients with panic disorder experience repeated
unexpected attacks of intense, disabling anxiety. In
between attacks they experience at least one month of
worry about having further attacks and/or fear of losing
control, going mad or dying. Agoraphobia is an irrational
fear of being trapped in a place from which escape is
impossible. Patients with posttraumatic stress disorder
are repeatedly distressed by re-experiencing highly
traumatic events. Generalised anxiety disorder involves
persistent
excessive
and/or
unrealistic
worry
accompanied by other signs and symptoms, such as
muscle tension, restlessness and feeling on edge. Acute
psychosis is a severe mental disturbance involving
hallucinations and/or delusions.

c) Michelle has anti-A and anti-B antibodies


d) Persons with Type O blood are "universal
recipients"
e) Michelle lacks the D antigen

Answer: C
The ABO blood group system is the most important in
transfusions. Persons with Type O blood are "universal
donors" because their red blood cells lack A or B
antigens. Type O individuals produce their own anti-A
and anti-B. However, their cells are not recognised by
any naturally occurring anti-A or anti-B antibodies
(otherwise known as isoagglutinins), when their red
blood cells are transfused. The Rh system is the second
most important blood group system in pretransfusion
testing. Rh 'positive' individuals have the D antigen of
the Rh system, while people lacking the D antigen are
Rh 'negative'.

Question 23
Sue is 30 years old and concerned she will get breast
cancer because her mother had breast cancer diagnosed
when she was 45 years of age. Which of the following
statements is TRUE regarding breast cancer?
a) BRCA1 and BRCA2 mutations account for
60% of breast cancer cases
b) Sue does not have an increased risk of
breast cancer because her mother had breast
cancer
c) Breast cancer is a disease of younger women
d) Sue should have bilateral mastectomies to
prevent breast cancer developing
e) If Sue does develop breast cancer she is
most likely to develop it after she is 50 years of
age

Question 22

Answer: E

Karen is a 21 year old university student. She is


accompanied to your consulting rooms by two women
friends who observed her collapse this morning in a
bathroom of their university residence. Karen tells you
that soon after she woke today she suddenly began
feeling unwell, with intense fear, palpitations, sweating,
shortness of breath, nausea and tingling in her fingers. .
She does not remember anything after entering the
bathroom. This is the third similar attack that she has
experienced during the past two months. After the first
attack, she worried that she 'might be going crazy' but
postponed seeking medical advice for fear of being
institutionalised, like one of her aunts who has
schizophrenia. She does not smoke, drink alcohol or use
illicit drugs. The only medication she takes is
paracetamol occasionally for headaches. On examination
you find: PR 85/min, BP 135/95 mm Hg, moist palms,

Although mutations in BRCA1 and BRCA2 are associated


with an increased risk of breast cancer, and the lifetime
risk of developing breast cancer in women who have
these mutations approaches 80%, these lesions account
together for less than 10% of breast cancer cases.
Women who have first-degree relatives who have
developed breast cancer do have an increased risk of
developing breast cancer themselves, and if their firstdegree relative with breast cancer was diagnosed before
age 50 they have a higher risk of developing breast
cancer than women whose first-degree relative was
diagnosed after age 50. However, in all cases, breast
cancer is uncommon in young women. Furthermore,
most women with affected first-degree relatives with
breast cancer who themselves develop breast cancer do
so after 50 years of age In the absence of mutations in
BRCA1 or BRCA2, the risk associated with a positive

family history does not seem of sufficient magnitude to


justify routine bilateral mastectomy.

Question 24
At birth Sammy has a cleft lip but otherwise looks
normal. Sammy's parents are very distressed about this,
and are concerned to know if Sammy has anything else
wrong which they cannot see yet, or that may develop
when he's older. Which of the following is TRUE?
a) Sammy is likely to have Pierre Robin
syndrome and a cleft lip is just part of this
syndrome
b) Sammy has a cleft lip due to his mother's
use of antidepressants
c) Sammy is likely to have this isolated
abnormality and no other problems except the
cleft lip
d) Sammy is likely to have congenital
dislocation of the hips as well as his cleft lip
e) Cleft lip and cleft palate are associated with
talipes (clubfoot deformities)

Answer: C
The cleft may vary from involvement of the soft palate
only, to a complete cleft of the soft and hard palates,
the alveolar process of the maxilla, and the lip. The
mildest form is a bifid uvula. These children have normal
intelligence and development. Cleft lip with or without
cleft palate occurs in 1:700-1000 live births, more often
in Asian groups and less often in African Americans;
more often in males. Cleft palate alone occurs in 1:2000
across all races with slightly more females affected.
There may be genetic and environmental factors
including maternal smoking and use of alcohol, retinoic
acid and anticonvulsants. Associated anomalies occur in
about 15 to 20% of cases of cleft lip with or without cleft
palate but in 50%
of cases of cleft palate alone. Pierre Robin syndrome
typically presents with micrognathia (small mandible)
and a cleft soft palate. Congenital dislocation of the hip
seems to be secondary to laxity of the ligaments around
the hip or to in utero positioning. Clubfoot (talipes)
deformities, result in the foot being plantar flexed,
inverted, and markedly adducted. Neither of these
congenital abnormalities is associated with cleft lip or
palate.

Question 25
Mandy has had migraines since she was a teenager.
They are the classical migraine with a prodrome when
she is clumsy, yawns a lot, is tired, has a stiff neck and
feels irritable. Then she gets the aura, with 'sparks' in
her vision. Then she gets a severe headache that starts
at the back of her neck and moves to one of her temple
areas and then her forehead. She feels sick and wants
to curl up in bed, in a dark room, and let the headaches
pass, which it usually does in about 6 hours. Which of
the following statements is TRUE regarding migraine
headaches?
a) Migraine headaches are equally common in
women and men
b) In Australia 30% of the population have
migraine headaches

c) Migraine is rare in children less than 10


years of age
d) The most common form of migraines has a
prodrome and an aura
e) Migraine is accompanied by nausea in 90%,
vomiting in 60% and diarrhoea in 15% of
attacks

Answer: E
There are two main types of migraine: classical migraine
(migraine with aura) and common migraine (migraine
without aura), the latter accounting for the majority of
migraine headaches. About 10% of the population in
Australia have migraine.
Migraine usually starts during the teenage years or early
adult life and occurs more commonly in women than
men (ratio 3:1). In children the incidence is 3-7%.
Migraine may be accompanied by a variety of symptoms
other than the typical nausea, vomiting and photophobia

Question 26
Dimitri is a 45 year old man who presents with
insomnia. He goes to sleep at night without difficulty,
but wakes frequently from distressing dreams in which
he is being forced to watch people being tortured. Then
he has great difficulty in going back to sleep. Dimitri has
been in Australia for two years, having emigrated from
Kosovo, where he was imprisoned for a year. His wife
says he had a 'bad experience' while in detention, but
will not talk about it. Over the past six weeks, he has
become irritable, prone to outbursts of anger and has
begun drinking heavily. This has led to marked tension
in the home and Dimitri's workplace. Which of the
following is the probable cause of Dimitri's distress?
a) Acute stress disorder
b) Posttraumatic stress disorder
c) Panic disorder
d) Phobic disorder
e) Generalised anxiety disorder

Answer: B
Posttraumatic stress disorder is an anxiety disorder of
more than one month's duration, consequent upon a
severe traumatic experience in the individual's past, and
which s/he now re-experiences in one or more ways
(e.g. flashbacks or dreams). This is accompanied by
avoidance of stimuli which recall the event, numbing of
the individual's responsiveness, symptoms of arousal
(e.g. insomnia) and distress or social/occupational
impairment.(see refs for full diagnostic criteria). In
contrast , an
acute stress disorder develops soon after the traumatic
experience. A panic attack is the cardinal manifestation
of panic disorder. Patients experience intense, disabling
anxiety and may fear they are losing control, going mad
or dying. Generalised anxiety disorder involves
persistent
excessive
and/or
unrealistic
worry,
accompanied by other signs and symptoms, such as
muscle tension, restlessness and feeling on edge.
Patients with phobic disorders display marked fear of
objects or situations which provoke an immediate
anxiety reaction.

Question 27
Jane is 24 years of age and 10 weeks pregnant with her
first child. She has just been diagnosed with her first
ever urinary tract infection. Which drug would you
choose to treat Jane's urinary tract infection?
a) Trimethoprim
b) Cephalexin

sympathetic overactivity or cerebral compromise,


resulting from hypoglycaemia, rapidly progress to coma,
if untreated. Hypoglycaemic coma commonly occurs in
well-controlled diabetic patients, and is due to their
diabetic medications eg: longer acting sulphonylureas.
However blood glucose should always be tested (dipstick
and laboratory confirmation) in an unconscious patient
(diabetic or not) and hypoglycaemia assumed to be the
cause of any coma, until proven otherwise.

c) Amoxycillin
d) Norfloxacin
e) Erythromycin

Answer: B
The important time for teratogenic effects of drugs given
in pregnancy is in the first trimester. All drugs, if
possible should be avoided in the first 12 weeks of
pregnancy. However if Jane has a urinary tract infection
she requires treatment. Trimethoprim and norfloxacin
(usually used to treat pyelonephritis) are category B3
drugs in pregnancy and should be avoided. Amoxycillin
and Cephalexin are both category A in pregnancy,
however
amoxycillin
is
only
recommended
if
susceptibility of the organism is proven. Erythromycin is
also category A but unsuitable in the management of
urinary tract infections.

Question 28
May was found at home in a coma and brought into
hospital, where she is now recovering well. May is 80
years of age, and she has been well most of her life, but
in the last 5 years she has gained about 10kg in weight.
During the past week or two, before she was brought
into hospital, May has been tired, sleepy, 'dry as a chip',
forever running to the toilet to pass urine, and yet she
had been unable to drink enough to satisfy her thirst.
What was the MOST LIKELY diagnosis when May was
brought into hospital?
a) Diabetic ketoacidotic coma
b) Hypoglycaemic coma
c) CVA with coma

Question 29
Mike has come to you to discuss vasectomy. He is 45
years of age, and he has three children to his current
partner Sam. Sam
has tried many different
contraceptives, but none have been satisfactory. She
has finally told Mike he has to do something about
contraception for them now they have had all the
children they want. Mike is very nervous about any type
of surgery, especially if it involves his genital area. He
has never been near a surgeon in his life. Which of the
following statements is CORRECT? Vasectomy:
a) Is not as permanent as male sterilisation
b) Is not effective immediately
c) Is totally functionally reversible
d) May be followed by a reduced testosterone
level
e) May result in a reduced volume of semen
production

Answer: B
Vasectomy is sterilisation of the male and it involves a
small incision in the scrotal skin under local anaesthetic.
The vas deferens is separated from its blood supply and
approximately 1cm of it is removed between ligatures.
Post-vasectomy it takes up to 3 months for the sperm to
be eliminated from the ejaculate (the volume of sperm
in the vas deferens between the point of excision and
the tip of the penis). Vasectomy is to be considered
irreversible as microsurgery may repair the vas
deferens, but sperm function may never return ( due to
the production of sperm antibodies). There is no change
to male testosterone levels, balding patterns or libido.

d) Hyperosmolar non-ketotic coma


e) Hypothyroid crisis

Answer: D
Hyperosmolar non-ketotic coma (HONC) occurs in
elderly patients with Type 2 diabetes mellitus, but the
history of diabetes is usually unknown. It has an
insidious onset that includes polyuria and polydipsia,
severe dehydration, and an impaired level of
consciousness, which correlates with plasma osmolality.
Coma is usually associated with an osmolality
>440mmol/l. Respiration is usually normal. Patients
may rarely present with a CVA, seizures or an MI, but
the underlying disorder is primarily diabetes. Blood
glucose is usually >40mmol/l, there is severe
hypernatraemia and dehydration, with a relatively
normal arterial pH, unless there is coexisting lactic
acidosis. Rehydration and insulin are the mainstays of
treatment and causes of infection should be sought as
well as ECG changes consistent with infarct or
ischaemia. Diabetic ketoacidotic coma only occurs in
Type 1 diabetes. Hypoglycaemic coma has more rapid
onset than HONC. The preceding symptoms of

Question 30
Pamela, an 18 year old first year music student,
complains of disabling anxiety. She says she has always
been 'nervous in front of strangers', but her problem has
been aggravated since she started her music studies.
She feels well during the weekend, but is very anxious
during the week, and wonders whether she should
withdraw from the course. Her main problem is fear of
solo performances. Her tutor requires all students to
perform solo each week without prior warning for of a
group of staff members . Pamela finds this very
unnerving. She cannot think or play properly under
these conditions, and has 'frozen' and burst into tears
on more than one occasion. Which of the following is the
most likely diagnosis?
a) Generalised anxiety disorder
b) Panic disorder
c) Phobic disorder
d) Obsessive compulsive disorder
e) Posttraumatic stress disorder

Answer: C
The features of phobic disorder are (a) a marked
persistent fear of objects or situations, exposure to
which provokes an immediate anxiety reaction that may
take the form of a panic attack; (b) avoidance behaviour
to avoid the phobic stimulus; and (c) anxiety is
provoked only in specific situations. Pamela has a social
phobia, characterised by fear of social or performance
situations, where she is exposed to unfamiliar
individuals, or to possible evaluation by others.
Medication with selective serotonin reuptake inhibitors
(SSRIs) may be helpful, but the mainstay of
management is behaviourally focussed psychotherapy.
Generalised anxiety disorder, panic disorder, obsessive
compulsive disorder and posttraumatic stress disorder
are other anxiety disorders each with its characteristic
presentation.

Question 31
Alison has been taking the tricyclic antidepressant drug
(TCAD) amitriptyline for 6 years. She started taking it
when her husband John passed away with cancer. Alison
is "much better" now, as she has adapted to life without
John. She is sleeping well, her appetite has returned,
and even though she still desperately misses John, she
no longer avoids neighbours and friends, and she feels
less like crying every minute of the day. Alison stopped
her
amitriptyline
suddenly
last
week
without
consultation with her doctor. Which of the following is
NOT common after abrupt cessation of TCADs?
a) Cholinergic activation - abdominal cramps,
diarrhoea and vomiting
b) Sleep disturbance - insomnia and vivid
dreams
c) Somatic distress - flu-like symptoms and
headache
d) Cardiovascular symptoms - palpitations and
arrhythmias
e) Psychiatric symptoms - anxiety and agitation

Answer: D
All the other options are withdrawal syndromes
associated with withdrawal from tricyclic antidepressant
drugs. TCADs can cause adverse effects such as
orthostatic hypotension, conduction defects and
arrhythmias while they are being used. However upon
withdrawal of TCADs cardiovascular symptoms are not
common. Withdrawal from benzodiazepines is more
likely to be associated with cardiovascular symptoms
including palpitations, flushing and hyperventilation.

b) Mandy will develop renal failure within 12


hours
c) Mandy should be given oral methionine
d) Other commonly prescribed medication
taken at the time of a paracetamol overdose
will not alter subsequent liver damage
e) FFP (fresh frozen plasma) is the treatment of
choice when the prothrombin time is abnormal
following paracetamol overdose

Answer: A
N-acetyl cysteine (iv) is given to all severe paracetamol
overdoses (>10grams) presenting with symptoms or
abnormal investigations (liver function tests (LFTs),
prothrombin time (PT)). All patients with paracetamol
plasma levels on or above the "Normal" treatment line
(when plasma paracetamol levels are plotted against
time in hours), presenting up to 24hours following
ingestion, should also be given N-acetyl cysteine. Only
patients presenting within 10-12 hours, who are allergic
to N-acetyl cysteine, should be given oral methionine.
Oliguria and renal failure generally occur late (day 3
following ingestion) following paracetamol overdose.
However 10% of patients develop acute renal failure
from acute tubular necrosis. Vitamin K, 10mg, given
intravenously (iv) is preferable in paracetamol overdose
and FFP (fresh frozen plasma) should be avoided, unless
there is active bleeding. FFP may make future
management, including liver transplant more difficult.
Patients on enzyme-inducing drugs (e.g. phenytoin,
carbamazepine, rifampicin, phenobarbitone) or those
who are malnourished (e.g. anorexia, alcoholism)
develop paracetamol toxicity and require intervention at
lower plasma paracetamol levels than previously healthy
patients on no enzyme-inducing medications.

Question 33
Shamila is a 16 year old schoolgirl who consults you
because she is very unhappy at home and says she is
considering suicide. You assess her as being clinically
depressed. Which ONE of the following strategies would
you adopt NEXT to deal with the threat of suicide in this
case?
a) Refer Shamila to a psychiatrist
b) Referral to a local mental health crisis team
c) Admit Shamila urgently to the psychiatric
ward of the local hospital
d) Ask Shamila if she has made any suicidal
plans
e) Commence cognitive behavioural therapy
immediately

Question 32
Mandy ingested 30grams of paracetamol 18 hours ago,
and she is slightly nauseous and tearful but otherwise
asymptomatic. Mandy is an adult Caucasian female, 65
kg, with no pre-existing illnesses. She is a non-smoker,
does not drink alcohol and is on no other medications.
She has no known allergies. You ordered some
investigations when Mandy arrived at the hospital and
they show that she has elevated hepatic transaminases
(ALT,
AST),
prolonged
prothrombin
time
and
hypoglycaemia. Which of the following is TRUE?
a) Mandy should be given N-acetyl cysteine

Answer: D
When patients have suicidal thoughts, the treating
doctor should take careful note of the context. Patients
who have made definite plans to commit suicide, or who
have obtained the means with which to carry it our, e.g.
a weapon, are at much greater risk of killing themselves
than those who have simply contemplated the matter in
theory. In cases where there is serious intent to commit
suicide, the patient should be regarded as seriously
depressed and referred for urgent specialist attention.
How this is achieved will differ in different areas. In

some cases the best approach may be to refer the


patient to the local mental health crisis team. Where
such a team does not exist, urgent referral to a
psychiatrist or urgent admission to a psychiatric facility
is indicated.

Question 34
Marty is a 42 year old man who presents complaining of
chronic headaches. He says he has come to see you only
because his wife insisted. On questioning he is not very
informative but admits to having a few beers after work
most days. On examination you note his complexion is
flushed, there is facial telangiectasia and some
periorbital puffiness. His BP is 150/95 mm Hg. You
suspect that Marty's problems relate to hazardous
drinking. Which ONE of the following strategies would be
best for obtaining confirmation of your suspicions?
a) Confront Marty outright and demand the
truth about his drinking
b) Phone Marty's wife while he is with you and
ask her about his drinking
c) Administer an Alcohol
Identification Test (AUDIT)

Use

Disorders

d) Take a blood sample for a carbohydrate


deficient transferrin (CDT) test
e) Take a blood sample for a blood alcohol
concentration (BAC) measurement

Answer: C
Patients with a drinking problem often do not openly
acknowledge how much they are drinking, so other
means have to be employed to determine whether they
are drinking hazardously. The best approach is to
administer a questionnaire (such as AUDIT or CAGE)
which explores the patient's drinking pattern and its
potential effects on his/her life. Obtaining corroborative
information from family members is also helpful but
should not be the main approach to obtaining
information. Laboratory tests are also useful but there is
a considerable incidence of false
negatives. The CDT test is relatively insensitive - it
requires a consumption level of 60 or more g of alcohol
per day to record a positive result. The BAC will only be
positive if the patient has been consuming alcohol
during the preceding hours before the test.

Question 35
Myra is an 80 year old woman who is brought to your
consulting room by her daughter and son-in-law who are
concerned that she may have dementia because of her
increasing forgetfulness. Which ONE of the following
initial strategies would be best to determine whether
Myra may have dementia?
a) Take blood to measure thyroid function
b) Do a thorough neurological examination
c) Do a general physical examination including
urinalysis
d) Establish rapport and administer the Mini
Mental State Examination
e) Take a medication history and administer
the Alcohol Use Disorders Identification Test

Answer: D
The Mini Mental State Examination is the appropriate
test to examine the patient's orientation. It will detect
cognitive impairment, whether due to dementia,
depression or delerium. Differentiating these three
conditions will usually be possible by a thorough history
and examination.

Question 36
Tom is a 65 year old man who presents with fatigue and
poorly localised muscular aches and pains in the back
and legs. You suspect that he may be depressed but he
denies feelings of depression. Which of the following
alternatives would be the BEST way of confirming your
preliminary diagnosis?
a) Discuss Tom's symptoms with his wife
b) Administer
questionnaire

standardised

c)
Undertake
a
therapeutic
antidepressant medication

depression
trial

of

d) Refer Tom to a psychiatrist


e) All of the above

Answer: B
A number of standardised questionnaires are available
for the detection of depression. Among the simplest are
those promoted by the Beyond Blue website, designed
to assist GPs in the diagnosis of depression, viz the K10
and SPHERE questionnaires. The other options could all
be helpful but are not recommended diagnostic
strategies in themselves. Referral to a psychiatrist is
only recommended for problematic or severe cases.

Question 37
Molly is a 34 year old woman who presents with chronic
back pain following a fall at work one year ago. She is
seeking
a
repeat
prescription
for
oxycodone
(Oxycontin), a powerful long-acting opiate analgesic.
She is not receiving any other mode of treatment and is
not undertaking back exercises. Molly is a trained nurse
but has been unemployed since the accident because of
disabling pain and is seeking a disability pension. What
would be the most appropriate NEXT STEP toward
solving Molly's problem?
a) Provide a repeat prescription to reduce the
number of times she needs to come to see you
b) Help her complete the necessary paperwork
for the pension
c) Detail her drug use and assist her to switch
to non-narcotic analgesia
d) Refer her to a multi-disciplinary pain clinic
e) Encourage her to begin back strengthening
exercises

Answer: D
Molly's problem is chronic because of its duration. Her
case raises several issues: (i) How severe is her pain
and does she really need a powerful analgesic? (ii) Is
she misusing her prescription because she has become

dependent? (iii) What is the best approach to her


problem? The best way of answering these questions is
to refer her to a multi-disciplinary pain clinic, where she
can obtain the benefit of assessment and advice by
appropriate experts, as necessary, e.g. orthopaedic
surgeon,
anaesthetist
(specialising
in
pain
management), psychiatrist. The other options could
assist but are unlikely to provide a solution to Molly's
problems.

Question 38
Roberto is a 67 year old patient who is depressed
following the recent death of his wife, and has moved to
live with his daughter and son-in-law, because of
difficulty in coping with living alone. You prescribe
fluoxetine, a selective serotonin reuptake inhibitor
(SSRI), but after 6 weeks of treatment, Roberto is still
depressed. What is the MOST LIKELY reason for
Roberto's failure to respond?
a) A different SSRI would have been effective
b) SSRIs are not the appropriate type of drug
for this patient
c) He has severe depression requiring specialist
management
d) He has psychotic depression
electroconvulsive therapy (ECT)

requiring

e) His bereavement and loss of independence


have not been dealt with

Answer: A
The next step is to confirm or dispel your suspicions of
domestic violence by directed but non-judgemental
questions about the domestic situation, in particular how
Cherie and her husband work out disagreements;
whether she feels safe at home, and so on. It is
preferable to question both parties if possible. Once a
diagnosis of domestic violence is made, it is important to
establish a supportive doctor-patient relationship and
formulate a safety plan with the victim, including
provision of information about abuse, the likelihood of
recurrence, access to shelters and support groups etc.
The option of informing the police and State authorities
should be discussed and appropriate action taken
according to the circumstances of the case. The prime
consideration in domestic disputes is the safety of the
victim and the children.

Question 40
Muriel is an 85 year old nursing home resident. The
nursing staff are concerned about her, as she has
vomited several times today and this afternoon
complains of abdominal pain. She is not clear about its
location, but it appears to be right-sided. She does not
have a fever. Which of the following statements is
CORRECT?
a) It is important to have a high index of
suspicion for gall bladder disease
b) Muriel has early gastroenteritis
c) Appendicitis is less common in elderly
patients, but the risk of perforation is

Answer: E

also low

In patients like Roberto, psychological reactions to


changed life conditions are likely to play an important
part in the causation of depression. He has suffered
bereavement and the loss of his independence, both of
which are likely to be significant factors. Hence ,
psychological approaches to management (e.g. cognitive
behavioural therapy) are more appropriate.

d) The most likely diagnosis is mesenteric


ischaemia
e) Muriel probably has diverticulitis, as 85% of
cases involve the ascending colon

Answer: A
Question 39
Cherie is a 38 year old married woman with two young
children. She consults you because of anxiety which she
attributes to 'the kids getting on my nerves.' You notice
that she has a black eye and bruising of her left
forearm, consistent with a defence injury. When asked
for an explanation, she says she walked into the door of
an open cupboard in the dark, when getting up to attend
to the younger child at night. You suspect domestic
violence. What is your next step in making a diagnosis?
a)
Non-judgemental
domestic conditions

questioning

about

b) Report suspicions to the police


c) Report suspicions to relevant State authority
d) Provide information about shelters and
support groups
e) Offer family counselling

Elderly patients may present very differently from their


younger counterparts and their abdominal pain is
frequently misdiagnosed. However, approximately 3550% of patients older than 65 have gallstones, and may
have associated biliary tract disease. The mortality rate
for elderly patients with cholecystitis is approximately
10%, so a high index of suspicion for gall bladder
disease is important, especially as symptoms and signs
are often not classical. Although relatively common, a
positive diagnosis of gastroenteritis should only be made
after other potential causes have been considered and
rejected.- Gastroenteritis in this age group should be a
'diagnosis of exclusion'. Appendicitis is less common in
the elderly, with only 10% of cases being in the over 60
age group. However, the risk of perforation is
approximately 50%. Mesenteric ischaemia is rare, but
has a high mortality. Vomiting and diarrhoea are often
present, but the pain in this condition is severe.
Diverticular disease is common in the elderly, but
diverticulitis- involving at least micro-perforation of the
colon, - occurs in 85% of cases in left(descending)
colon.

Block 6
Question 1

Answer: B

A 35 year old primigravida who's pregnancy had been


uncomplicated, presents at 36 weeks with moderate
oedema, BP155/95 mm Hg and ++ proteinuria. Which
ONE of the following measures would you recommend?

Pityriasis rosea is a common mild acute inflammatory


condition of the skin mainly affecting young adults. The
cause is unknown, but a virus is suspected. The oval
salmon-pink or copper-coloured eruptions with scaly
margins are confined to the trunk and upper limbs and
are often arranged along the skin creases (resulting an
an appearance of a Christmas tree). Itching is usually
mild. Lesions may be preceded by a 'herald patch' and
disappear spontaneously in 4-10 weeks. For information
on the other options see the references below.

a) Bed rest for the remainder of pregnancy


b) Induction of labour without further delay
c) Vigorous antihypertensive therapy
d) Review again at 37 weeks
e) Urgent referral to a specialist obstetrician

Answer: E
This is a high risk patient on two counts, viz. her age at
first pregnancy and the fact that she has pre-eclamptic
toxaemia. These require specialist management in the
latter stages of pregnancy and delivery.

Question 2
Which of the following conditions is FREQUENTLY
associated with prerenal acute renal failure?
a) Acute pyelonephritis
b) Severe dehydration
c) Rhabdomyolysis

Question 4
Bill is a 52 year old man who presents with nocturnal
heartburn that has left him exhausted due to lack of
sleep. Which of the following statements is CORRECT in
relation to his gastroesophageal reflux disease (GORD)?
a) Gastric acid hypersecretion is present in all
such patients
b) Disturbed oesophageal motility increases his
risk of oesophagitis
c) H2 receptor antagonists are the preferred
medical management
d) Maintaining an oesophageal pH of less than
3 is optimal
e) Bile salts are as deleterious
oesophageal lining as gastric acid

to

the

d) Prostatic hypertrophy
e) Renal artery obstruction

Answer: B
Prerenal acute renal failure is caused by underperfusion
of the kidneys, e.g. due to dehydration, haemorrhage or
shock. Acute pyelonephritis, rhabdomyolysis and renal
artery obstruction are causes of intrinsic acute renal
failure, i.e. the cause lies in the kidney. Prostatic
hypertrophy is an example of a postrenal cause of acute
renal failure, due to obstruction to the outflow of urine.

Question 3
This 22 year old girl presents with a rash on her trunk
which has been present for three days (see figure). It is
mildly itchy. What is the MOST LIKELY diagnosis?
a) Lichen planus
b) Pityriasis rosea
c) Lichen simplex
d) Atopic dermatitis
e) None of the above

Answer: B
The common denominator for virtually all episodes of
gastroesophageal reflux is the loss of the normal
gastroesophageal barrier to reflux. This is usually
secondary to a transient or permanent loss of lower
oesophageal sphincter resistance (eg. Gastric distension
with air or food, increased intragastric or intraabdominal
pressure, and delayed gastric emptying). Disturbed
oesophageal motility allows prolonged exposure of the
oesophageal lining to acidic fluids which is a major risk
factor in the development of oesophagitis in GORD.
Gastric acid hypersecretion may be present in some
cases. The preferred medical management when
significant symptoms are present is the use of a proton
pump inhibitor which will help to maintain the
oesophageal pH above 4. Bile salts reduce the resistance
of the oesophageal lining, but are not as deleterious as
gastric acid.

Question 5
Edith is a 75 year old woman who complains that her
eyesight isn't as good as it used to be. While testing,
you notice a lens opacity. Edith has well-controlled Type
2 diabetes and has a past history of polymyalgia
rheumatica 20 years ago. She has osteoarthritis of the
knees managed with intermittent NSAID therapy. Which

of the following is the MOST significant factor in the


development of her cataract?
a) Advancing age
b) Diabetes mellitus
c) Steroid therapy
d) Radiation
e) Trauma

Answer: A
Although all the options increase the chance of
cataracts, the most significant factor is advancing age.
About 50% of 65-74 year olds have lens opacities
increasing to 70% of those 75 years and over.

Question 6
Leigh is a 60 year old woman who has been
hypertensive for 5 years. Her BP now is 160/115 mm
Hg. Recently she has been getting increasingly short of
breath. Clinical assessment confirms congestive cardiac
failure. Which of the following drugs would be preferred
for management?
a) Propranolol

Answer: B
Difficulty swallowing (or dysphagia) is a functional
problem and a barium swallow is preferable to an
endoscopy in this instance. Observations on the barium
swallow may suggest oropharyngeal or cricopharyngeal
dysfunction (including misdirection of barium into the
trachea
or
nasopharynx),
prominence
of
the
cricopharyngeal muscle, a Zenker's diverticulum or a
narrow pharyngeo-oesophageal segment. Disordered
oesophageal motility or structural abnormalities such as
small diverticula, webs, and minimal extrinsic
impressions of the oesophagus may be recognised only
with motion-recording techniques.

Question 8
Francesco is a 52 year old man with a history of
rheumatic fever as a child. He presents with shortness
of breath on exertion without orthopnoea or discomfort
at rest. His echocardiogram demonstrates an isolated
mitral stenosis of moderate severity. A typical finding on
cardiac catheterisation would be:
a) Normal left atrial and normal left ventricular
diastolic pressures
b) High left atrial and normal left ventricular
diastolic pressure

b) Verapamil

c) High left atrial and high left ventricular


diastolic pressures

c) Diltiazem

d) Right ventricular hypertrophy

d) Lisinopril

e) Left ventricular hypertrophy

e) Felodipine
Answer: B
Answer: D
Linisopril is an angiotensin converting enzyme inhibitor
(ACEI). This is the treatment of choice, as it lowers
systemic vascular resistance and venous pressure and
reduces the levels of circulating catecholamines, thus
improving myocardial performance. It is important to
observe for first-dose hypotension. Calcium channel
blockers (e.g. verapamil, diltiazem) may have a
detrimental effect on left ventricular function in patients
with heart failure. Non-selective beta blockers such as
propranolol are not well tolerated in heart failure.
However third generation beta blockers such as
carvedilol as well as beta-1 selective agents metoprolol
and bisoprolol improve symptoms and exercise tolerance
as well as lowering the risk of progression of heart
failure and death. They are started once the ACEI dose
is stable and fluid status is optimal. The beta blocker
starts at a very low dose and is slowly titrated up.

Question 7
In which of the following situations would a barium
swallow be preferable to an endoscopy as a FIRST LINE
investigation?
a) Patient complains of coughing after meals
b) Patient complains of difficulty swallowing
c) Patient with nocturnal symptoms only
d) Patient with bloating after meals
e) Patient has water-brash

Mitral stenosis of moderate severity indicates narrowing


of the mitral valve orifice area from 5cm to 2cm
squared. This results in an increased left atrial pressure
and dilatation of the left atrium but a normal left
ventricular pressure. Eventually pulmonary vascular
pressures may rise with associated worsening of
symptoms and
then right ventricular hypertrophy may ensue.

Question 9
Jean, a 52 year old woman, presents with hot flushes
related to menopause. She has noted a recent loss of
libido. Which of the following statements concerning the
effects of menopause on libido is INCORRECT?
a) Sleep deprivation, secondary to hot flushes
and night sweats, can lead to depression
b) Vaginal dryness and painful intercourse due
to oestrogen deficiency are common
c) As the menopause approaches, erratic
periods or menorrhagia may impact on sexual
desire
d) Oestrogen deficiency heightens sensitivity to
touch stimuli, causing pain and discomfort
e) Reduced muscle tone of the pelvic floor can
affect orgasm

Answer: D

Answer : C

Clinical manifestations of oestrogen deficiency include:

Gastroesophageal reflux is caused by a transient or


permanent loss of lower oesophageal sphincter tone. As
a consequence, it more often occurs after meals.
Controlling acid secretion at all times with a proton
pump inhibitor is the medical treatment of choice. The
other factors may play a role but are not the most
important.

1. Variable length of menses - erratic, frequent


or prolonged - which can impact on sexual
desire
2. Hot flushes and night sweats, which can
cause chronic sleep deprivation and potentially
depression
3. Vaginal atrophy which can lead to vaginal
dryness,
dyspareunia
and
secondary
vaginismus due to fear of intercourse
4. Urethral/bladder atrophy and loss of pelvic
floor muscle tone, which can cause urethral
incontinence. Loss of pelvic floor muscle tone
can also reduce vaginal sensitivity and muscle
contractions involved in orgasm
5. Touch aversion or reduced sensitivity to
touch stimuli in genital tissues

Question 12
Which of the following describes an ependymoma?
a) A benign skin lesion of the face
b) A central nervous system tumour
c) A cystic structure in the upper eyelid
d) A patch of discoloured skin
e) A visible defect in the iris

Question 10

Answer : B

Which of the following statements is CORRECT? In


motor neurone disease there is:

Ependymomas are central nervous system tumours


affecting the brain and spinal cord, derived from the
single layer of cells (the ependyma) lining the ventricles
and spinal canal.

a) A long
exacerbations
b) Sensory
distribution

history
loss

of

following

remissions
a

and

dermatomal

c) Focal epilepsy
d) Sparing of lower motor neurons
e) Involvement of upper and lower motor
neurons

Answer : E
The commonest form of motor neurone disease is
amyotrophic lateral sclerosis (ALS), a sporadic condition
of
unknown
cause
characterised by relentless
progressive degeneration of upper and lower motor
neurones in the spinal cord, the somatic motor nuclei of
the cranial nerves and the motor cortex of the brain.
The sensory system is not involved. Remission in this
disease is unknown.

Question 13
A healthy six year old child without cyanosis or
dyspnoea on exercise is examined for migration to
Australia. His pulse is 84 per minute, B.P. 100/60, radial
pulse and jugular venous pressure normal and there is
no evidence of cardiomegaly. On auscultation in the 2nd
left intercostal space the 1st and 2nd heart sounds are
audible with fixed splitting of the 2nd heart sound and a
midsystolic pulmonary ejection murmur is heard. The
MOST likely diagnosis is:
a) Pulmonary stenosis
b) Atrial septal defect (ASD)
c) Innocent pulmonary ejection murmur
d) Ventricular septal defect (VSD)
e) Patent ductus arteriosus (PDA)

Question 11

Answer : B

Joan is a 46 year old woman who has a long past history


of bloating and water brash. She usually takes an
antacid after her meals. In your advice to Joan you

In an asymptomatic patient an ASD is often diagnosed


as a loud P2 with fixed splitting and an ejection murmur
heard in the pulmonary area due to increased blood flow
to the right heart. A VSD large enough to produce these
signs would be symptomatic and usually would cause
cardiomegaly. Innocent pulmonary ejection murmurs do
not cause fixed splitting of P2, and a PDA causes a
continuous murmur. In pulmonary stenosis P2 is often
soft or inaudible and the JVP is usually elevated.

would explain that the MOST important factor in


managing moderate and severe reflux is:
a) Avoidance of alcohol with meals
b) Avoidance of spicy food
c) 24-hour control of acid secretion
d) Limitation of the size of meals
e) Not lying down too soon after meals

Question 14

Answer: B

Female androgen insufficiency syndrome includes all of


the following EXCEPT:

A wet saline mount is 30-80% sensitive for Trichamonas


vaginalis in symptomatic women. A drop of vaginal
discharge is placed on a slide with 1-2 drops of normal
saline and examined under high power (x 400). The oval
or flagellated protozoan is seen among a large number
of white blood cells and epithelial cells. Its erratic,
twitching motility may be precipitated by warming the
slide, thus distinguishing it from the similarly sized
polymorphs. Culture is 95% sensitive especially in
asymptomatic men and women, but not widely available
and results take 7 to 10 days. Trichomonads are often
reported on Pap smears from the cervix with a
sensitivity of 60 to 70%. However a high false positive
rate means further testing is necessary to confirm the
diagnosis. Management involves treatment with
metronidazole or tinidazole 2G stat orally. Contacts
should also be treated and evaluated for other sexually
transmitted infections.

a) Persistent fatigue
b) Reduced body hair
c) Decreased libido
d) Blunted motivation
e) Dysphoria

Answer : B
Reduced body hair is not a feature of female androgen
insufficiency. The syndrome includes:

1. Diminished sense of well being, dysphoric mood


and/or blunted motivation
2. Persistent, unexplained fatigue

Question 17

3. Decreased libido, pleasure and sexual receptivity

Yee Leng, aged 40 years, is a new patient in your


practice. She tells you she has been experiencing
migraine headaches for a number of years. Which of the
following features would NOT be consistent with
migraine as a cause of her headaches?

4. Potential bone loss, decreased muscle strength,


changes in cognition/memory

a) Numbness or tingling in one arm and hand

Question 15
The SINGLE MOST important diagnostic sign of
developmental dysplasia of the hip in the neonatal
period is?
a) An unusually wide perineal region

c) Failure of the thighs to abduct readily


d) Failure of the thighs to externally rotate
e) A clunk on abduction of the flexed thigh

Answer: E
On examination of the newborn a clunk or jerk noticed
on abduction or adduction of the flexed thigh is
suggestive of developmental dysplasia of the hip
(previously called congenital dislocation of the hip).
Ultrasound examination gives useful information as to
the relationship of the femoral head to the acetabulum
and the existence of any acetabular dysplasia during the
first 3 months of life. If not diagnosed in the neonatal
period the older child presents with an abnormal gait, a
shortened leg on the affected side, asymmetrical
abductor creases, restricted hip abduction (especially in
flexion) and a positive trendelenberg's sign.

Question 16
trichomoniasis

is

c) Pain behind one eye radiating to the neck


d) Bilateral pain
e) Vertigo lasting about 30 minutes

b) Asymmetrical skin folds in the thighs

To optimise management,
effectively identified by:

b) A headache which lasted 10 days

MOST

a) Cytological examination of a cervical smear


b) Microscopic examination of a 'wet prep'
c) Stained smear of vaginal discharge
d) Culture of vaginal discharge
e) Histological examination of a punch biopsy of
vaginal mucosa

Answer: B
Migraine attacks can last from hours (most common) to
days, but never weeks. There are several different
clinical patterns of migraine. Attacks can vary from
intermittent headaches, indistinguishable from tension
headaches,
to
discrete
episodes
that
mimic
thromboembolic cerebral ischaemia. Symptoms may
include visual auras, nausea, vomiting, tingling,
generalized
headache,
vertigo
and
transient
hemiparesis.

Question 18
Betty, who is aged 29 years and nulliparous, comes to
you for a routine Pap smear. The cervix looks healthy
and there are no abnormalities on clinical examination.
A week later she returns for the pathology result which
is reported as: 'Abnormal cells are present, consistent
with a diagnosis of carcinoma in situ CIN 3.' What is the
implication of this result?
a) Betty has malignancy of the cervix and
requires radiotherapy followed by hysterectomy
and clearance of lymph nodes
b) Betty has a malignancy of the cervix and
hysterectomy is required
c) The smear should be repeated after Betty
has applied clindamycin vaginal cream for 7
days
d) Betty requires a cone biopsy
e) Betty requires a colposcopy

Answer: E

Question 21

Betty needs referral to a gynaecologist for colposcopy


and directed biopsy with definitive treatment if CIN 3 is
confirmed.

Bill, aged 45 years, has been experiencing headaches on


and off for 5 years. Over the past 2-3 weeks he feels
that he has had an almost continuous headache. Which
of the following features would make you suspect a
tension-type headache?

Question 19

a) He is unable to continue with daily work

Mary aged 29 years, has suffered from migraines for


three years. She is keen to reduce the frequency of
attacks and asks about trigger factors. Which of the
following advice would you NOT offer her?

b) The headache wakes him in the middle of


the night
c) It is very rarely bilateral

a) Use relaxation techniques to manage stress

d) It is usually bifrontal or bioccipital

b) Eliminate known dietary


Chocolate, red wine, MSG

e) He suffers associated nausea

triggers

eg.

c) Maintain a regular sleep pattern


d) Restrict time watching television to less than
2 hours a day
e)
Avoid
contraceptives

oestrogen-containing

oral

Answer: D
There is no known link between watching television and
migraine, except that some programs may be stressful.
The other options refer to recognised trigger factors.

Answer: D
Tension headaches are often bifrontal or bioccipital.
They are believed to be a result of tension within the
scalp muscles. These headaches are always innocent
and can be associated with tight band sensations,
pressure behind the eyes, and throbbing and bursting
sensations. Precipitating factors include worry, noise,
concentrated visual effort, fumes or depression. There
are no abnormal physical signs, other than tenderness
and tension in the nuchal and scalp muscles.

Question 22
Question 20
Which of
CORRECT?

following

statements

about

syphilis

is

George is a nineteen year old man who presents with


dysuria. On examination there is a thick yellow urethral
discharge. Which ONE of the following is CORRECT with
regard to gonococcal infection?

a) Syphilis is only acquired through sexual


contact

a) Urine PCR is now the gold standard for


diagnosing gonococcal urethritis

b) The typical primary lesion of syphilis is a


painful, hard ulcer on the genitalia

b) Spread to involve
epididymis is common

c) Definitive diagnosis is by culture of fluid from


the ulcer

c) The incubation period is usually 14 to 21


days

d) Signs of secondary syphilis


mucocutaneous lesions and non
lymphadenopathy

d) Ceftriaxone IM as a single dose is an


effective treatment

include
tender

e) Once treated there is no need for follow up

the

prostate

and

e) Pharyngeal gonorrhoea is diagnosed by


finding Gram-negative diplococci on a smear

Answer: D

Answer: D

Secondary syphilis is marked by mucocutaneous lesions


and non tender lymphadenopathy. Syphilis may be
acquired by skin to skin nonsexual personal contact and
transplacental infection. The primary lesion of syphilis is
a painless, hard chancre on the external genitalia. The
traditional, preferred testing method is dark-field
microscopy. Specimens are taken from active lesions
and examined directly. Serology should always be taken
to diagnose syphilis. These tests are complex and
require expert interpretation. Serology is also used to
monitor the effectiveness of treatment in syphilis.
Penicillin, given by injection, is a very effective
treatment. Repeated blood tests are necessary for at
least a year after treatment to monitor possible
treatment failure.

Routine treatment for gonococcal infection is either with


ceftriaxone 250 mg IM as a single dose or ciprofloxacin
500 mg orally as a single dose. The symptoms of
gonococcal infection usually appear within 2 to 10 days
of infection, but the incubation period is sometimes
much longer, and some men never develop symptoms.
In men, infection can spread to the prostate gland,
epididymis, and the testes, although this is not common.
Gram-stained smears are only sensitive and specific for
urethral smears in symptomatic men. Culture is still the
gold standard with >95% sensitivity and the ability to
test for antibiotic sensitivities. Since the organism dries
out quickly and is fairly fastidious, samples must reach
the laboratory within 12 hours. PCR testing is validated
for endocervical and first catch urine specimens only but
fails to allow subsequent antibiotic sensitivity testing

which is significant in light of emerging quinolone


resistance. It has the advantage of not requiring live
organisms, being less invasive as a screening tool and
enabling a check for chlamydia on the same sample.
PCR tests are 90-100% sensitive and 98-100% specific.

Question 23
Belinda, aged 44 years, presents complaining of heavy,
prolonged periods (menorrhagia) and severe period pain
(dysmenorrhoea) that has gradually become worse
during the past year. Her periods are still quite regular.
Which of the following possible causes is UNLIKELY?
a) Adenomyosis

Question 25
John is a 35 year old man who has had four recent
episodes of intense retro-orbital pain. You suspect
cluster headache. Which of the following features would
be LEAST consistent with this diagnosis?
a) He always vomits with the pain
b) He has ipsilateral nasal stuffiness
c) He notices a drooping of the eye on the
same side as the headache
d) Three of the episodes occurred within 2 days
e) Episodes seem to last from 10 minutes to 2
hours

b) Endometriosis
c) Uterine cancer
d) Fibromyoma
e) Ovarian failure

Answer: E
Ovarian failure presents as irregularity and scarcity of
menstruation,
rather
than
menorrhagia
and
dysmenorrhoea. Adenomysosis, endometriosis, uterine
cancer and fibromyoma are all possible causes of
menorrhagia and secondary dysmenorrhoea. Other
causes
include
uterine
polyps,
intra-uterine
contraceptive devices, pelvic inflammatory disease and
cervical stenosis.

Answer: A
Cluster headaches are recurrent bouts of excruciating
pain centred around one eye and lasting for minutes to
hours. The affected side of the face and nostril feel
congested. Commonly, there is a transient ipsilateral
Horner's Syndrome. Vomiting may be associated, but is
not always present.

Question 26
In the treatment of persistent tension-type headaches,
in which mild analgesics and relaxation techniques are
insufficient, which of the following medications would be
MOST suitable?
a) Propranolol 40mg at night

Question 24

b) Paroxetine 20mg in the morning

Martha is a 20 year old woman who presents with a


vaginal discharge, bilateral painful vesicles and ulcers on
her external genitalia. There is tender inguinal
lymphadenopathy. She also has systemic symptoms,
including headache, myalgia and a temperature of 38.5
degrees Celsius. Which of the following is CORRECT?

c) Amitriptyline 10-75mg at night


d) Diazepam 2-5mg at night
e) Carbamazepine 100-200mg twice a day

a) Antiviral agents eradicate the virus

Answer: C

b) Antiviral agents have no effect on recurrence

In patients in whom simpler treatments, such as


paracetamol, and relaxation techniques, have failed,
amitriptyline (10-75 mg) at night is indicated in the
treatment of persistent tension headaches. The other
medications listed are not indicated. Diazepam (short
term use) may be effective in middle-aged men, but
generally should be avoided because of risk of
dependence.

c) Antiviral agents do not hasten healing


d) Topical lignocaine is beneficial for the pain
e) The primary aim is to keep the lesions clean
and dry

Answer: E
This scenario is typical of primary genital herpes. The
primary aim is supportive treatment by keeping lesions
as clean and dry as possible while spontaneous healing
occurs. Analgesics by mouth are often useful,
particularly at night time. No treatment is available to
eradicate the virus, but antiviral agents reduce viral
shedding from lesions, hasten healing and reduce the
risk of recurrence while being administered. The
recommended regimen is valaciclovir 500mg twice a day
for 5 days. Topical lignocaine and zinc creams should
not be used.

Question 27
Which of
trismus?

the

following

describes

the

a) A grimace
b) Tonic spasms of the jaw muscle
c) A triad of symptoms
d) Paralysis of the tongue
e) A facial tic

meaning

of

Answer: B

Question 30

Trismus is the prolonged tonic spasm of the muscles of


the jaw. It is an involuntary early sign of tetanus.
Prolonged voluntary clenching of the jaw with teeth
grinding is called bruxism, and is associated with
emotional stress.

Which of the following statements about


management of chronic pain is INCORRECT?

Question 28
James is a 32 year old man who presents two days after
experiencing his first epileptic seizure. He was well until
the seizure which occurred without warning. Since then
he feels that his left leg has become weaker. James had
meningitis as a child, and two years ago sustained a
fractured skull in a motor vehicle accident. There is a
family history of epilepsy (brother and an uncle have
epilepsy).
Which
of
the
following
groups
of
investigations is MOST likely to help in establishing the
cause of James' epilepsy?
a) Metabolic
magnesium

screen

EEG

plasma

b) Metabolic screen + CT scan + plasma


magnesium
c) Metabolic screen + EEG + CT scan

the

a) Depression is often an aggravating factor


b) Antidepressants are only indicated in clinical
depression
c) Psychological assessment is important to a
good outcome
d) A history of nerve damage may indicate
neuropathic pain
e) Opioid medication may be required for some
patients

Answer: B
Antidepressant medication may be helpful in patients
with chronic pain even if they do not have depressive
symptoms. These include patients with diabetic
neuropathy, tension headache, rheumatoid arthritis and
chronic low back pain.

Question 31

d) Metabolic screen + drug screen + EEG

In a child with chickenpox which of the following drugs


is MOST LIKELY to cause Reye's syndrome?

e) Metabolic screen + drug screen + CT scan

a) Paracetamol
Answer: C

b) Aspirin

A metabolic screen will detect biochemical causes, such


as electrolyte abnormalities, acid base disturbances,
hypocalcaemia etc. The EEG will help establish the
diagnosis of epilepsy, classify the type and identify the
particular epilepsy syndrome. A CT scan will identify any
underlying structural abnormality, e.g. brain tumour. A
drug screen is only necessary if there is reason to
suspect drug abuse. Magnesium is one of the
components of the metabolic screen.

c) Codeine

Question 29
Which of the following statements about brain tumours
is CORRECT?
a) Frontal lobe tumours present late
b) Contralateral limb weakness or sensory loss
are due to frontal tumours
c) Disturbances of speech
nondominant temporal tumours

are

due

to

d) Impaired insight is due to midbrain tumours


e) Mood disturbances usually relate to tumours
in the parietal lobe

d) Penicillin
e) Prednisolone

Answer: B
Reyes Syndrome involves acute encephalopathy and
fatty infiltration of the liver following an acute viral
infection, including influenza and varicella. Foreign
chemicals, especially salicylates (including aspirin), and
intrinsic metabolic defects have also been implicated.
The use of salicylates (eg aspirin) during an acute viral
illness such as chicken pox (varicella) increases the risk
of Reyes syndrome by as much as 35-fold.

Question 32
Turner syndrome includes all of the following features
EXCEPT:
a) Ovarian agenesis
b) Webbing of the neck
c) A predominantly "X0" sex chromosome
pattern

Answer: A
Often frontal lobe tumours remain asymptomatic or
undiagnosed until they are very large, possibly because
local dysfunction of this brain region causes subtle and
non-specific neurological or behavioural deficits.

d) The possession of 45 chromosomes


e) A chromatin-positive buccal smear

Answer: E

Question 35

Patients with Turner's syndrome characteristically have


a 45 XO chromosome pattern, and streak gonads
(ovarian agenesis). Characteristic features include short
stature, webbing of the neck, cubitus valgus, widely
spaced nipples and puffy hands and feet. The range of
intelligence is normal. A chromatin-positive buccal
smear requires two X chromosomes. One X chromosome
is randomly inactivated early in embryogenesis.
Therefore, the buccal smear in Turner's syndrome does
not have a Barr body (ie it is chromatin negative).

Mandy, a 4 year old girl, is due to accompany her


parents on a flight to England in two months time. Her
mother is worried about the effect of air travel on
Mandy's ears. Which of the following will NOT increase
the likelihood of ear pain during the flight?
a) A recent cold
b) Nasal congestion
c) Hay fever
d) Recent otitis media

Question 33
Sarah is an 8 week old girl who has persistent
regurgitation. Which of the following features suggests
the need for further investigation?
a) Sarah is underweight for her age
b) Sarah regurgitates after every meal
c) Sarah has episodes of uncontrollable crying
d) Sarah arches her back on occasion and stops
feeding
e) Sarah was born 2 weeks premature

e) Perforation of the ear drum

Answer: E
Normal function of the middle ear and tympanic
membrane is maintained by a patent eustachian tube
which keeps the middle ear and external canal pressures
equal.
Blockage of the eustachian tube, e.g. with secretions as
in the common cold, results in pressure imbalances
between the middle and external ear. During air travel,
the resulting barotrauma may cause pain, deafness,
vertigo, and tinnitus. All the options listed, except (e),
can cause blockage of the eustachian tube.

Answer: A

Question 36

Regurgitation
after
every
meal
suggests
gastroesophageal reflux, but of itself is not a worrying
feature. Underweight for age, however, suggests failure
to thrive and needs investigation. Unsettled and irritable
behaviour is very common in the first 6-12 weeks of life.
In isolation it is not a concern.

Sue, a three year old girl, presents with shortness of


breath and wheeze that have developed over the last
two days. Examination reveals an afebrile, moderately
tachypnoeic child with widespread scattered wheezes on
auscultation. You decide to administer a bronchodilator
(salbutamol). The recommended method of delivery of
salbutamol for Sue is:

Question 34
Which of the following is suggested by a history of
transient episodes of vertigo, slurred speech, diplopia,
and paraesthesia in a man aged 65 years?
a) Basilar artery insufficiency
b) Anterior communicating artery aneurysm

a) Syrup
b) Nebuliser
c) Breath activated inhaler
d) Metered dose inhaler with a spacer
e) Metered dose inhaler with a spacer and face
mask

c) Hypertensive encephalopathy
d) Pseudobulbar palsy

Answer: E

e) Occlusion of the middle cerebral artery

For the treatment of acute asthma in a child <6 years of


age,
the
recommended mode of
delivery
of
bronchodilator (salbutamol) is via a metered dose
inhaler and small volume spacer with face mask. For
those >6 years of age, a large volume spacer may be
substituted. Salbutamol administered via these routes
has been shown to be equally effective to nebulised
salbutamol. Six puffs of salbutamol via MDI and spacer
is the equivalent of a 2.5 mg nebule, while 12 puffs
equals a 5mg nebule.

Answer: A
Episodes of vertigo, diplopia and paraesthesia indicate
alteration of brainstem function caused by basilar artery
insufficiency. Middle cerebral occlusions usually presents
with
contralateral
hemiplegia
and
homonymous
hemianopia. Slurred speech or aphasia may occur in
both
conditions.
Anterior
communicating
artery
aneurysms are likely to result in emotional lability.
Hypertensive encephalopathy comprises headache,
confusion, stupor or convulsions. Pseudobulbar palsy
presents with dysarthria, dysphagia and emotional
lability.

Question 37

Question 39

In acute iritis (anterior uveitis), the pupil is:

The MOST common cause of persistent cough in children


is:

a) Eccentric, constricted and reacts briskly to


light

a) Chronic post viral cough

b) Concentric, dilated and reacts briskly to light

b) Asthma

c) Eccentric, dilated and reacts sluggishly to


light

c) Post-pertussis

d) Concentric, constricted and reacts sluggishly


to light
e) Concentric, dilated and reacts sluggishly to
light

Answer: D
Anterior uveitis typically presents with a unilateral
painful red eye, blurred vision, photophobia and tearing.
There is a perilimbal flush and pupillary miosis with a
sluggish
response
and
increased
pain
during
accommodation to light. This is due to spasm of the
ciliary body and iris muscles. Slit lamp examination will
demonstrate keratic precipitates on the cornea. Urgent
treatment is needed to prevent complications.

Question 38
Oliver, an 8 month old boy, presents with a history of
anorexia and recurrent upper respiratory tract
infections. Physical examination reveals pallor and
splenomegaly. His blood picture shows a hypochromic,
microcytic anaemia with numerous target cells and
nucleated red cells. What is the MOST LIKELY diagnosis?

d) Gastro-oesophageal reflux
e) Passive smoking

Answer: A
The commonest cause of persistent cough (particularly
with a prominent nocturnal component) in children is
post viral. It can also be associated with postnasal drip.
However, care must be taken to exclude asthma which
is a fairly common cause of persistent cough in children.
Other less common conditions to be considered include
gastrointestinal reflux, passive smoking and post
pertussis.

Question 40
Trevor is a 2.4 kg male infant with Apgars 9:10 after a
normal vaginal delivery. He becomes jaundiced at 12
hours of age. Which of the following conditions would be
the MOST LIKELY cause of the jaundice?
a) Gram negative septicaemia
b) Jaundice of prematurity
c) Biliary atresia

a) Thalassaemia major

d) Physiological jaundice

b) Iron deficiency anaemia

e) Rh incompatibility

c)
Glucose-6-phosphate
deficiency

dehydrogenase

d) Sideroblastic anaemia
e) Sickle cell anaemia

Answer: A
Thalassaemia major (homozygous beta-thalassaemia) is
a severe disease which presents during the first year of
life with: failure to thrive, intermittent infection, severe
hypochromic, microcytic anaemia and signs of
extramedullary haemopoiesis (hepatosplenomegaly and
bone expansion). It does not present at birth because
the production of foetal haemoglobin is not affected.

Answer: E
Jaundice appearing in the first 24 hours of life is most
commonly due to haemolytic disease of the newborn
due to incompatibility to Rh, ABO or one of the other
rare antigens. Other causes of early jaundice include
transplacental infections such as CMV, toxoplasmosis
and rubella. Jaundice of prematurity, physiological
jaundice and septicaemia present most commonly
between days 2-5. Biliary atresia presents with jaundice
after the first week of life.

Block 7
Question 1

Answer: E

Mrs B is 55 years old and presents because of painful


mouth ulcers (see figure). You diagnose aphthous
ulcers. She is otherwise well. Which of the following
statements about aphthous ulcers is INCORRECT?

Intracerebral haemorrhages tend to be dramatic and


accompanied by a severe headache. However, there
really is no clinical way of reliably distinguishing
between
an
intracerebral
haemorrhage
and
a
thromboembolic infarction, as both produce a sudden
focal deficit.

a) The cause is rarely found


b) There is often a family history
c) The ulcers often begin in childhood

Question 4

d) They are often an indicator of Behcet's


syndrome

Bel is 20 years old and has had a Type I allergic reaction


to a bee sting. She states that there is a family history
of bee sting allergy. Which of the following is NOT useful
advice for Bel?

e) Aphthous ulcers may present like herpetic


vesicles

a) Do not drink out of an open soft drink can


that has been left outdoors
Answer: D
Behcet's syndrome is a systemic condition with
recurrent multiple apthous ulcers AND two of the
following: - recurrent genital ulceration, eye lesions,
skin lesions or positive pathergy test (non-specific
inflammatory skin reaction following intradermal saline
injection). Recurrent apthous ulceration in isolation is
not associated with Behcet's syndrome. The other
statements concerning apthous ulcers are all true.

b) Have a supply of antihistamines on hand


c) Insect repellents are useful to prevent bee
stings
d) Do not walk barefoot around swimming
pools
e) Always carry an adrenalin 1:1000 injection,
e.g. EpiPen, and know how to use it

Question 2

Answer: C

The drug of choice for treatment of erysipelas is:

Insect repellents have not been shown to be useful in


preventing bites from stinging insects. Anyone with a
known allergy to stinging insects should know how to
administer adrenalin 1:1000 subcutaneously and have it
with them at all times. EPIpen is a commercial
preparation which is supplied with an auto-injection
device. Avoiding behaviours likely to lead to a sting such as those mentioned in the options and avoiding
colourful clothes and perfumes which attract insects - is
also important.

a) Penicillin
b) Methicillin
c) Ampicillin
d) Tetracycline
e) None of the above

Answer: A
Erysipelas is a form of superficial cellulitis of the skin
with lymphatic involvement. It is almost always caused
by Strep pyogenes and therefore the treatment of
choice is penicillin (erythromycin or cephalexin can be
used in penicillin allergic patients).

Question 5
John is a 30 year old professional athlete who suddenly
develops persistent dull upper left chest pain which is
not related to exertion. There are no associated cardiac
or respiratory features except for a mild restriction in
breathing. John is afebrile and otherwise well. Which of
the following diagnoses is LEAST likely?

Question 3

a) Spontaneous pneumothorax

Which of the following criteria enable a clear distinction


to be made between haemorrhage and thrombosis in a
patient with a cerebrovascular accident?

b) Functional chest pain

a) The progress of the clinical features


b) The degree of loss of consciousness

c) Costo-chondral syndrome
d) Muscle strain
e) Pleurodynia ( Bornholm's disease)

c) The abruptness of onset


d) The presence or absence of headache
e) None of the above

Answer: E
Bornholm's disease is due to an infection by Coxackie B
virus. It is often associated with an acute upper

respiratory tract infection with fever, pleuritic chest pain


and upper abdominal pain. These pains can be severe
and associated with tachypnoea. A spontaneous
pneumothorax, functional chest pain, costochondritis or
acute muscular strain would be more likely in this
patient.

Question 6
Which of the following statements concerning bee sting
allergy is CORRECT?
a) If a patient has had a life-threatening
episode in the past, he or she is at risk of a
future one
b) A mild reaction in the past puts a patient at
risk of a future life threatening episode
c) Wasps only sting once
d) A bee sting is smaller in volume than a wasp
sting
e) A raised serum IgG persists for years in
those at risk of anaphylaxis

Answer: A
A previous life-threatening reaction is an indicator that a
similar episode may occur in the future. Reactions don't
necessarily escalate. Wasps produce a smaller volume of
sting, but are capable of stinging multiple times. Serum
IgE levels remain elevated in those at risk of an
anaphylactic reaction. IgG appears protective.

Question 7
The nephrotic syndrome can be caused by all of the
following EXCEPT:

a) The causative organism is more likely to be


bacterial than viral
b) There is a higher risk of complications
c) This may be the first presentation of
inflammatory bowel disease
d) Stool microscopy and culture to identify the
organism is worthwhile
e) All of the above

Answer: E
The presence of blood in the stool of a patient with a
constitutional illness is strongly suggestive of a bacterial
infection. Identification of the causal bacteria is usually
desirable for clinical and public health reasons. Most of
the conditions are reportable to the relevant state health
department. In some cases fulminant infection can occur
and treatment based on antibiotic sensitivities of the
organisms will be required. Many of these bacteria
(Yersinia, Campylobacter, Shigella etc) can lead to a
systemic illness with polyarthropathy.

Question 9
Abdul is a 58 year old man who presents with transient
episodes of vertigo, slurred speech, diplopia, and
paraesthesia. Which of the following is the MOST likely
diagnosis?
a) Basilar artery insufficiency
b) Anterior communicating artery aneurysm
c) Hypertensive encephalopathy
d) Pseudobulbar palsy
e) Occlusion of the middle cerebral artery

a) Minimal change disease (lipoid nephrosis)


b) Amyloidosis

Answer: A

c) Membranous glomerulonephropathy

Transient ischaemic attacks involving the posterior brain


circulation, i.e. the basilar artery, are characterised by
diplopia, vertigo, vomiting, dysarthria, ataxia and
hemisensory loss.

d) Renal vein thrombosis


e) Diabetic nephropathy

Answer: D
Renal vein thrombosis is more likely to be a
complication of nephrotic syndrome rather than a cause.
In nephrotic patients the blood is more coagulable than
normal and the circulation may be sluggish owing to
hypovolaemia, both of which are likely to induce
thrombosis. The other options, together with focal and
segmental glomerulosclerosis, are responsible for 90%
of cases of nephrotic syndrome. They are all conditions
which disturb the structure or function of the glomerular
basement membrane.

Question 8
Bill, aged 35 years, has been unwell for 2 days with
severe abdominal cramps and diarrhoea. He visited
friends living on a farm during the previous week. He
stated that on inspection of his stool there appeared to
be blood in it. What is the significance of blood in the
stool?

Question 10
Andrew, aged 60 years, complains of traces of blood in
his stool for the past few weeks. Which of the following
is the MOST likely cause?
a) Diverticulosis
b) Cancer of the rectum
c) Haemorrhoids
d) Angiodysplasia
e) Colitis

Answer: C
Haemorrhoids and/or anal fissures are the commonest
and therefore most likely causes of traces of fresh blood
in the stool. These are easily identified by physical
examination. However, there are a number of other
more serious pathologies which must be excluded,

including colorectal cancer diverticulosis, angiodysplasia


and colitic conditions. These can be diagnosed by a
combination of rectal examination, sigmoidoscopy and
colonoscopy plus biopsy.

d) Pregnancy
e) Chronic hepatitis

Answer: B
Question 11
Cyanosis is LEAST likely to occur in:
a) Ebstein's anomaly (anomalous attachment of
tricuspid valve)
b) Tetralogy of Fallot
c) Tricuspid atresia

The ESR is a non-specific indicator of inflammatory and


neoplastic disease. The ESR increases with age and is
raised in pregnancy, the puerperium and in anaemia. It
is increased in acute and chronic inflammatory disease
and neoplastic disease. A low ESR (<1mm/hour) may be
seen in polycythaemia rubra vera and sickle cell disease.
The C-reactive protein is a more sensitive early indicator
of an acute phase response.

d) Ventricular septal defect


e) Transposition of the great vessels

Question 14
Which of the following statements about weight loss is
CORRECT?

Answer: D
Cyanotic heart disease occurs when the systemic venous
return is directed backward into the systemic circulation
without transitting the pulmonary vascular bed, i.e.
instead of pure left to right shunting, right to left or bidirectional shunting occurs.
Patients with ventricular septal defects do not usually
present with cyanosis. However, if there is a particularly
large defect, pulmonary vascular damage can occur
leading to pulmonary hypertension and right to left
shunting with cyanosis (Eisenmenger's syndrome). The
other options listed are all associated with right to left
shunting and cyanosis.

Question 12
Which
of
the
following
pathogens
causing
gastrointestinal infection is NOT associated with bloody
diarrhoea?
a) Listeria monocytogenes
b) Campylobacter spp
c) Yersinia enterocolitica
d) Salmonella serovars
e) Shigella

a) Dieting reduces the basal metabolic rate


(BMR)
b) Dieting increases the BMR
c) Exercise decreases appetite
d) Inactivity increases appetite
e) None of the above

Answer: A
Changes to a person's stable weight, either an increase
by forced feeding, or a decrease by food restriction
(dieting), induce compensatory physiological responses
that resist these changes. Thus with low calorie regimes
the BMR decreases to limit energy expenditure and
appetite increases, factors which complicate the process
of weight loss for the dieter. Exercise generally
increases appetite in response to the fall in blood
glucose caused by exercise.

Question 15
Which of the following is the MOST COMMON
pathological condition associated with aortic aneurysms?
a) Atherosclerosis
b) Syphilitic aortitis

Answer: A
Listeria monocytogenes usually only causes illness in
immunocompromised patients and pregnant women. L.
monocytogenes is quite unlike most food-borne
pathogens in that it does not cause gastrointestinal
symptoms but causes invasive infections like meningitis,
septicaemia, and chorioamniitis leading to stillbirth.
Gastrointestinal infections by all the other pathogens
listed result in bloody diarrhoea to varying degrees.

Question 13
The erythrocyte sedimentation rate (ESR) is raised in all
of the following EXCEPT:
a) Multiple myeloma
b) Polycythaemia rubra vera
c) Macrocytic anaemia

c) Trauma
d) Rheumatic aortitis
e) Cystic medial necrosis

Answer: A
Atherosclerosis is the most common pathological
condition associated with aortic aneurysms. The other
options are also associated with aortic aneurysms but
are less common.

Question 16
Bob has had moderately raised plasma cholesterol and
triglyceride concentrations for 6 months and you have
advised him to start on medication since dietary
measures, including abstaining from alcohol, have had

minimal effect. Investigations have shown that he has a


raised plasma very low density lipoprotein (VLDL) level,
i.e. he has type IV hyperlipoproteinaemia. Which of the
following medications would you prescribe?
a) Cholestyramine
b) Gemfibrozil

available for absorption. Zinc deficiency occurs in a


variety of conditions (including all the other options
listed above) but not vegetarianism. Vegetarians obtain
adequate amounts of zinc from legumes, nuts and
cereals.

Question 20

c) Simvastatin

Non haem iron absorption is promoted by all of the


following EXCEPT:

d) Atorvastatin
e) Fish oil

a) A high phosphate content of the diet


b) Low pH in the stomach

Answer: B

c) Vitamin C taken with a meal

Gemfibrozil is a fibrate which is the class of drugs


recommended
for
treatment
of
type
IV
hyperlipoproteinaemia. It stimulates lipoprotein lipase
activity and thereby strips triglycerides from VLDL,
causing VLDL degradation and lowering of VLDL levels,
with consequent lowering of plasma cholesterol and
triglyceride
levels.
The
mode
of
action
and
recommended indications for use of the other options
are discussed in the references below.

d) Iron deficiency

Question 17
In which of the following conditions does angina pectoris
occur in the absence of coronary artery disease:

e) Erythroid hyperplasia

Answer: A
A high phosphate content of the diet inhibits iron
absorption by forming insoluble iron phosphates. All the
other factors mentioned promote iron absorption. Acid
and vitamin C keep iron in the ferrous form. Iron
deficiency and erythroid hyperplasia influence iron
absorption through unknown mechanisms via the gut.

a) Mitral stenosis

Question 21

b) Mitral insufficiency

Paul is a 45 year old man


infarction. He develops a
beats/minute with frequent
The IMMEDIATE treatment of

c) Coarctation of the aorta


d) Aortic stenosis

with an acute myocardial


sinus bradycardia of 38
ventricular extrasystoles.
choice is:

a) Intravenous propranolol

e) Aortic insufficiency

b) Immediate direct current cardioversion


c) Intravenous morphine

Answer: D
In severe aortic stenosis when the aortic orifice is
reduced to one-third or less of its normal size, angina
pectoris may occur because of insufficient perfusion of
the coronary arteries. This is usually associated with
fainting and dyspnoea.

Question 18
Whole body zinc stores may be depleted in all of the
following EXCEPT:
a) Coeliac disease

d) Intravenous atropine sulphate


e) Intravenous lignocaine

Answer: D
Atropine should be the initial agent at doses of 0.5mg IV
every 5 minutes until the desired response is achieved
by removing vagal inhibition Transcutaneous cardiac
pacing is indicated if the above is not effective
(bradycardia remains <40/min), with internal pacing
being the definitive treatment for progressive or
persistent bradycardias.

b) Diabetes mellitus
c) Alcoholism

Question 22

d) Vegetarianism
e)
Acquired
(AIDS)

immunodeficiency

syndrome

All of the following features are consistent


haemolytic uraemic syndrome (HUS) EXCEPT:
a) Bloody diarrhoea
b) Thrombocytopaenia

Answer: D

c) High fever with rigor

Zinc is an essential component of many enzyme systems


including carbonic anhydrase, alcohol dehydrogenase
and alkaline phosphatase. The best dietary sources are
meat, shellfish and legumes; zinc in grains is less

d) Neurological abnormalities
e) Acute renal failure

with

Answer: C

Question 25

Haemolytic uraemic syndrome is most common in


children following a gastrointestinal infection with a toxic
strain of E.coli. Bloody diarrhoea is the usual presenting
feature followed by haemolysis and renal failure.
Microvascular
damage
results
in
profound
thrombocytopaenia and may involve the central nervous
system. If fever is present it is usually low-grade. Early
supportive treatment allows a full recovery in 70% of
children but there is a 3-5% mortality and 15-30% will
have chronic renal failure.

Tom is a 30 year old agricultural worker who works with


organophosphates. He presents with acute pain in the
right iliac fossa. You suspect acute appendicitis but are
concerned about the risk of a laparotomy BECAUSE:

Question 23
Which of the following triads of clinical features is found
in beri beri?

a) There is a risk of erroneous diagnosis


b) There is a tendency to prolonged bleeding
post-operatively
c) There is an increased hazard in general
anaesthesia
d) There is a likelihood of delayed healing of
the wound
e) There is a possibility of mental disorder after
operation

a) Cardiac failure, glossitis, dermatitis


b) Cardiac failure, neuropathy, dermatitis
c) Glossitis, dermatitis, memory loss
d) Glossitis, neuropathy, memory loss
e) Cardiac failure, neuropathy, memory loss

Answer: E
Beri beri is due to vitamin B1 (thiamin) deficiency. It is
now mainly confined to the poorest areas of SE Asia.
Neuropathy presents as stiffness, numbness and
weakness of the legs and loss of ankle reflexes,
progressing to polyneuropathy involving the trunk and
arms. Thiamin deficiency impairs cardiac energy
metabolism leading to cardiac failure and oedema.
Chronic thiamine deficiency results in Wernicke's
encephalopathy. Pellagra due to niacin deficiency results
in glossitis, diarrhoea, dermatitis, and dementia.

Question 24
Sheila is a 19 year old woman who presents with
ascites, high venous pressure and a small quiet heart.
Which of the following surgical procedures would be
MOST likely to relieve her problem?
a) Mitral commissurotomy
b) Closure of the foramen ovale
c) Ligation of a patent ductus arteriosus
d) Correction of coarctation of aorta
e) Pericardiectomy

Answer: E
Sheila has constrictive pericarditis in which the
pericardial cavity is obliterated and dense scar tissue
encases and constricts the heart. Diastolic filling of the
ventricles is limited resulting in a decrease in cardiac
output. The right ventricular diastolic pressure is
increased, leading to venous hypertension which in turn
produces hepatomegaly, ascites and peripheral oedema.
Prompt pericardiectomy is the treatment of choice. This
corrects the hemodynamic abnormalities and patients
improve rapidly with a massive diuresis.

Answer: C
Organophosphates
irreversibly
inhibit
acetylcholinesterase
and
cause
accumulation
of
acetylcholine at muscarinic and nicotinic receptors. In
general anaesthesia, muscle relaxant drugs like
prostigmine cause reversible blockade of cholinesterase.
Should a patient have absorbed subclinical doses of
organophosphates (e.g. in agricultural work with
pesticides), there is increased risk of excessive
neuromuscular blockade with use of muscle-relaxants,
and the patient may show signs of toxicity, or may
continue to be paralysed beyond the duration of the
anaesthetic agent given.

Question 26
Pamela is a 55 year old woman who comes to see you
for dietary advice. She is an estimated 10kg above her
ideal body weight and is keen to reduce this before the
summer. Which of the following strategies would NOT be
appropriate in this situation?
a) Advise Pamela against the use of appetite
suppressant medication
b) Encourage daily exercise of at least 30
minutes with heart rate to 70% of maximum
for her age
c) Assist Pamela to redefine her goals in terms
of
non-weight
targets
such
as
waist
measurement and improved stamina
d) Recommend an 8 week course of a very low
calorie diet (400-600kcal/day) to get started
e) Warn Pamela that a diet very low in
carbohydrate is less likely to result in sustained
weight loss

Answer: D
Very low calorie diets are most appropriate when body
weight is >130% of ideal. They must be supervised by a
medical officer due to the potential for electrolyte
imbalance. All the other options are appropriate in this
situation. Behavioural modification that involves simple
and sustainable changes in a person's diet, exercise and
lifestyle routine is likely to be the most effective way to
achieve and maintain an ideal body weight.

Question 27
The FIRST sign of salicylate poisoning in children is
usually:
a) Delirium

reassurance is all that is needed. However, if the ectopic


beats provoke more significant arrhythmias, Betablockade may be effective. Charles would be given
general advice regarding quitting smoking and
minimising alcohol, caffeine and other stimulants such
as cough medicines.

b) Coma
c) Hyperventilation
d) Hyperpyrexia
e) Convulsions

Answer: C
Aspirin has a two-fold toxic effect. First, it inhibits
oxidative phosphorylation leading to a metabolic
acidosis. The increased hydrogen ion concentration of
the extracellular fluid stimulates the respiratory centre
of the brain to cause hyperventilation. This is the
primary effect in children. Second, aspirin directly
stimulates
the
respiratory
centre
to
cause
hyperventilation leading to a respiratory alkalosis. This
phenomenon is seen mainly in adults.

Question 30
James is a 50 year old woodcutter whom you treated for
a Colles' fracture two days ago. His wife telephones
asking you to prescribe a sleeping tablet for him
because James is having difficulty in sleeping. Which of
the following is the MOST appropriate next step?
a) Suggest she gives James a nightcap of
whisky
b) Prescribe a short-acting benzodiazepine
c) Describe some relaxation exercises
James to do immediately before bedtime

for

d) Prescribe an analgesic such as paracetamol


and codeine combination
e) Ask James to come to the surgery so you
can check the plaster

Question 28
Chronic inorganic lead poisoning is associated with all of
the following laboratory features EXCEPT:
a) Basophilic stippling of red cells
b) Increased red cell protoporphyrin level
c) Normochromic normocytic anaemia
d) Uraemia
e) Aminoaciduria

Answer: C
The anaemia of chronic lead poisoning is usually
hypochromic microcytic. The other options are all
features of lead poisoning, due to toxic effects on
erythropoieses and on the kidney.

Question 29
Charles is a 48 year old businessman who presents for a
general check-up and mentions that he is experiencing
occasional fluttering sensations in his chest. A routine
electrocardiograph (see figure) is taken.
Your first line of management should be:

Answer: E
The prudent action is to ask James to come to the
surgery to check the plaster and the condition of his
hand as soon as possible, since it may be that his
problem is related to swelling around the fracture site
with increased pressure inside the plaster cast. This can
lead to an adverse outcome unless the pressure is
relieved soon. Once this is ruled out or dealt with, the
next issue to be addressed is adequate pain relief. In
most cases, once adequate pain relief is ensured,
insomnia will no longer be a problem. If insomnia does
persist, a short-acting benzodiazepine may be
prescribed for no more than 7-10 days to avoid
development of dependency. An alcoholic nightcap is not
a good idea, as it usually causes the patient to wake
during the night and have difficulty getting back to
sleep.

Question 31
Which
of
the
CONTRAINDICATION
attenuated vaccine?

following
situations
to immunisation with

is
a
a live

a) Pregnancy
b) Breastfeeding

a) Reassurance

c) Mild acute febrile illness

b) Referral for cardioversion

d) Current antibiotic therapy

c) Begin a trial of verapamil

e) Immunodeficiency in a household contact

d) Commence beta-blockade
e) Give lignocaine stat

Answer: A
The ECG shows Charles has premature atrial ectopic
beats. Often these are asymptomatic. They may,
however, be sensed as an irregularity or heaviness of
the heart beat. Treatment is not normally required;

Answer: A
Pregnancy is a valid contraindication to immunisation
with a live attenuated vaccine eg oral polio infection.
Exposure to HIV, other immunodeficiency states and
immunosuppressant
treatments
are
also
contraindications. Diarrhoea, minor acute illnesses,
antibiotic therapy and breast feeding are not valid
contraindications.

Question 32

Answer: A

The interossei muscles of the hand are supplied by:

An indwelling catheter allows monitoring of fluid status


as well as allowing urinary drainage. IV fluid therapy is
not urgent due to the potential to exacerbate brain
swelling, in the acute phase. Anticoagulant therapy is of
no value in treating a fully developed and completed
CVA. It may be used in transient ischaemic attacks, in a
developing progressive thrombosis, or in prevention of
DVT in a bedridden patient. The other measures of
physiotherapy and nursing care should follow.

a) The radial nerve


b) The median nerve
c) The ulnar nerve
d) All of the above
e) None of the above

Question 35

Answer: C
The ulnar nerve (c8- T1) supplies the adductors and
abductors of the fingers, the adductor of the thumb, the
medial two lumbricals and the muscles of the
hypothenar eminence.

Which of the following statements about simple febrile


convulsions is CORRECT?
a) It usually occurs between 6-8 years of age
b) Prognosis depends on the
illness

precipitating

Question 33

c) The risk of developing epilepsy is 10%

Brenda brings Jake, her 2 month old son for his first
Triple antigen injection. Which of the following
statements is CORRECT in relation to the acellular
pertussis vaccine?

d) The convulsions last less than 15 min


e) Investigation with lumbar puncture and CT is
essential

a) It provokes a stronger immune reaction


b) The costs are the same as whole cell vaccine

Answer: D

c) It causes a lower incidence of fever, crying


and irritability

Simple febrile convulsions last less than 15 minutes.


They usually occur between 3 months and 5 years, with
most occurring between 17 and 23 months of age. There
is no difference in IQ at age 7 years between children
who have had a febrile convulsion and their seizure- free
siblings. The risk of developing epilepsy following a
simple febrile convulsion is 0.9% at age 7 years.

d) It works against the bacteria rather than the


toxin
e)
It
provides
Bronchiseptica

protection

against

P.

Question 36

Answer: C
Acellular pertussis is part of DTPa and DTPa-hepB
vaccines. Triple antigen vaccine
containing acellular pertussis has similar efficacy to that
of whole cell pertussis-containing vaccines, but causes
significantly less reaction with a much lower incidence of
fever, irritability and local reactions. The cost is greater.
The vaccine targets the toxin rather than the bacteria.

Katie, 12 years of age, collapses suddenly at school, and


is transported by ambulance with dextrose drip inserted.
On examination, a dolls eye reflex is present but she is
not responding to painful stimuli. Her vital signs are as
follows:
Resp. rate 14/min
Pulse rate 50/min
Sa02 100 %

Question 34

B/P 180/110

An obese man, aged 60 years, is admitted unconscious


with a diagnosis of completed stroke confirmed on CT
scanning as due to a cerebral thrombosis. Of the
following supportive measures, the most important
IMMEDIATE management is:

What is the NEXT step of management?

d) Give intravenous corticosteroids

b) Commencement of anticoagulant therapy


to

prevent

hypostatic

d) Commencement of intravenous fluids to


prevent dehydration
e) Positioning to
decubitus ulcers

prevent

development

b) Replace the dextrose with normal saline


c) Intubate and ventilate

a) Insertion of an indwelling urinary catheter

c)
Physiotherapy
pneumonia

a) Arrange for an urgent head CT scan

of

e) Draw venous and arterial blood for urgent


analysis

Answer: C
Katie has raised intracranial pressure as indicated by the
hypertensive response in the presence of bradycardia
and coma. She is at high risk of herniation and needs
urgent
treatment
to
reduce
the
intracranial
hypertension. The most rapid effect is achieved by

endotracheal intubation and hyperventilation to reduce


arterial pCO2 to 30-35mmHg. This will be accompanied
by an intravenous infusion of mannitol. Urgent
assessment to determine the cause of this presentation
will include blood tests and CT scan. Corticosteroids may
be helpful in cases of intracranial tumour or CNS
infection. Specific treatment will be required for
metabolic abnormalities eg. diabetic ketoacidosis,
infection or haemorrhage.

Question 37
The defect in visual fields MOST commonly associated
with a pituitary tumour is:
a) Crossed homonymous hemianopia

Question 39
Which of the following statements about immunisation is
CORRECT?
a) If a child has a cold, they can't be vaccinated
b) Fever is an uncommon adverse event after
immunisation
c) If a reaction is suspected, a test dose should
be used
d) Anaphylaxis to egg is not a contraindication
to MMR vaccine
e) Pre-term infants
antibody response

have

an

inadequate

b) Central scotoma
Answer: D

c) Bitemporal hemianopia
d) Total blindness in one field
e) Peripheral concentric constriction
enlargement of the blind spot

and

Answer: C
As a pituitary tumour extends upwards from the
diaphragma sellae and compresses the optic chiasm, it
classically causes superior quadrantic defects followed
by bitemporal hemianopia. It can however cause any
variety of visual field defects, including unilateral (or
bilateral) field defects in all quadrants, due to the
variable position of the chiasm above the pituitary.

Question 38
The MOST helpful diagnostic test to
potentially enlarging pituitary neoplasm is:

evaluate

An anaphylactic reaction to egg is not a contraindication


to MMR vaccine. Infants with minor colds without high
fever can be safely immunised. Vaccination should only
be postponed if a child is acutely unwell or has a high
fever (above 38.5 degrees Celsius). A fever (low grade
temperature) is a common adverse effect after
immunisation. Test doses are not recommended, they
are just as likely to produce an adverse event as the full
dose.
Premature
infants
should
receive
their
vaccinations at the same age as term infants, ie first
dose of hepatitis B vaccine at birth and then DTPa-hep
B, HIB and polio vaccinations at 2 months after birth (no
correction for gestation).

Question 40
Which of the following would be MOST helpful in
distinguishing
cerebral
infarction
from
cerebral
neoplasm?
a) History of headache
b) Hemiplegia

a) Cerebral angiography

c) Chronology of development

b) Serial serum prolactin concentration

d) Carotid bruit

c) MRI

e) Focal abnormality on electroencephalogram

d) Computer assisted colour spectrum visual


field plotting
e) CT scan with contrast

Answer: C
Once identified, pituitary neoplasms should be evaluated
by an MRI scan. This gives information regarding
pressure effects on surrounding structures and the need
for surgical treatment. Serum prolactin estimation gives
information about anterior pituitary function, but not
size. Angiography is unnecessarily invasive and CT is not
as sensitive as MRI. Visual field plotting will document
any damage to the optic tract - an unwanted outcome

Answer: C
Chronology of development is the most important factor
in differentiating cerebral infarction from cerebral
neoplasm. Cerebral infarction tends to be a simple,
sudden event or a series of stepwise events within hours
to days. In comparison, neoplasms tend to be preceded
by symptoms such as headache, progressive cognitive
decline, seizures and vomiting, and may feature steadily
progressive neurological signs. The other options are all
variably present in both conditions and are not
diagnostic.

Block 8
Question 1
Bill is 65 years old and has just been diagnosed with
type 2 diabetes. He returns to discuss the condition.
What would you tell him about diabetic retinopathy?
a) He should see an ophthalmologist straight
away for a thorough eye examination
b) It is safe to wait 2 years after diagnosis for
his first ophthalmologist referral
c) His age is the biggest risk factor for diabetic
retinopathy
d) Retinopathy is an uncommon complication of
type 2 diabetes

Question 3
Akira, a long standing patient of your practice wants to
discuss a friend of his who has had his toenail removed
because of a melanoma. Which of the following
statements about subungual melanoma is CORRECT?
a) This is
melanoma

common

form

of

malignant

b) It usually has a good prognosis


c) Removal of the nail may be curative
d) Five year survival depends mainly on tumour
thickness
e) This form of melanoma rarely metastasises

e) None of the above


Answer: D
Answer: A
At diagnosis, one in six patients with Type 2 diabetes
has retinopathy. If untreated, this progresses to cause
retinal scarring, contraction of the vitreous humour and
retinal detachment. Eventually about 85% of all patients
will show signs of retinopathy, the biggest risk factors
being duration of diabetes and glycaemic control. All
patients with diabetes should be screened for
retinopathy at the time of diagnosis, and then at least
every two years thereafter. Laser therapy is very
effective and halves the risk of visual loss from diabetic
retinopathy.

Melanoma located on the palm, sole or nail bed is called


acral lentiginous type and accounts for 2-8% of all
malignant melanomas, with subungual melanoma being
rare. Lifetime risk of melanoma is approaching 1:50,
higher if fair-complexioned, a history of blistering
childhood sunburn, multiple melanocytic naevi or
dysplastic naevi, and a positive family history. Although
melanoma is much less common in non-white groups,
their proportion of acral type is 30-50%. Because of the
location, they are often diagnosed late so have a poorer
outcome. Prognosis is mainly dependent on tumour
thickness and stage as identified by nail bed biopsy.
Treatment involves removal of the digit clear of the
margin.

Question 2
Complete the following sentence: 'In the first few days
of a low calorie diet in the management of obesity ...'

Question 4

b) Most of the weight lost is water

Jim, who has type 2 diabetes, states that he has been


advised by another doctor to have laser treatment to
both eyes. He does not see the need as his vision is fine.
What advice would you offer him?

c) Lean body mass is not affected

a) To have the procedure as quickly as possible

d) Glycogen stores are retained

b) That if he doesn't proceed his vision will


eventually deteriorate

a) More fat than water is lost

e) Sodium loss is reduced

c) That treatment may result in worsening of


night vision
Answer: B
When treating obesity with caloric restriction, the initial
marked weight loss is largely due to fluid loss, but these
changes tend not to persist. During the first 24 hours or
so energy is derived mainly from catabolism of
carbohydrate in the form of glycogen stored, in
association with water, chiefly in the liver. As glycogen
is degraded to glucose and then carbon dioxide and
water, there is significant water release and excretion.
Once glycogen stores have been depleted, glucose
synthesis depends on gluconeogenesis from breakdown
of protein. Finally, only when the glucagon:insulin ratio
in the blood is high enough to promote significant fat
breakdown does fat in adipose tissue stores become the
main source of energy for tissue metabolism with
consequent sustained weight loss.

d) That it is usually performed under local


anaesthetic
e) All of the above

Answer: E
Laser therapy is performed using an apparatus similar to
a slit lamp. Patients rarely require admission to hospital.
If a large amount of photocoagulation is planned,
regional anaesthetic is used. Laser therapy is very
effective and halves the risk of visual loss from diabetic
retinopathy. If untreated, proliferative retinopathy can
cause sudden visual loss in one or both eyes. The new
blood vessels bleed onto the retinal surface causing
scarring, contraction of the vitreous humour and retinal
detachment.

Question 5
A 25 year old man presents with a non-tender swelling
on the right side of the neck, followed by a similar
swelling on the left side two weeks later. He has
recently suffered from periodical fever, malaise and
weight loss. Physical examination shows enlarged lymph
nodes in the neck, axillae and groins, and a spleen
palpable 2 cm below the left costal margin. Chest X-ray
shows bilateral moderately enlarged hilar nodes. The
haemoglobin is 105 G/L (130 - 180), and leucocytes
number 11 x 109 G/L (4.3 - 10.8 x 109) with a normal
distribution of white cells. The red cells appear
normochromic and normocytic.
What is the MOST LIKELY diagnosis?

with removal of crusts is indicated. Persistent lesions


often respond to topical mupirocin applied three times a
day for 7-10 days. If extensive or causing systemic
symptoms oral antibiotics should be used eg
phenoxymethylpenicillin if a Streptococcus is suspected
as the primary infection and flu(di)cloxacillin if Staph
aureus is suspected. Exclusion from child care is only
necessary up to 24 hours after antibiotic treatment
begins.

Question 7
The diagnosis of Hodgkin's disease should be confirmed
by:

a) Aleukaemic leukaemia

a) Lymph node biopsy

b) Hodgkin's disease

b) Repeat blood film and ESR

c) Infectious mononucleosis

c) Bone marrow aspiration

d) Non-Hodgkin's Lymphoma (NHL)

d) Biopsy of the spleen

e) Secondary syphilis

e) Thoraco-abdominal CT scan

Answer: B

Answer: A

Hodgkin's disease usually presents with painless cervical


lymphadenopathy. The disease can spread to involve the
other parts of the reticuloendothelial system such as the
liver and spleen. It is commonly accompanied by
anaemia (normochromic, normocytic), fever, anorexia
and weight loss. Non-Hodgkins Lymphoma presents with
symptoms and signs similar to Hodgkin's Disease, but at
a much later stage of life. Mean age of onset for
Hodgkin's disease is 31 years; mean ages for NHL are
65-70 years. Aleukaemic leukaemia usually presents
with
anaemia,
thrombocytopaenia
and
granulocytopaenia.
Infectious
mononucleosis
is
associated with a sore throat, myalgia, arthralgia and a
maculo-papular rash.

Hodgkin's disease is established by the review of an


adequate lymph node biopsy specimen by an expert
pathologist. Staging will involve all of the following:careful history and examination, full blood examination,
ESR and biochemical analysis including LDH, CT scan of
chest, abdomen, and pelvis, and bone marrow biopsy.
Patients may also undergo a PET or gallium scan.
Staging laparotomy is rarely performed.

Question 8
Which of the following is INCORRECT in relation to
medication usage in diabetics?
a) Aspirin may worsen diabetic retinopathy

Question 6
Anna, a 3 year old child, develops pustular lesions on
her face which subsequently form a honey-coloured
crust and start spreading. You diagnose impetigo. Which
of the following statements would be included in your
advice to her parents?
a) Since Anna is otherwise well, she may
attend her child care centre

b) Metformin is the agent of choice in the


overweight patient
c) Sulphonylureas
pregnancy

are

contraindicated

in

d) Insulin may improve outcomes following


myocardial infarction
e) Beta blockers may increase the risk of
hypoglycaemic episodes

b) There is no danger of spread to family


members
c) Topical treatment will
sufficient to achieve cure

not

usually

be

d) Anna must be isolated until the lesions have


completely resolved
e) Lesions should be adequately covered to
reduce self-inoculation

Answer: E
Impetigo is usually caused by group A streptococci,
other streptococci or Staph aureus. It is usually highly
contagious, so precautions must be taken to reduce
spread. This will include covering the lesions, careful
hand washing, and disposal of items used near the
lesions. If impetigo is mild, topical antiseptic cleansing

Answer: A
Aspirin will not worsen diabetic retinopathy. Trials have
shown that there is no benefit or harm with aspirin in
diabetic retinopathy. Sulfonylureas are contraindicated
in pregnancy because of their effect on the foetus.
Metformin improves glucose uptake by skeletal muscle
cells and has favourable effects on weight and the lipid
profile. It is therefore the agent of choice for overweight
patients. Beta blockers affect glucose metabolism and
may mask early signs of hypoglycaemia. Beta blockers
should not be used in patients with labile insulindependent diabetes mellitus. Even mild hyperglycaemia
increases the risk of extension of damage to
myocardium following an infarct. Studies show that tight
control of blood glucose level using insulin post-AMI
improves outcomes.

Question 9
The specific microscopic feature which should be looked
for in the investigation of Hodgkin's Disease is:

following antibiotics would be MOST effective in treating


Melanie?
a) Phenoxymethylpenicillin

a) A large atypical lymphocyte

b) Amoxycillin

b) A lymphoblast

c) Flucoxacillin

c) A myelocyte

d) Doxycycline

d) A Reed-Sternberg cell

e) Erythromycin

e) A Langhan's giant cell


Answer: C
Answer: D
Hodgkin's disease is characterised by a predominance of
small lymphocytes and, less commonly, large ReedSternberg cells. Reed-Sternberg cells are the histological
hallmark of the disease, they are large malignant
lymphoidal cells with multilobulated nuclei and
prominent inclusion-like nucleoli.

This is a superficial wound which has become


impetigenous, ie like impetigo. In addition the infection
has spread to the surrounding skin and is causing
systemic effects. The causative organism is most likely
Staph aureus. Flucloxacillin (or dicloxacillin) is the most
appropriate antibiotic.

Question 12
Question 10
A man aged 54 years complains of weakness, lassitude,
low back pain and 5kg loss of weight over six months.
Physical examination is normal. Urinalysis shows
protein. Blood count shows: haemoglobin 110g per L
(130 - 180) with red cells normochromic and
normocytic, white cell count 7.5 x 109 per L (4.3 - 10.8)
with a normal differential count; E.S.R. is 102mm in one
hour; Serum protein 98 G per L (55 - 80), x-rays show a
generalised demineralisation of the vertebrae and
sharply defined osteolytic lesions in the skull. Which of
the following investigations is MOST LIKELY to be helpful
in establishing the diagnosis?
a) Bone marrow biopsy
b) Bone scan
c) Prostatic biopsy

Maria comes in to discuss her husband's diabetes with


you. She has read about the glycaemic index (GI) of
food and wants to know more about it, so she can cook
the right foods for him. Which of the following facts is
INCORRECT?
a) Hommus (chickpea salad dip) has one of the
lowest GIs
b) Boiled potatoes have a GI equivalent to 50G
glucose
c) Carbohydrates that are quickly digested
have a higher GI
d) The same amount of carbohydrate may
show a wide variation in GI
e) High GI foods increase metabolism and
assist in weight loss

d) Serum calcium and alkaline phosphatase


e) Thoraco-abdominal CT scan

Answer: A
Multiple myeloma is characterised by the presence of
paraprotein in the serum produced by abnormal
proliferating plasma cells and Bence-Jones protein,
comprising light chain components of immunoglobulins
in the urine. Clinically, it results in bone destruction,
bone marrow infiltration and renal impairment. ESR and
serum protein is raised, and proteinuria is present.
Skeletal survey shows characteristic lytic lesions, easily
seen on the skull. Bone marrow aspirate shows
characteristic infiltration by plasma cells. Definitive
diagnosis depends on identification of abnormal
paraprotein with or without immunoglobulin light chains
in
the
serum
or
urine.
This
is
done
by
immunoelectrophoresis.

Answer: E
The GI is measured by the ability of a particular
carbohydrate food to raise the blood glucose level on a
scale of 1-100 where 50gm glucose is 100. Various
factors affect the GI of a food, including the size of the
starch particles, ratio of amylose to amylopectin, and
the presence of fibre, fat, protein and organic acids.
Wholegrain foods and legumes have a low GI compared
with refined cereals and breads. Lower GI foods are
slower to digest and improve satiety. They assist in
weight management and have a protective effect
against diabetes and heart disease.

Question 13
Which of the following pathological
associated with multiple myeloma?

features

a) Increased plasma alkaline phosphatase


Question 11
Six year old Melanie has been brought to your surgery
with a graze on her knee sustained 3 days ago. Her
mother says the lesion has increased in size and is now
partly covered by a thick golden crust and exudes
serous fluid. The surrounding skin is inflamed and her
temperature is 38.5 degrees Celsius. Which of the

b) Decreased plasma acid phosphatase


c) Increased serum globulin
d) Decreased serum beta2 microglobulin
e) Decreased serum calcium

is

Answer: C
A raised serum globulin is characteristically found in
patients with multiple myeloma. The condition is due to
overproduction of a single immunoglobulin species from
a clone of malignant plasma cells. These antibodies may
not be biologically active, but can cause problems
clinically because of hyperviscosity of the blood, leading
to thrombotic phenomena, and deposition in tissues
such as the kidney, leading to renal failure. There is
hypercalcaemia due to lysis of bone, but plasma alkaline
phosphatase and acid phosphatase levels are usually
normal. A rise in beta2 microglobulin correlates with
myeloma cell mass and a worse prognosis.

Question 14
Complete the following sentence: 'The term 'keratitis'
refers to inflammation of the ...'

diabetes. In order to prevent complications, the patient


needs to achieve strict glycaemic control through weight
loss, diet, and medication as needed. Self-monitoring is
essential. All patients with diabetes should know and
practice routine foot care. They are at risk of damage
because of vascular disease, neuropathy, increased risk
of infection and slower healing. Podiatrists can
implement preventative therapy at an early stage.

Question 16
In patients with multiple myeloma what is the MOST
COMMON clinical course?
a) Steady progression over 2-5 years with
complications
b) Gradual development of myelofibrosis

a) Keratinised layer of the skin

c) A remitting/relapsing course over many


years

b) Germinal layer of the nailbed

d) Rapid decline and death within 1-2 years

c) Glans penis

e) Chronic phase followed by acute terminal


leukemia

d) Cornea
e) Eyelids

Answer: D
Keratitis is inflammation of the cornea; other corneal
lesions include keratoconjunctivitis, keratoconus and
keratectasia. The term "keros" (meaning horny) is also
used to describe layers of skin and of the nails.

Question 15
Susan comes to see you concerned about her 68 year
old mother who has recently been diagnosed with type 2
diabetes. Her mother is refusing to make any dietary or
lifestyle changes, insisting that tablets will keep
everything under control. Which of the following advice
would be appropriate to give Susan about her mother's
condition?

Answer: A
The majority of patients with multiple myeloma
experience
progression
of
the
disease
with
complications,
including
anaemia,
renal
failure,
pathological
fractures,
infections,
neurological
manifestations and bleeding. Only 10% will have an
indolent course with slow disease progression over many
years. In young people, it is an aggressive disease
which may be rapidly fatal (within 1 year). The advent
of bone marrow transplant has improved survival rates
significantly.

Question 17
Complete the following sentence: 'The skin condition
that is COMMONLY responsible for persistent dandruff is
a) Irritant dermatitis

a) This form of diabetes is often mild and strict


adherence to medication regimes will usually be
sufficient in those over 65 years of age

b) Seborrhoeic dermatitis

b) It is difficult to adjust to life with a chronic


illness and encouragement to gradually make
changes over the next few years will suffice

d) Lichen planus

c) There may already be complications present


so it is essential that diet, exercise and good
blood sugar control be adhered to
d) It is only when diabetes is insulin-requiring
that the issue of complications necessitates
tighter control
e) It may be necessary to do a mini mental
state examination to exclude early dementia as
a reason for poor compliance

Answer: C
Type 2 diabetes is not a mild disease. About one third of
those surviving 15 years will require insulin to treat
symptoms or complications. Complications such as
diabetic retinopathy, nephropathy, neuropathy, macro
and microvascular disease occur in both types of

c) Tinea capitis

e) Pityriasis versicolor
Answer: B
Seborrhoeic dermatitis is a chronic inflammation of the
skin characterised by erythema and scaling. It occurs in
areas of skin where sebaceous glands are most active,
and therefore is common on the face and scalp. It can
also affect the centre of the chest and back, axilla, groin
and perianal area.

Question 18
Tom is a 70 year old diabetic and has recently started
on insulin. Which of the following statements about
insulin pens is INCORRECT?
a) The dose should be delivered quickly and
pen immediately withdrawn
b) Short, fine needles of 29-31 guage allow
virtually pain-free injections

c) Pens can now deliver more than one type of


insulin
d) Cartridges containing clouded insulin must
be changed when 12 units remain
e) All but one of the insulin pens in Australia
are made by Novo-Nordisk

Answer: B
Dandruff is caused by seborrhoeic dermatitis which
becomes most prominent when the sebaceous glands
are most active. This occurs most commonly in
adolescence.

Question 21
Answer: A
Insulin pens make insulin injections simpler, since
drawing up and mixing are not necessary and so
multiple daily injections are easier. They are very easy
to use - "just insert the needle and press the button".
The button will not depress if the cartridge is empty. In
clouded insulin gently tilting the cartridge several times
allows a small glass ball to mix the solution. Once less
than 12 units remains there is not enough solution for
the mixing ball to function optimally. The injection
should be given slowly and the needle should be left in
place for six seconds after injection.

Question 19
Roy, 84 years of age, returns for review of test results
taken to investigate his tiredness. His total protein is
elevated with a monoclonal rise in the gammaglobulin
fraction. The full blood count, renal function studies, and
X rays are all normal. There are no urinary Bence Jones
proteins. Referral for a bone marrow biopsy shows
<10% plasma cells. Which of the following statements
about his condition is INCORRECT?
a) His tiredness is due to hypothyroidism which
is often associated with this condition
b) Up to 10% of people over 75 years of age
have similar findings
c) He has about 1% chance per year of
developing multiple myeloma
d) Life expectancy is shorter by 2 years due to
this abnormality
e) There may be an associated carcinoma of
prostate, kidney or gastrointestinal tract

Betty, aged 55 years, has diabetes and comes in for her


annual eye checkup. Which of the following statements
about the eye in diabetes is CORRECT?
a) If the patient doesn't have symptoms, the
examination is likely to be negative
b) Fundoscopy without pupillary dilatation is
adequate for screening purposes
c) A normal red reflex makes the presence of a
cataract unlikely
d) Pupillary reflex is unaffected by diabetes
e) Assessment of refractive error is not a
routine part of the diabetic eye check

Answer: C
At diagnosis, one in six patients with Type 2 diabetes
has retinopathy and eventually about 85% will develop
this complication. Irrespective of symptoms, all patients
with Type 2 diabetes should be screened on diagnosis,
and then at least every two years thereafter.
Examination includes checking red reflex for cataracts,
dilating the pupils and performing fundoscopy to
observe the macula, optic disc, and other areas. Blood
glucose concentration should also be checked. Dense
cataracts cause the red reflex to be totally obscured;
smaller cataracts will be seen as opacities against the
red reflex. Refractive errors occur as the lens shape
alters with changes in blood glucose concentrations.

Question 22
A 60 year old man presents with severe abdominal pain,
shock, moderate abdominal rigidity and intense back
pain. Which of the following diagnoses is MOST LIKELY?

Answer: A

a) Acute retrocaecal appendicitis

Monoclonal gammopathy of uncertain significance


(MGUS) is occasionally associated with other conditions
such as non-lymphoreticular neoplasms, chronic
inflammation or infection, and thyrotoxicosis. With a low
risk of progression, clinical and blood test review is
necessary every 6-12 months. All the other statements
are correct.

b) Leaking aortic aneurysm


c) Renal colic
d) Acute cholecystitis
e) Collapse of L4 vertebral body

Answer: B
Question 20
Complete the following sentence: 'Dandruff commonly
presents in....................'
a) Childhood
b) Adolescence
c) Early adulthood
d) Middle age
e) Old age

A leaking abdominal aortic aneurysm typically presents


with severe abdominal pain, shock, abdominal rigidity
and intense lower back pain. It can be mistaken for
renal colic, acute cholecystitis, and retrocaecal
appendicitis. However circulatory shock is not usually
present in these conditions. The BP may be increased
due to pain. Collapse of the L4 vertebral body results in
more localised pain without shock.

Question 23

Question 26

Which of the following pathological changes is MOST


LIKELY to occur in x-irradiated skin?

Which of the following statements about adolescent girls


with suspected iron deficiency anaemia is INCORRECT?

a) Telangiectasis
b) Melanoma
c) Keloid formation
d) Excessive hair growth
e) Chronic hyperpigmentation

a) Early symptoms can include problems with


academic performance
b) The onset of menstruation is the main
contributor to this condition
c) A normal plasma ferritin level excludes the
diagnosis
d) The blood film
hypochromic picture

Answer: A
Telangiectasia (dilated superficial blood vessels), is the
most visible aspect of chronic radiation dermatitis. The
changes following radiotherapy are permanent in the
long run, and also include epidermal atrophy and
changes in hyalinization and hair growth.

Question 24
In haemolytic jaundice, bilirubin is absent from the urine
because plasma bilirubin is:
a) Not usually elevated
b) Deposited in skin and sclera
c) Unconjugated and not water soluble
d) All protein bound and thus not filterable
e) Converted by the liver to urobilinogen

shows

microcytic,

e) Coeliac disease is a possible cause

Answer: C
Iron deficiency is common in adolescent girls with a
prevalence of 9%. A low plasma ferritin indicates low
total body stores of iron. However, ferritin is an acute
phase reactant, so its plasma level is increased by acute
or chronic inflammation. This may mask iron deficiency.
Early symptoms of iron deficiency include changes in
cognitive function and memory, decreased concentration
and fatigue.

Question 27
Which of the following conditions is MOST LIKELY to
arise in an actinic keratosis?
a) Malignant melanoma
b) Squamous cell carcinoma

Answer: C

c) Basal cell carcinoma

In haemolytic jaundice there is an increased breakdown


of red blood cells leading to increased production of
bilirubin in reticulo-endothelial cells. As this occurs prior
to the bilirubin reaching the liver (where it gets
conjugated with glucuronic acid), the increased plasma
bilirubin is unconjugated and not water soluble. It will
therefore not pass into the urine. Urinary urobilinogen is
increased because there is more bilirubin available for
conversion to urobilinogen by gut bacteria, and
urobilinogen is water soluble.

d) Kerato-acanthoma
e) Bowen's disease

Answer: B
Squamous cell carcinoma develops in solar keratosis
over a long period of time. The rate of malignant change
is thought to be 1/1, 000.

Question 25

Question 28

A patient who has been treated with a preparation


containing horse serum develops urticaria followed by
swelling of the tongue and dyspnoea. Which of the
following is the MOST APPROPRIATE immediate
treatment?

With regard to Hepatitis C in Australia, which of the


following statements is INCORRECT?

a) Tracheotomy
b) Subcutaneous adrenaline
c) Intravenous hydrocortisone
d) Intravenous promethazine (Phenergan)
e) Oxygen therapy

Answer: B
This is acute angio-oedema and there is a risk of upper
airways closure and anaphylaxis, so subcutaneous
adrenaline should be given first.

a) About 16,000 new infections are occurring in


Australia each year
b) Around 83% of existing infections are due to
sharing equipment used to inject illicit drugs
c) Couples in which one partner is hepatitis C
positive are advised to always use condoms to
avoid sexual transmission
d) For mothers who are hepatitis C positive,
breastfeeding is encouraged unless nipples are
cracked or bleeding
e) This infection is now the most common
reason for liver transplantation in Australia

Answer: C

Question 31

Although there is some uncertainty about hepatitis C


being transmitted sexually, it is not classified as an STD
(sexually transmitted disease). If it occurs, sexual
transmission is rare and probably more likely if there is
blood-to-blood contact during sex. With a new or casual
partner or in case of possible blood-to-blood contact,
safe sex practices should always be used to protect
against a range of STDs. All the other statements are
correct. Further information may be found in the
reference.

Which one of the following is NOT characteristic of an


upper motor neurone lesion?

Question 29

a) Clonus
b) Extensor plantar response
c) Intact superficial reflexes
d) Increased tone
e) Hyperactive tendon reflexes

Answer: C

In which of the following conditions are nails MOST


COMMONLY affected?
a) Systemic lupus erythematosis (SLE)
b) Psoriasis
c) Iron deficiency
d) Infective endocarditis (IE)

The following signs result from lesions in the motor


system proximal to the alpha motor neurone: spasticity
(hypertonia predominant in flexors of arms and
extensors of legs which is of a clasp-knife nature);
paralysis or weakness predominantly of extensors in
arms and flexors in legs; hyperreflexia; extensor plantar
response; clonus and Hoffmann's reflex. The extensor
plantar response is an example of loss of a superficial
reflex.

e) Hypoalbuminaemia
Question 32
Answer: B
Psoriasis commonly affects the nails. Pitting of the nails,
occurs in 25% of patients. Other effects include
hyperkeratosis, ridging, and onycholysis i.e. separation
of the distal nail from the nail bed (which is often
mistaken for fungal paronychia). Splinter haemorrhages
and telangiectasia occur in SLE; spoon-shaped nails
(koilonychia) in iron deficiency; splinter haemorrhages
and clubbing in IE; and white nails (leukonychia) in
hypoalbuminaemia.

Moira is a 70 year old woman who presents with


abdominal discomfort for 6 months. On examination she
has generalized lymphadenopathy and splenomegaly.
What is the MOST LIKELY diagnosis?
a) Acute lymphoblastic leukaemia
b) Acute myeloid leukaemia
c) Hodgkin's lymphoma
d) Infectious mononucleosis
e) Chronic lymphocytic leukaemia

Question 30
Miriam has recently developed bloating and heartburn
after meals. On reviewing her medications which of the
following is UNLIKELY to be a cause of this new
problem:
a) Iron supplements
b) Progestogen therapy
c) An ACE inhibitor
d) A non-steroidal
(NSAID)

ant-inflammatory

Answer: E
Chronic lymphocytic leukaemia is a disease of late
middle-aged
and
elderly
people.
It
may
be
asymptomatic in the early stages and is often diagnosed
on a routine blood count. Symptoms, which are insidious
in onset, include lethargy, fevers, loss of weight and
infections. Signs include moderate enlargement of the
lymph nodes, liver and spleen.

drug

e) Long term tetracycline

Answer: C
While ACE inhibitors may cause nausea, more common
adverse effects are hypotension, cough, dizziness and
hyperkalemia. Abdominal pain and nausea are common
side effects of iron supplements. Bloating and nausea
are commonly reported with progestogen therapy.
Dyspepsia, nausea and diarrhoea are commonly
reported by users of NSAIDs and these are contraindicated in those with peptic ulcer disease.
Oesophageal ulceration can occur with tetracycline, thus
patients are advised not to lie down within one hour of
taking a dose.

Question 33
Francesco Napoli, aged 46 years, presented complaining
of years of gastro-oesophageal reflux. He was found to
have positive Helicobacter pylori antibodies. Regarding
H. pylori, which of the following statements is
CORRECT? H. pylori:
a) Is the most common bacterial infection of
humans worldwide
b) Is positive in about 60% of adult Australians
c) Infects humans from a reservoir in healthy
domestic animals
d) Causes peptic ulcer disease and duodenal
cancer
e) Is present in >80% of patients with nonulcer dyspepsia

Answer: A
Some parts of the world, particularly developing
countries have 80-90% prevalence of H pylori infection
in adults. Australian prevalence is about 30-40%. Most
infections are acquired in childhood from a parent.
Humans are the only reservoir. This infection is a known
cause of peptic ulceration and the organism is now
classified as a biological carcinogen due to its strong
connection with gastric cancer. Only 25-50% of people
with non-ulcer dyspepsia are positive for H pylori and
most will not have resolution of symptoms after H pylori
eradication.

sounds are dual. The MOST LIKELY cause of his


condition is:
a) Hospital acquired pneumonia
b) Post-infarct left ventricular remodelling with
failure
c) Extension of the infarct secondary to stent
failure
d) Pericarditis with tamponade
e) Bacterial endocarditis

Question 34

Answer: D

Mrs Elaine Wu underwent endoscopy for recent onset of


dyspepsia. The biopsy verified the presence of
Helicobacter pylori-positive gastric ulceration. She is not
allergic to any medication. The MOST APPROPRIATE first
line therapy is:

Pericarditis is a recognised, although uncommon,


complication of invasive cardiac treatments. The patient
can lose the pain of angina, only to have it replaced by a
more vague chest discomfort. Inflammatory pericarditis
results in an effusion which can rapidly escalate into
tamponade. None of the other options would show signs
of biventricular failure this quickly.

a) A histamine type 2 receptor antagonist for 6


to 8 weeks
b) A proton pump inhibitor (PPI) for 4 to 6
weeks
c)
Ranitidine-bismuth-citrate
400mg
bd,
amoxycillin 1G bd, and clarithromycin 500mg
bd for 7 days
d) PPI bd, amoxicillin 1G bd, and clarithromycin
500mg bd for 7 days
e) PPI bd, clarithromycin 500mg bd, and
metronidazole 400mg bd for 7 days

Answer: D
Acceptable triple therapy regimens for the eradication of
H.pylori vary depending on
the availability of medications and the incidence of
resistance
to
antibiotics
used.
In
Australia,
metronidazole-resistant strains are now common
(>50%) and metronidazole can no longer be
recommended for first-line therapy if there is no allergy
to penicillins. The combination of a standard dose of PPI,
amoxicillin 1G and clarithromycin 500mg all delivered bd
for 7 days gives an eradication rate of >90%. It is
available as a single script on the Pharmaceutical
Benefits Scheme as either "Klacid Hp7" or "Nexium
Hp7". In penicillin hypersensitivity the amoxicillin is
replaced with metronidazole 400mg bd with efficacy 8085%. In treatment failure, there may be attempts to
conduct bacterial culture and sensitivity testing.
Otherwise a trial of quadruple therapy using a PPI,
bismuth subcitrate, metronidazole and tetracycline for
10-14 days will give 75% chance of eradication. See
references for further discussion about when to consider
H pylori testing and eradication and how to follow up
treated patients.

Question 35
A 67 year old man presented three days after a stent
was inserted for the treatment of persistent angina. He
now complains of a persisting "different" chest pain and
shortness of breath on exertion. On examination you
find he is pale and slightly sweaty with: pulse rate 110
regular with pulsus paradoxus; BP 100/90; T 38.0
degrees Celsius; pedal oedema; bilateral basal
crepitations in his chest; and a JVP elevated 3cm. Heart

Question 36
A 60 year old engineer was admitted to hospital because
of fever, cough, and pleuritic chest pain. His
temperature was 40 degrees Celsius. Physical
examination and x-ray of the chest indicated right lower
lobar
pneumonia.
Sputum
smear
and
culture
demonstrated pneumococci. Treatment with intravenous
penicillin was commenced. After several days, fever and
leucocytosis decreased and x-ray of the chest showed
some clearing of infiltrate. On the 7th hospital day, his
temperature spiked to 39.4 degrees Celsius, there was
an increase in cough and dyspnoea. X-ray of the chest
showed an increase in pulmonary infiltrate. Which of the
following is the MOST LIKELY explanation of this clinical
picture?
a) Development of pneumococcal resistance to
penicillin
b) Laboratory contamination of the original
culture
c) Superinfection by a different type of bacteria
d) Adverse effect of antibiotic therapy
e) Pulmonary thromboembolism

Answer: D
This clinical picture demonstrates deterioration in the
patient's condition. Drug fever, or serum sickness,
usually occurs on the 7th to 12th day of antibiotic
therapy and can produce unexpected fevers, skin rash
and an eosinophilic pulmonary infiltrate. It is commonly
due to penicillins, nitrofurantoin, sulphonamides,
thiazides and tricyclic antidepressants. As he had been
improving over 6 days, it is unlikely that the original
specimens were contaminated, nor that the organisms
had had the opportunity to develop resistance.
Pulmonary embolism does not cause a high fever.
Superinfection is commonly due to gram-negative
bacteria, fungi or resistant staphylococci and usually
appears on the 4th or 5th day.

Question 37

Question 39

Which of the following is NOT a likely complication from


gastro-oesophageal reflux disease?

Brian and his wife are contemplating a trip across Africa.


Which of the following situations is most likely to put
them at risk of contracting typhoid fever?

a) Achalasia of the oesophagus

a) Intimate contact with an infected person

b) Ulcerative oesophagitis

b) Eating contaminated food

c) Barrett's oesophagus

c) Swimming in contaminated water

d) Adenocarcinoma of the oesophagus

d) Contact with an infected dog

e) Oesophageal stricture

e) Scratches from infected wildlife

Answer: A
Achalasia is a motility disorder of the lower oesophagus
of unknown cause and is unrelated to gastrooesophageal reflux disease (GORD). It is characterised
by a dilated lower oesophagus in the presence of a tight
lower oesophageal sphincter which does not respond to
normal peristalsis. GORD leads to chronic inflammation
of the lower oesophagus, which in turn can ulcerate or
cause a fibrosed stricture. Barrett's oesophagus (or
metaplasia) is also a result of long-standing GORD,
occurring in 10% of patients with reflux. 10% of
patients with Barrett's go on to dysplastic then
malignant change (adenocarcinoma) of the oesophagus.

Answer: B
Salmonella
typhi
is
spread
primarily
through
consumption of contaminated food or water. It has no
known animal host other than humans. Intimate contact
with infected persons is a relatively uncommon mode of
transmission. Swimming in contaminated water would
not lead to infection unless the water was ingested.

Question 40
Which of the following is NOT a feature of Parkinson's
disease?
a) Bradykinesia

Question 38
If a patient is said to have odynophagia you would
assume that:

b) Rigidity
c) Difficulty starting a movement

a) It is painful to swallow

d) Hypophonia

b) It is painful to speak

e) Intention tremor

c) The patient has trouble swallowing and


hearing
d) The patient has aberrant taste sensations
e) There is
oesophagus

incoordinate

action

of

the

Answer: A
Odynophagia is pain during swallowing as opposed to a
difficulty swallowing (dysphagia). It is usually a
symptom of mucosal destruction.

Answer: E
Parkinsonism is a syndrome consisting of tremor, rigidity
and bradykinesis. The tremor is most marked at rest,
coarser than cerebellar tremor and is seen as a pill
rolling of thumb over fingers. Bradykinesis involves
slowness in initiating and executing movement and
speech. The gait is characterised by a shuffling forward
with flexed trunk (called a festinant gait). Speech is
hypophonic with a characteristic monotonous, stuttering
dysarthria.

Block 9
Question 1
Jeremy has just been born via normal vaginal delivery.
His Apgar scores are being documented. These are the
observations made at 1 and 5 minutes of age: 1 minute:
blue and pale; heart rate <100beats/minute; irregular,
slow respiration; coughs with stimulation; limp 5
minutes: pink body, blue extremities; heart rate
>100beats/minute; good cry; coughs with stimulation;
active Which of the following BEST indicates Jeremy's
Apgar scores at 1 and 5 minutes respectively?
a) 4 and 9
b) 5 and 10
c) 3 and 9
d) 4 and 8
e) 5 and 9

Answer: A
1
minute:
blue
and
pale
(0);
heart
rate
<100beats/minute (1); irregular, slow respiration (1);
coughs with stimulation (2); limp (0). Score of 4. 5
minutes: pink body, blue extremities (1); heart rate
>100beats/minute (2); good cry (2); coughs with
stimulation (2); active (2). Score of 9. Please see
reference to review Apgar scores.

Question 2
Richard is 4 years old. He has just fallen over in the
playground at kindergarten and knocked out (avulsed)
his upper left front tooth. The tooth has been found on
the ground. Richard's teeth are all primary teeth. He has
not lost any teeth prior to this. Which if the following
statements describes the BEST initial management?
a) Immediately replace the tooth in its socket
taking care to ensure its correct orientation
b) Keep the tooth moist in saline, milk, or
Richard's saliva and seek dentist review within
the hour
c) Carefully examine his mouth to check for
other damage to teeth, bone or soft tissue
d) Thoroughly clean the root of the tooth, wrap
in plastic and transport on ice for dental review
within 2 hours
e) Prescribe antibiotics, preferably penicillin or
clindamycin if penicillin-allergic

Answer: C
Avulsed primary teeth are not usually reimplanted due
to the risk of damage to the developing permanent
tooth. Loss of a primary tooth rarely impacts on future
occlusal development or tooth space. At least 50% of
children will have a dental injury before 12 years of age.
This may involve luxation of a tooth and soft tissue or
alveolar bone damage or even mandibular fracture in

chin point trauma. It is essential that all teeth are


checked and that any missing teeth or fragments are
located. Radiographs may be necessary to check for
ingestion, inhalation or soft tissue displacement of teeth.
Avulsed permanent teeth should be kept in saline, milk
or the child's saliva and replaced within half an hour.
Antibiotics are used only if clinically indicated.

Question 3
Natalie is 12 years old. She presents with fever, an
exudative pharyngitis, generalised lymphadenopathy
and splenomegaly. This condition is MOST LIKELY
caused by:
a) CMV (cytomegalovirus)
b) Group A streptococcus
c) HSV (herpes simplex virus)
d) RSV (respiratory syncitial virus)
e) EBV (Epstein Barr virus)

Answer: E
Natalie is most likely to have glandular fever (infectious
mononucleosis) due to Epstein-Barr virus. Pharyngitis is
the major symptom, and is exudative in 50% of cases.
Splenomegaly is present in 50 - 75% of cases. CMV may
cause a similar illness, but pharyngitis and adenopathy
are less apparent. Pharyngitis due to Group A
Streptococcus usually does not cause splenomegaly.
HSV may cause gingivostomatitis but it rarely extends to
the pharynx. RSV is a pathogen affecting the lower
respiratory tract.

Question 4
Janet was born 3 hours ago at 35 weeks gestation. She
has clinically apparent Down's syndrome. She has
passed meconium normally. Janet has vomited several
times, and the vomitus is bile-stained. Which of the
following features would you expect to see on abdominal
x-ray?
a) Normal abdominal gas pattern
b) Diffuse bowel distension
c) Multiple dilated loops of bowel and intraabdominal calcifications
d) Dilated proximal colon and absence of gas in
the pelvic colon
e) Dilated stomach and proximal duodenum
with no air distally

Answer: E
Janet has duodenal atresia, a condition commonly
associated with other congenital abnormalities including
Down's syndrome. Abdominal x-rays show the typical
'double bubble' appearance, due to distension of the

stomach and proximal duodenum with air, and the


absence of gas distal to the obstruction. Diffuse bowel
distension suggests meconium plug syndrome or
Hirschsprung's
disease
(congenital
megacolon).
Hirschsprung's disease may also be seen as dilated
proximal colon and absence of gas in the pelvic colon.
Multiple dilated loops of bowel and intra-abdominal
calcifications are seen with jejunoileal atresia or
meconium ileus.

Answer: C
The ductus arteriosus closes in normal term infants at 3
to 5 days of age. Patent ductus arteriosus is a common
congenital cardiac abnormality, especially in premature
babies. The incidence decreases with increasing maturity
of the baby and is approximately 1:2000 in term infants.

Question 7
Question 5
Phillipa, aged 2 years, has just been diagnosed with
autism. You are discussing the disorder with her
distressed parents. Which of the following statements is
CORRECT?
a) Phillipa's parents have a 25% chance of
having another child with autism
b) Autistic children are extremely resistant to
change in routines or patterns of behaviour
c) The diagnosis of autism is increasing with a
prevalence of approximately 2 per 10,000
children
d) The range of intelligence of children with
autism is the same as non-autistic children
e) Autistic children often form a strong bond
with their main carer and suffer marked
separation anxiety

Answer: B
Autism is one of a group of diagnoses called Pervasive
Developmental Disorders. The incidence is as high as 26 per 1000 children. Parents of a child with autism have
a 2-9% chance of having another affected child. Autistic
disorder presents before the age of 3 years with
sustained impairments in reciprocal social interactions,
communication and restricted, stereotypical behaviour
patterns. The lack of attachment or bonding to carers
and resistance to change is typical. Up to 70% will have
IQ's lower than normal. At the other end of the Autism
Spectrum Disorders, children with Asperger's Syndrome
may have normal or high IQ's, no language delay, are
more adaptable but show impaired social interaction and
restricted patterns of behaviour or interests. Early
diagnosis and treatment has been shown to improve
long-term outcomes for many children.

Question 6
Olivia is 7 days old. She was born at term in an
uncomplicated normal vaginal delivery. She has a rough
continuous 'machinery' murmur maximal at the left
sternal edge, second intercostal space. Clinical findings
are consistent with patent ductus arteriosus (PDA).
When does the ductus arteriosus USUALLY close in a
normal term infant?
a) With the infant's first inspiration
b) Within one hour of birth
c) By day 3 to 5 of life
d) By the age of six weeks
e) Within the first six months

Benson is 4 years old. He was recently unwell with an


influenza-like illness, from which he appears to have
recovered completely. Benson has now developed
marked periorbital swelling, and some peripheral
oedema. He is not unwell, and has no other specific
symptoms. Dipstick testing of Benson's urine reveals
>3+ proteinuria. Which of the following is the MOST
APPROPRIATE next step in the management of this
condition?
a) Renal biopsy
b) Commence oral prednisolone
c) Commence oral penicillin
d) Commence oral diuretic therapy
e) Reassurance and expectant management

Answer: B
This presentation characterises a child with nephrotic
syndrome. In young children, this is usually idiopathic
nephrotic syndrome of childhood (minimal change
disease). Renal biopsy is not usually performed unless
there is poor response to treatment. The best initial
management is corticosteroid treatment. Prednisolone is
commenced at 2mg/kg/day (maximum 60mg/day) until
urine protein levels fall to trace or negative levels, or for
a maximum of 8 weeks, then therapy is tapered.
Diuretics are rarely indicated and should be used with
extreme care. Antibiotic prophylaxis may be considered
during relapses, and pneumococcal vaccine may be
worthwhile. Expectant management is inappropriate.
Complications include infection, thromboembolism and
hypovolaemia and all are treated aggressively. Some
advocate prophylactic penicillin whenever there is
severe oedema. Occasionally aspirin may be used to
prevent clotting however the main strategy is
mobilisation and avoidance of dehydration. About 70%
of children have relapses but most of these will have
ceased relapsing by 18 years of age. It is very rare for
steroid-sensitive minimal change disease to progress to
chronic renal failure.

Question 8
Catherine is a six week old baby. She presents to
hospital with a history of vomiting after feeds, which her
parents describe as projectile. Which of the following
arterial blood gas (ABG) results is CONSISTENT WITH a
diagnosis of pyloric stenosis?
a) pH 7.41, pCO2 40, pO2 100, HCO3- 24
b) pH 7.51, pCO2 48, pO2 90, HCO3 30
c) pH 7.50, pCO2 29, pO2 105, HCO3 19
d) pH 7.19, pCO2 52, pO2 80, HCO3 18
e) pH 7.28, pCO2 60, pO2 85, HCO3 33

Answer: B

Answer: A

Infants with pyloric stenosis develop a hypochloraemic


alkalosis with potassium depletion, secondary to
vomiting. The compensatory respiratory response is
hypoventilation to retain carbon dioxide. The parameters
given in option (a) represent a normal ABG analysis.
Option (c) represents a respiratory alkalosis with
metabolic compensation, (d) shows a mixed metabolic
and respiratory acidosis, and option (e) is an example of
respiratory acidosis with metabolic compensation.

Genu varum (bow-legs) is common in toddlers to the


age of 3 years. If asymmetrical or the intercondylar
distance at the knees when standing is greater than 6
cm, Xray and referral is advisable to exclude the
possibility of Blount's disease or rickets. Genu valgum is
common between 3 and 7 years of age. The majority
improve spontaneously. Refer if the intermalleolar
distance at the ankles is greater than 8cm when
standing. Femoral anteversion usually leads to toeing in
due to excessive internal rotation of the femur.
Corrective footwear is not necessary, nor is
physiotherapy.

Question 9
Steven is 8 years of age. He is miserable and
complaining of a sore throat. He has a fever of 39.2
degrees Celsius, tender cervical lymphadenopathy and
tonsillar exudate with no cough. Steven has a
documented Type I hypersensitivity reaction to
penicillin. Which of the following statements describes
the BEST management of Steven's condition?

Question 11

a) Regular paracetamol, fluids and expectant


management

Timothy is 2 years old. He presents with a small crop of


lesions on his left outer thigh, just above the knee. The
20 or so lesions are pale papules ranging in size from 2
to 5mm in diameter. Each papule has a central
umbilication. They do not seem to be itchy or bothering
Timothy, but his parents are very worried about them.
Which if the following statements is TRUE?

b) Phenoxymethylpenicillin 250mg orally 12


hourly for 10 days

a) Timothy is being abused, as this is a sexually


transmissable disease

c) Amoxycillin 375mg orally 8 hourly for 10


days

b) A moderately potent topical steroid is the


treatment of choice

d) Roxithromycin 100mg orally 12 hourly for 10


days

c) Timothy should be investigated for causes of


immunodeficiency

e) Cefaclor 250mg orally 8 hourly for 10 days

d) Resolution may take from 3 months to 5


years
e) The most likely causative pathogen is human
body louse

Answer: D
Steven has tonsillitis and known penicillin allergy. On
the basis that his symptoms are suggestive of
streptococcal infection, Steven would benefit from
antibiotics and the best management option is
roxithromycin. Phenoxymethylpenicillin and amoxycillin
are obviously contraindicated. Cefaclor is also
contraindicated on the basis of possible cross-reactivity
in a patient with a well-documented penicillin allergy.
Regular paracetamol and fluids may well be of benefit,
however antibiotics will improve symptoms and reduce
the incidence of complications in bacterial tonsillitis.

Question 10
Robert is 4 years of age. He is developing normally and
is of normal height and weight. However his mother is
concerned as she has noted that he seems to be quite
'knock-kneed'. Robert's physical examination finds an
intermalleolar distance of 6cm when he is standing with
knees together. He is otherwise normal. Which of the
following statements is CORRECT regarding genu
valgum in this case?
a) Reassurance is appropriate
represents normal development

as

this

b) Robert should be immediately referred for an


orthopaedic opinion
c) Xray may confirm the cause as femoral
anteversion
d) Podiatrist advice regarding an appropriate
orthotic should be sought
e) Physiotherapy has been shown to improve
prognosis if commenced before age 5

Answer: D
Timothy has molluscum contagiosum. This is a poxvirus,
causing characteristic lesions(firm, smooth, round,
usually flesh-coloured papules with a central indentation
or 'umbilication'). Chemical therapies are usually
ineffective in treating these lesions, and conservative
management is usually best. Resolution may take up to
two years. The virus is spread by autoinnoculation, and
severe or widespread disease may require more active
management. This is a very common disorder in
childhood. It may be spread by sexual contact in
adolescents and adults, but is usually acquired from
family members and other children in childhood.
Molluscum contagiosum may be seen in advanced
stages of HIV infection, but is not a flag to investigate
for HIV in this age group.
Question 12
Victor is a healthy 6 month old baby. He has a left
hydrocoele, present since birth. It is relatively large and
brilliantly transilluminable. Which of the following
statements is CORRECT?
a) The swelling is usually reducible
b) The swelling is frequently painful
c) Persistent swelling at 6 months requires
surgical correction
d) The swelling will usually resolve in the first
years of life
e) Aspiration of the swelling is usually curative

Answer: D

Answer: B

The natural tendency for the processus vaginalis to close


in the first years of life will usually cause resolution of
the hydrocoele. Aspiration may temporarily reduce the
swelling but poses a risk for bleeding and secondary
infection. Surgery is usually only required for a
hydrocoele persisting beyond 2 years of age, unless it is
extremely large. A hydrocoele is not reducible, and is
not usually painful.

If foreign body aspiration is suspected, the patient


should have a chest x-ray comprising inspiratory and
forced expiratory views. Inspiratory views may show
localised hyperinflation due to gas trapping distal to the
foreign body. The forced expiratory film may show
mediastinal shift away from the affected side.
Bronchoscopy is indicated if a foreign body is detected,
or if clinical suspicion persists despite negative chest xray.

Question 13
Nicholas is 14 years old. He has clinically apparent
Marfan syndrome. Which of the following cardiac
murmurs would you expect to hear on cardiac
auscultation?
a) Decrescendo high pitched diastolic murmur
at left sternal edge
b) Midsystolic ejection murmur at 2nd right
intercostal space
c) Low pitched rumbling diastolic murmur at
the apex
d) Pansystolic murmur at left sternal edge with
no radiation
e) Continuous machinery murmur at 2nd left
intercostal space

Answer: A
Marfan syndrome is an autosomal dominant connective
tissue disorder with an incidence of about 1 in 10,000,
of whom 25% present as a spontaneous mutation.
Aortic dilatation causes aortic regurgitation, the murmur
of which is usually decrescendo and high-pitched,
beginning immediately after the second heart sound and
extending for a variable time into diastole. A2 may be
soft. Mitral valve prolapse may develop very early in life
presenting as a midsystolic click. With time many go on
to develop mitral regurgitation with an apical pansystolic
murmur radiating into the axilla. The other options
describe, in turn, aortic stenosis, mitral stenosis,
ventricular septal defect and patent ductus arteriosis
which are not features of Marfan syndrome.

Question 15
Stephanie is 12 months old. She has just begun to walk,
but keeps falling over. It seems she has one leg slightly
longer than the other. You suspect she may have
congenital dislocation of the hip (CDH). Which of the
following statements regarding this condition is TRUE?
a) Ultrasound is the investigation of choice in a
child of this age
b) The incidence of CDH is approximately 1:100
live births
c) After the first month of life, signs of hip
instability become more evident
d) Use of two or three layers of nappies
(diapers) to splint the hip is frequently
beneficial
e) CDH is more common in female than in male
infants

Answer: E
Congenital dislocation of the hip is more common in
female than in male infants. The incidence of CDH is
approximately 1:1000 live births. After the first month
of life, signs of hip instability become less evident,
hence the condition is more easily missed after this
time. X-ray is the best investigation for suspected CDH
at the age of 12 months. Use of multiple nappies is
never indicated as a treatment for this condition, as they
are inadequate to obtain proper positioning of the hip.

Question 16
Question 14
Jake is 15 months old. His mother thinks he may have
inhaled the wheel from a small toy car at a birthday
party this afternoon. At the party he choked and
coughed briefly then seemed to settle. However, since
that time he has seemed a bit wheezy and has been
coughing intermittently. Which of the following would be
the BEST initial investigation to confirm the presence of
a lower respiratory tract foreign body?

Trent is 14 years old and a keen athlete. He presents


with left anterior knee pain. Tenderness of the left tibial
tubercle raises your suspicion of Osgood-Schlatter
disease. X-ray confirms your diagnosis and excludes
other pathology. Trent's coach is keen for him to be
treated immediately so he can resume training at full
capacity. Which of the following represents the BEST
management option for this condition?
a) Reduction or modification of activity

a) Bronchoscopy

b) Electrotherapy

b) Chest x-ray

c) Corticosteroid injection

c) Arterial blood gases

d) Plaster cast immobilisation

d) CT scan

e) Surgery

e) White cell count

Answer: A

Question 19

Management
of
Osgood-Schlatter
disease
is
conservative as this is a self-limiting condition. Ice and
analgesics are appropriate for acute management of
inflammation. The mainstay of treatment is to modify
the child's level of activity to minimise aggravation of
the condition. Complete abstinence from activity is
unnecessary and undesirable.

Renee is 5 years old. She had a mild cold with a clear


runny nose two weeks ago, which required no drug
treatment, and has been well since. She presents with a
purpuric rash on the fronts of her legs and feet, and her
buttocks. The rash was urticarial initially. She appears
well and happy, with no other abnormal clinical findings.
Renee's platelet count is normal. What is the MOST
LIKELY diagnosis?

Question 17
Christina is 4 years old. She has just been found to have
drunk a good portion of a full bottle of children's
paracetamol. Which of the following statements
regarding paracetamol overdose in children is
CORRECT?

a) Henoch Schonlein purpura


b) Meningococcal septicaemia
c) Idiopathic thrombocytopaenic purpura
d) Disseminated intravascular coagulation
e) Aplastic anaemia

a) The risk of hepatotoxicity is greatest in


children under the age of 5 years
b) Paracetamol levels are best measured as
soon as possible post-ingestion
c) N-acetylcysteine may be beneficial even
when administered more than 24 hours postingestion
d) Liver function tests are used to assess the
need for treatment
e) Normal liver function tests at 48 hours post
ingestion are indicative of patient well being

Answer: C
N-acetylcysteine is given to patients whose paracetamol
levels are in the toxic range according to a standard
nomogram, and may still be beneficial when
administered more than 24 hours post-ingestion. The
nomogram for calculating the requirement for Nacetylcysteine therapy is based on paracetamol blood
levels, not on liver function test parameters. Significant
abnormalities of liver function may not develop until 72
hours post-ingestion. The incidence of hepatotoxicity is
ten times higher in adults and adolescents than in
children under 5 years of age. If paracetamol is ingested
in a liquid form, blood levels may be taken two hours
after ingestion to give an accurate toxicity picture,
otherwise the level should be taken at 4 hours postingestion.

Answer: A
Henoch Schonlein purpura is the most common small
vessel vasculitis in children. It is commonly preceded by
a viral upper respiratory tract infection. Less commonly
a drug may be implicated in its onset. The rash may be
urticarial initially, indurating and becoming purpuric. The
extensor surfaces of the feet and legs, arms and the
buttocks are frequently involved. New lesions can
appear for 2-4 weeks. This disease can also be
associated with fever, polyarthralgia, abdominal pain
and melaena, proteinuria and haematuria. The platelet
count is usually normal, but may even be elevated.

Question 20
Harvey aged 10 years, was playing football this
afternoon when he was heavily tackled. He had to leave
the field as he was injured in the incident, and has had
severe left flank pain since. He also complains of some
dizziness. Examination confirms left flank tenderness.
You suspect Harvey may have a ruptured spleen. Which
of the following plain abdominal x-ray findings would
CONFIRM this diagnosis?
a) Loss of the left psoas shadow
b)
Upward
displacement
hemidiaphragm

of

the

left

c) Enlargement of the spleen


d) Fracture of the left lower ribs

Question 18
Jordan is 5 years old and has cerebral palsy. He has a
spastic hemiparesis. Which of the following features is
NOT seen with this type of abnormality?
a) Hypertonia
b) Fasciculations
c) Hyperreflexia

e) Absence of gas in the splenic flexure

Answer: A
The left psoas shadow may be obscured by
haemorrhage from a ruptured spleen. Lower rib
fractures provide evidence of local trauma but are not
diagnostic of splenic rupture.

d) Ankle clonus
e) Extensor plantar reflex
Answer: B
In spastic cerebral palsy, the motor features are of an
upper motor neuron abnormality. Fasciculations are
seen with lower motor neuron dysfunction. The other
options are correct.

Question 21
Jenny and Sam are beside themselves with worry. Their
son, Justin, is 11 years old and in grade 6 at school.
They are finding it increasingly difficult to encourage
Justin to go to school. He frequently complains of being
unwell in the mornings, with recurrent stomachache and
headache. His symptoms tend to be less apparent at
weekends and during school holidays, and often improve
as the day goes on when he is allowed to stay at home.

This pattern of behaviour has been typical for Justin for


some years, and he has been repeatedly examined and
investigated. No organic cause for his symptoms has
been diagnosed. He denies hallucinations. He performs
well at school, and his teacher says he seems to get
along well with his peers, with no evidence of bullying.
Which of the following is the MOST likely cause of
Justin's school refusal?

Answer: A
Greg is likely to have an acquired autoimmune
haemolytic anaemia, with the production of cold
agglutinins in response to the EBV. This causes a
normochromic normocytic anaemia, as well as the other
features listed. Hypochromic microcytic anaemia is
associated with iron deficiency.

a) Antisocial personality disorder


b) Gender identity disorder

Question 24

c) Separation anxiety disorder

You are discussing the genetic inheritance of a condition


with a family. Peter and Simone have just learned that
their son, Paul, has Duchenne muscular dystrophy.
Which statement is TRUE regarding the mode of
inheritance of this condition?

d) Early onset schizophrenia


e) Panic disorder

a) Autosomal dominant
Answer: C

b) X-linked recessive

Justin's
presentation
most
likely
represents
developmentally inappropriate separation anxiety. The
absence of a realistic cause for his fear of school and the
tendency for his symptoms to abate when allowed to
remain at home, as well as the absence of any organic
disease or positive psychotic symptoms (such as voices)
support this.

c) Autosomal recessive
d) Chromosomal deletion
e) Chromosomal translocation

Answer: B
Question 22
Alexander is 19 months old. He has not received any
childhood
immunisations
as
his
parents
are
conscientious objectors to vaccination. Alexander has
been unwell for several hours with fever, irritability and
dysphagia. He presents drooling and leaning forward
and is obviously distressed. What is the MOST LIKELY
organism to be causing Alexander's condition?

Duchenne muscular dystrophy is a sex-linked disorder,


inherited via an abnormal recessive gene on the X
chromosome. Thus, only males are affected. Unaffected
female carriers transmit the disease. For children of a
carrier female, there is a 50% chance that each
daughter will be a carrier, and a 50% chance that each
son will have the disease. Affected males cannot
transmit the disease to their sons, but all of their
daughters are carriers.

a) Streptococcus pneumoniae
b) Haemophilus influenzae type B

Question 25

c) Non-typable Haemophilus influenzae

Ngaire is a six week old baby girl. Her parents have


noted a swelling in the right side of her neck. She seems
well, is breastfed and is gaining weight. Which of the
following should NOT be considered in the differential
diagnosis of a lateral cervical swelling?

d) Streptococcus pyogenes
e) Staphylococcus aureus

a) Acute cervical adenitis


Answer: B

b) Branchial cleft cyst

Alexander has classic epiglottitis. All of the listed


organisms are known to cause epiglottitis. However,
Haemophilus influenzae type B is almost always the
causative organism in an unimmunised patient.

c) Cystic hygroma

Question 23
Greg is 7 years old. He has recently had confirmed
glandular fever (Epstein Barr virus). Over the past 3
days he has developed weakness, pallor, dark urine and
fatigue. On examination, he is jaundiced and has mild
splenomegaly. Which of the following is NOT a feature
you would expect to find on laboratory investigation?

d) Sternocleidomastoid muscle haematoma


e) Thyroglossal duct cyst

Answer: E
Thyroglossal duct cyst should not be considered in the
differential diagnosis of a lateral cervical swelling. This
congenital abnormality occurs in the midline, between
the hyoid bone and the suprasternal notch. The other
options may present as a lateral cervical swelling.

a) Hypochromic microcytic anaemia


b) Spherocytes

Question 26

c) Nucleated red blood cells

Cindy is 3 years old. She celebrated her birthday 3 days


ago, and has been wearing her favourite present, a new
necklace. Cindy has now developed a rash on her neck
and anterior chest where the necklace rests against her
skin. The rash is slightly raised, erythematous and itchy.

d) Hyperbilirubinaemia
e) Elevated LDH

Which of the following statements best describes the


MOST LIKELY explanation for this?
a) Type I hypersensitivity
b) Type II hypersensitivity
c) Type III hypersensitivity
d) Type IV hypersensitivity
e) The rash is unlikely to be related to the
necklace

Answer: D
The rash is most probably a contact dermatitis brought
on by the presence of the necklace. The most likely
cause of Cindy's rash is a Type IV hypersensitivity
reaction.
This is a T-cell mediated immune response, and is
usually a delayed response to a specific antigen.

Question 27
Rebecca's mother contracted rubella whilst she was
pregnant, which was confirmed by serological testing. As
a result Rebecca has been born with congenital rubella
syndrome. Which of the following is NOT a feature of
congenital rubella syndrome?

Answer: C
In the previously healthy individual, thrombocytosis
follows splenectomy. Anticoagulant treatment is
required if the platelet count exceeds 10,000x 10*9/L to
prevent thrombotic complications. All of the other
statements are true. Pulmonary dysfunction may ensue
because of trauma to ribs, the sub-diaphragmatic area
and the lungs, from the accident. Post-splenectomy
there is a greater susceptibility to overwhelming
bacteraemia because of decreased bacterial clearance,
levels of IgM and opsonisation of encapsulated bacteria.
Pneumovax should be given pre-operatively if possible
and immunisation to Haemophilus influenzae type B and
Meningococcus should be updated. The pancreas may be
damaged at the time of surgery, which could result in a
pancreatic cyst or fistula. This may become infected
giving rise to the subphrenic abscess.

Question 29
Stefan is 7 years old. He has been diagnosed with
Attention Deficit Hyperactivity Disorder (ADHD). After
careful consideration and evaluation, it has been decided
to commence him on pharmacological therapy. Which of
the following drugs is NOT used in the treatment of
ADHD?
a) Dexamphetamine
b) Methylphenidate
c) Imipramine

a) Growth retardation

d) Clonidine

b) Deafness

e) Diazepam

c) Limb hypoplasia
d) Cardiac defects

Answer: E

e) Mental retardation

Answer: C
Congenital infection with rubella causes a number of
manifestations,
the
main
ones
being:
mental
retardation, cardiac anomalies, growth retardation,
ocular anomalies, deafness, cerebral disorders including
chronic encephalitis, and haematologic disorders. Limb
hypoplasia is not usually seen. This may be a feature of
congenital varicella zoster infection.

Question 28
Eloise, aged 16, has been involved in a serious motor
vehicle accident in which she has suffered a ruptured
spleen. Unfortunately her clinical signs deteriorate, and
splenectomy is indicated. In obtaining informed consent
from Eloise and her parents, you discuss possible postoperative complications. Of the following statements,
each is true EXCEPT
a) There is
dysfunction

possibility

of

pulmonary

Question 30
Sally is 6 years old. She is being admitted to hospital for
tonsillectomy and adenoidectomy. Informed consent is
being sought from Sally's parents, and the risk of
haemorrhage and the possibility of a blood transfusion
being required are discussed. Sally's parents wish to
know the risk of Sally contracting hepatitis C via blood
transfusion. The risk of transmission of hepatitis C via
blood transfusion is:
a) 1:100
b) 1:1,000
c) 1:10,000

b) Pancreatic damage may occur at operation


c) Thrombocytopoenia may
requiring platelet transfusion

Diazepam is not indicated in the treatment of Attention


Deficit Hyperactivity Disorder (ADHD). Dexamphetamine
and methylphenidate are psychostimulants and may be
considered as first line therapy. Imipramine is a tricyclic
antidepressant which may be considered as second-line
therapy only if psychostimulants are not tolerated, are
contraindicated or are ineffective. Clonidine is widely
used, although its effectiveness remains unproven.

follow

surgery,

d) 1:100,000
e) 1:1,000,000

d) Post splenectomy bacterial infection may be


life-threatening

Answer: E

e) A subphrenic abscess may be a later


complication

All donor blood products are screened for hepatitis C


antibody. The estimated risks for false-negative tests for

hepatitis C on donor blood, and thus the risk of hepatitis


C are approximately 1:1,000,000. The Australian Red
Cross Blood Service tests all donations for HCV antibody
as well as nucleic acid PCR so the current estimate of
risk is 1:3,663,000.

Question 31
Paulette, a curious toddler aged 18 months is playing
happily with Grandma's sewing box. As her mother
approaches, Paulette swallows something she has had in
her mouth. Grandma keeps a variety of objects in this
box. Which of the following objects is MOST likely to
require active intervention if Paulette has swallowed it?
a) A two dollar coin
b) A duck shaped button
c) A hearing aid disc battery

Answer: B
Daniel almost certainly has an intussusception, which is
most common in children 3 months to 3 years of age
and typically presents with the features described
above. Air enema is an appropriate investigation and in
ileocolic intussusception, often resolves the abnormality
with less than 10% chance of recurrence. If
unsuccessful, urgent surgery is necessary. In infants
presenting with abdominal pain the differential diagnosis
will include colic, gastroenteritis, constipation, urinary
tract infection, volvulus and incarcerated hernia.
The suspicion of a surgical condition increases if there is
severe or increasing pain, bile-stained vomitus, guarding
or abdominal rigidity and distension. The passage of
blood and mucus producing th so-called "red-currant
jelly" is often a late feature of ischaemic bowel in either
intussusception or volvulus. Appendicitis is rare in
infants but in young children there is up to an 80% risk
of perforation at the time of diagnosis

d) A safety pin (probably closed)


e) A small marking pencil 3 cm long

Answer: C
The corrosive nature of the battery can lead to
gastrointestinal erosion and/or perforation. This is a
particular risk if the battery becomes lodged in the
oesophagus as erosion can begin within six hours of
ingestion. The majority of disc batteries, however, do
pass uneventfully through the remainder of gut if they
pass the level of the lower oesophageal sphincter. Coins
and round buttons generally pass through the gut
uneventfully .in a child., as do long items less than 6 cm
There is a very small risk of perforation with sharp
objects such as irregularly shaped buttons, or small
toys, but the majority of these do not require active
intervention. Even open safety pins will usually pass
through the gut without harm.

Question 32
Daniel, a previously well 10 month old infant, has been
severely distressed and screaming intermittently for the
last 1-2 hours. The screaming bouts last for 2-3
minutes, and recur every 10- 15 minutes although his
Mum says they are becoming more frequent. He draws
his legs up when he screams appearing to be in pain,
and is very flat and lethargic when he is not screaming.
He has vomited three times in the last hour, the last
vomit being a greenish fluid. What is the MOST
appropriate advice to give his worried mother?
a) Reassurance that colic is a common
condition which may respond to simple
treatments and H2 receptor antagonists
b) An air enema is likely to be both diagnostic
and effective as a treatment in this case
c) Admission will be necessary to provide
supportive treatment of this severe form of
gastroenteritis
d) Treatment includes analgesia and IV fluids
but avoidance of invasive procedures unless
there is per-rectal blood and mucus
e) Urgent surgery is necessary due to the
higher risk of appendiceal rupture in this age
group

Question 33
Tasneem has just been born with symptomatic
congenital cytomegalovirus infection (CMV). Which of
the following is the MOST COMMON finding in this
condition??
a) Hepatosplenomegaly
b) Microcephaly
c) Deafness
d) Chorioretinitis
e) Cataracts

Answer: A
CMV is the world's leading cause of congenital virus
infection. Approximately 1% of all infants are
congenitally infected and CMV infection is symptomatic
in 5-10% of these infants. Of symptomatic neonates,
60-80% will have hepatosplenomegaly, petechiae and
jaundice; 30-50% microcephaly, intra-uterine growth
retardation or prematurity; and the mortality is 20-30%.
Survivors have a 90% chance of going on
to develop sensorineural deafness, mental retardation,
seizures and motor delay. Asymptomatic neonates may
later develop microcephaly and psychomotor retardation
with up to 25% becoming deaf. Chorioretinitis occurs in
20% of symptomatic neonates but cataracts are
associated with congenital rubella infection, not CMV.
Since 50-60% of women of childbearing age are carriers
of CMV, then 40-50% of women are at risk of primary
infection in pregnancy which may be transmitted
transplacentally in up to 40%. For carriers, the risk of
reactivation in pregnancy is estimated to be anywhere
from one to 25% but risk of transmission to the foetus
in this case is only 2-5% and is nearly always
asymptomatic in the neonate

Question 34
Jennifer, age 25 years is concerned as she has been
trying to fall pregnant for the last three months and has
been unsuccessful. In giving her advice, which of the
following statements is CORRECT?
a) Pregnancy occurs in 80% of couples after 12
cycles; in those who have not conceived the
chance is 50% in each year thereafter

b) Healthy young couples have a 10% chance


of achieving pregnancy in one menstrual cycle,
that is, a fecundability of 0.10
c) If a couple less than 30 years of age have
not conceived in 12 months, they have no
chance of conception
d) With a regular cycle most women of this age
have conceived by 3 months so gynaecological
review is recommended
e) Preliminary investigations to confirm
ovulation and adequate sperm should now be
undertaken by her general practitioner

Answer: A
Infertility is defined as the failure of conception in a
couple having regular, unprotected sexual intercourse
for one year, provided that it is occurring not less than
twice weekly, and menstrual cycles are regular.80% of
couples will be pregnant after 12 cycles. Of those who
have not conceived after 12 cycles, about 50% will
conceive during a second year of attempted conception.
After this second year of attempted conception, the
chance of conception in those couples remaining is
about 50% in the following four years. The fecundability
of a young couple is approximately 0.25, that is, there is
a 20-25% chance of pregnancy for each menstrual
cycle. Fecundability for women declines from about 35
years of age, precipitously after age 40 Young women of
this age with this presenting complaint should have a
thorough history taken to exclude obvious causes of
infertility and to reassure the woman that she is normal.
It is important to clarify that coitus is achieving
adequate penetration to optimise sperm delivery, and
that timing matches the peak fertile period during her
menstrual cycle. Pre-pregnancy counselling will include
folate supplementation, rubella immunity and Pap smear
testing. Investigation of the couple is usually deferred
until 12 months have passed unless the woman is of
advanced age.

Question 35
Monique, age 18 years, keeps forgetting to take her
combined oral contraceptive pill and is keen to try depot
medroxyprogesterone, as her friend is using it with
great success. All of the following are side effects of this
medication EXCEPT:

effects. There is some debate as to the significance of


the slight fall in bone density seen in some studies of
women using this form of contraception. Thrombosis is a
side effect of oestrogens rather than progestogens.
Question 36
Malka, age 16 years, presents with primary
dysmenorrhoea which is preventing her from attending
school when she has her period. With regards to the
management of primary dysmenorrhoea which of the
following is TRUE?
a) Paracetomol is as effective as non-steroidal
anti inflammatory drugs
b) Combined oral contraceptives should not be
offered unless the young woman is sexually
active
c) Non steroidal anti inflammatory drugs should
be recommended as first line agents
d) Depo provera is of no assistance in the
management of primary dysmenorrhoea
e) Combined oral contraceptives should not be
used in conjunction with non-steroidal anti
inflammatories

Answer: C
Primary
dysmenorrhoea
results
from
uterine
vasoconstriction, anoxia, and contractions mediated by
prostaglandins. Non steroidal anti inflammatory drugs
(NSAIDS- eg: Naprosyn (mefanemic acid) are
recommended as first line agents in the management of
dysmenorrhoea as they inhibit prostaglandin synthesis
and therefore provide symptomatic relief. Paracetomol is
not as effective as NSAIDS in this condition due to it's
lack of effect on prostaglandin pathways. While it
provides simple analgesia it does not address the
causation of the problem. The combined oral
contraceptive pill reduces menstrual flow and inhibits
ovulation and is also effective in the treatment of
dysmenorrhoea. It can be used with NSAIDS, although
this is rarely necessary.. The lack of response to NSAIDs
and OCs (or the combination) may increase the
likelihood of a secondary cause for dysmenorrhoea.
Depo provera may also prove helpful in the
management of dysmenorrhoea as it inhibits ovulation
and often results in amenorrhoea.

a) Weight gain
b) Depression
c) Thrombosis
d) Amenorrhoea
e) Delayed fertility

Answer: C
Depot medroxyprogesterone acetate, marketed as
Depo-Provera and Depo-Ralovera, is an injectable
progestogen given every 12 weeks +/- 2 weeks. Women
can suffer from progestogenic side effects such as
weight gain, bloating, and menstrual disturbance.
Initially women may suffer from irregular bleeding,
however amenorrhoea occurs in 50% of women after 1
year of injections. There is usually a delay in the return
of fertility until approximately 9 months after the last
dose. Mood changes and acne are less common side-

Question 37
Maria, G3P3, 35 years is 4 weeks postpartum and
breastfeeding. She seeks advice regarding postpartum
contraception. Which of the following isTRUE?
a) Maria should not recommence sexual activity
until at least six weeks postpartum
b) Lactational amenorrhoea is 98% effective for
the first 6 months if fully breastfeeding (no
solids) and amenorrhoeic
c) Any of the oral contraceptive preparations
may be safely used after 6 weeks post-partum
d) Intra-uterine devices (IUDs) should not be
used until the uterus is back to normal at 6
months post-partum
e) Implantable progesterone-only contraception
(Implanon) is contra-indicated in breastfeeding
women

Answer: B
Sexual activity can recommence whenever the woman
feels comfortable enough to try it, taking into
consideration the time needed for episiotomies and
vaginal tears to heal. In terms of contraception, as long
as the woman is fully breastfeeding (no solids) and has
not started menstruating then lactational amenorrhoea
provides
98%
protection
against
pregnancy.
Contraception containing estrogen such as the COCP is
not recommended as it may reduce the volume of breast
milk. Progestogen only methods of contraceptives are
usually recommended as they are safe when
breastfeeding. These include the "minipill", depot
injections, Implanon or the Mirena IUD, all of which may
begin from 6 weeks post-partum. IUDs are safe to insert
6 weeks after a vaginal birth when the uterus has
returned to normal size and bleeding associated with the
birth has stopped. IUDs should be inserted at least 12
weeks after a caesarean birth as there is a slightly
greater risk of perforation of the uterus before this.

Question 38
Sally has come to discuss the combined oral
contraceptive pill (COCP) with you. Which of the
following is TRUE regarding some of the noncontraceptive effects of the COCP? It:
a) Increases the risk of benign ovarian cysts
b) Reduces the risk of endometrial cancer
c) Has no effect on the risk of colorectal cancer
d) Increases the risk of pelvic inflammatory
disease
e) Reduces the risk of invasive breast cancer

Answer: B
The COCP has little effect on benign ovarian cysts. If
anything, they may reduce in size. Ovarian cancer: The
longer the use of the pill, the greater the protection.
After more than 10 years on the pill, the risk of ovarian
cancer may be reduced by 60- 80 %. This protective
effect appears to last at least 15 years after stopping
the pill. Researchers theorize that the pill may lessen
ovarian cancer risk in part by inhibiting ovulation.
Endometrial (uterine) cancer: In women using the pill
the risk may be reduced by as much as 50%. This
protection also lasts at least 15 years after the pill is
stopped. Colorectal cancer: On the pill the risk may be
reduced by approximately 35%. Researchers believe the
protective effect may be due to a reduction in the
concentration of bile acids in the colon. Latest research
suggests that the COCP has no effect on the risk of
breast cancer. Pelvic inflammatory disease (PID):
Although rarely fatal, this infection can lead to infertility
from scarring of the fallopian tubes.
The pill cuts the risk of PID by half. It is believed the pill
works by making cervical mucus an unfriendly
environment for disease-causing bacteria.

Question 39
Mandy has requested you insert a copper-bearing IUD.
She is 34 years of age, with regular 28 day menstrual
cycles and she is in a stable monogamous relationship.
Mandy wants to know when, during her next menstrual
cycle she should have the IUD inserted. Which of the

following is the CORRECT advice to


regarding timing of insertion of an IUD?

give

Mandy

a) An IUD can be inserted any time during the


last 14 days of her menstrual cycle and no
additional contraceptive protection is needed
b) An IUD should only be inserted whilst Mandy
has menstrual bleeding and no additional
contraceptive protection is needed
c) An IUD can be inserted any time within the
first 12 days after the start of menstrual
bleeding and no additional contraceptive
protection is needed
d) An IUD can be inserted at any stage within
the middle two weeks (14 days) of her
menstrual cycle and no additional contraceptive
protection is needed
e) An IUD can be inserted any time during her
menstrual cycle provided she uses 7 days of
additional
contraceptive
protection
after
insertion

Answer: C
A copper-bearing IUD can be inserted any time within
the first 12 days (Day 1 = the first day of menstrual
loss) after the start of menstrual bleeding. No additional
contraceptive protection is needed. The probability of an
existing pregnancy is extremely low before day 12 of the
menstrual cycle based on the extremely low risk of
ovulation before day 8, plus the copper-bearing IUDs
provide 5-days of emergency contraceptive effect. These
recommendations do not apply to hormonal
IUDs as their emergency contraceptive effects if any,
are unknown.

Question 40
Beth wants to start using DMPA as a form of
contraceptive. Which of the following statements
regarding Depot Medroxyprogesterone Acetate (DepoProvera, Depo-Ralovera, depot MPA, DMPA) is TRUE?
a) DMPA is given as a subcutaneous injection
b) DMPA always makes women amenorrhoeic
c) DMPA causes a delay in return of fertility
d) Prolonged DMPA use increases the risk of
endometriosis
e) Prolonged DMPA use increases the risk of
ovarian cancer

Answer: C
Depot Medroxyprogesterone Acetate (Depo-Provera or
depot MPA or DMPA) is a long-acting hormonal
contraceptive which is usually given as an intramuscular
injection once every three months. Disadvantages of
DMPA include changes in the menstrual cycle during
treatment, usually resulting in oligomenorrhoea or
amenorrhoea, but sometimes resulting in troublesome,
irregular bleeding, especially in the first few months of
use. DMPA can also cause an unpredictable but
temporary delay in return of fertility following
treatment. There is increasing evidence for substantial
protection against endometrial cancer, ovarian cancer,

acute pelvic inflammatory disease and endometriosis.


There is no evidence for serious long-term complications
of treatment with DMPA, although there is current
controversy about the possible small risk of increased

bone loss in women using DMPA long-term, who have


continuous amenorrhoea and low serum oestradiol
levels.

Block 10
Question 1
Prolonged neurological sequelae in infants can occur if
the mother has taken which of the following in the third
trimester?

orbicularis oculi). In isolation, the dystonias are usually


of unknown cause and treatment is difficult. Dystonias
as a reaction to medication are much more common in
children, for example, metoclopramide (an anti-emetic)
results in an acute dystonic reaction in 1% of adults but
10% of children less than 10 years of age.

a) Antipsychotics
b) Tricyclic antidepressants
c) Anticonvulsants
d) Benzodiazepines
e) Opioids

Question 3
The interossei of the hand are supplied by:
a) The radial nerve
b) The median nerve
c) The ulnar nerve

Answer: A
When given in high doses in late pregnancy,
antipsychotic agents have caused prolonged neurological
disturbances in the newborn infant. Anticonvulsants are
associated with congenital defects rather than
neurological sequelae. Benzodiazepines can cause
hypotonia, respiratory depression and hypothermia in
the newborn infant if used during labour in high doses.
Withdrawal symptoms in neonates have been reported
with prolonged maternal use of tricyclic antidepressants.
Opioid analgesics may cause respiratory depression in
the newborn infant. Withdrawal symptoms in the
newborn have been reported with prolonged use of
opioids.

d) All of the above


e) None of the above

Answer: C
The nerve supply of the interossei of the hand is from
the deep branch of the ulnar nerve (spinal root C8-T1).

Question 4
Coral is 55 years old. Her periods ceased 8 years ago.
She now presents with vaginal itch, discharge and
soreness. The MOST LIKELY cause is:

Question 2

a) Candida infection

A 21 year old man walks into your surgery with his head
tilted sideways, his eyes rolled up and his tongue
sticking out. He speaks with difficulty but says that he
has been 'stuck' in this position since taking a new
medicine a few hours ago 'for his nerves'. You should
administer:

b) Vaginal atrophy
c) Vaginal dermatoses
d) Gardnerella infection
e) None of the above

a) Diazepam
b) Benztropine
c) Chlorpromazine
d) Phenytoin
e) Haloperidol

Answer: B
50% of women suffer symptoms of urogenital atrophy in
the postmenopausal years. Atrophy of the vaginal
mucosa can lead to vaginal dryness, soreness, pruritus
and discharge.

Answer: B

Question 5

The presentation described is an acute dystonic


reaction, which involves a spasmodic torticollis, (where
the head is pulled and held to the left or right by one or
other sternomastoid), upward drawn eyes and an open
mouth (oromandibular dystonia). It may occur
(particularly in young men) within a few days of starting
a neuroleptic medication. Treatment is with the
anticholinergic
medication
eg.
Benztropine
or
diphenhydramine. If the patient has a contraindication
to anticholinergics then diazepam may be helpful. The
dystonias are a group of disorders involving prolonged
spasms of muscle contraction. Spasmodic torticollis is
one type, as is trismus (clenched jaw) and
Blepharospasm (involuntary contraction of the

22 year old Melanie presents with a 2 day history of a


vaginal discharge, urinary frequency and dyspareunia.
Which organism is the MOST LIKELY cause of this
presentation?
a) Gardnerella
b) HIV
c) Chlamydia trachomatis
d) Giardia lamblia
e) Hepatitis B

Answer: C
Chlamydia trachomatis is an intracellular bacterium.
Genitourinary symptoms caused by this organism
include cervical discharge, cervical bleeding, menstrual
change, abdominal pain, fever, nausea, vomiting,
urinary frequency and dysuria. Infection rates are
highest in the 15-25 year old age group. It is important
to remember Chlamydia may be initially asymptomatic
especially in women and is the leading cause of
preventable infertility and ectopic pregnancy. Although it
is important to test for HIV, Melanie's symptoms are not
consistent with a seroconversion type illness. Hepatitis B
also does not present with gynaecological symptoms.
Gardnerella is a simple vaginal infection with no clinical
sequelae. Giardia is a single celled organism which
infects the small intestine.
Question 6
Mr Brown is 70 years old and suffers with severe pain in
his back from osteoarthritis. He also suffers with
emphysema and has had prednisolone for exacerbations
of this disease. He is currently on warfarin for a cardiac
arrhythmia and is also taking lansoprazole as a
maintenance dose after a duodenal ulcer was diagnosed
4 years ago. Which of the following DOES NOT increase
the risk of gastrointestinal side effects from nonsteroidal anti-inflammatory drugs (NSAIDs)?
a) Concurrent use of prednisolone
b) Previous duodenal ulcer
c) Concurrent use of anticoagulant
d) The fact that he is over 70 years of age
e) Male gender

Answer: E
Risk factors for gastro-intestinal side-effects of NSAIDs
include age greater than 65 years, previous history of
peptic ulcer and co-administration of prednisolone and
anti-coagulants. Male gender is not a risk factor.

tightening of the skin with increased risk of the


development of squamous cell carcinoma.

Question 8
Which of the following is INCORRECT?
a) The incidence
Australia is 1:200

of

haemochromatosis

in

b) C282Y homozygotes account for more than


90% of haemochromatosis in Australia
c) The majority of patients with one copy each
of the C282Y and H63D mutation never develop
haemochromatosis
d) 90% of C282Y homozygotes develop
symptoms of the disorder at some stage in
their lives
e) Carriers of one copy of the altered gene are
generally healthy

Answer: D
Studies estimate that up to 50% of C282Y homozygotes
will remain symptom free throughout life.

Question 9
You are called to see a 78 year old woman with a threeweek history of headaches and depressive symptoms.
She relates that 24 hours ago the vision in her right eye
suddenly dimmed. Today the visual acuity in the eye is
limited to perception of hand movements only,
compared with 6/6 in the left eye. She is not known to
be a diabetic and her blood pressure is only minimally
elevated. Which of the following possible causes of her
visual loss require IMMEDIATE investigation and
treatment to prevent blindness in the other eye?
a) Detached retina
b) Central retinal artery occlusion

Question 7

c) Central vein occlusion

A 55 year old woman presents with an itchy purple


papular rash on her wrist. She is not on any
medications. This is MOST LIKELY to be:

d) Acute glaucoma
e) Temporal arteritis

a) Lichen Planus
b) Lichen Simplex Chronicus
c) Lichenoid eruption
d) Lichen Sclerosus
e) Lichenification

Answer: A
Lichen Planus is an epidermal inflammatory disorder of
unknown cause characterised by pruritic, violaceous, flat
tipped papules mainly on the wrists and legs. Lichen
Simplex Chronicus is a thickening of the skin as a result
of constant scratching, the process of which is called
lichenification. Lichen sclerosus is an unexplained
vaginal
dystrophy
characterised
by
itch
and
hypertrophy. A lichenoid eruption is most typically due
to a drug reaction. Lichen Sclerosis et Atrophicus is a
condition of the penis and vulva producing atrophy and

Answer: E
Temporal arteritis is an uncommon disease of the elderly
and is characterised by the classic complex of fever,
anaemia, high ESR and headaches in an elderly person.
It is closely associated with polymyalgia rheumatica.
Temporal arteritis is the most common manifestation of
a systemic vasculitis. Headache is the predominant
symptom and may be associated with a thickened or
nodular artery. A serious complication, as described in
this lady, is ocular involvement - ischaemic optic
neuritis. Most patients have head or eye symptoms for
months before objective eye involvement. Acute
glaucoma causes a red, painful eye, reduced vision and
a fixed, mid- dilated pupil which may be slightly ovoid.
The pain may be severe and associated with nausea and
vomiting. Acute glaucoma may be preceded by blurred
vision or haloes around lights. It is a uniocular attack
due to blockage of drainage of aqueous fluid from the
anterior chamber via the canal of Schlemm. Urgent
treatment with hyperosmotic agents is necessary to
reduce the intraocular pressure.

Question 10
A 21 year old female patient presents with lower
abdominal pain and tenderness at 14 weeks of
gestation. Her temperature is 38.5 degrees Celsius. The
most important diagnosis to EXCLUDE is:
a) Pyelonephritis
b) Threatened abortion
c) Ectopic pregnancy
d) Degeneration of a uterine fibroid
e) Appendicitis

Answer: E
Appendicitis is the commonest surgical emergency and
has its maximum incidence between 20 and 30 years of
age. In pregnancy it occurs mainly in the second
trimester. Pain is generally higher and more lateral than
typical
appendicitis.
Ectopic
pregnancy
occurs
approximately one in every 100 clinically recognised
pregnancies. The classical triad of ectopic pregnancy
includes amenorrhoea (65-80%), lower abdominal pain
(95+%) and abnormal vaginal bleeding (65-85%).
Degeneration of a uterine fibromyoma typically occurs in
the second trimester of pregnancy and is due to
ischaemic necrosis. In threatened abortion there is
vaginal bleeding. Pain is usually not a significant feature
unless the cervix is beginning to open. Pyelonephritis
can mimic acute appendicitis in pregnancy.

Question 12
An elderly patient with rheumatoid arthritis has been on
prednisolone in a dose of 12.5 mg daily for 8 years.
After a fall, she complains of severe pain over her lower
thoracic spine and is found to be very tender over T11
and T12. A radiograph shows 'cod-fish' vertebrae, with
generalised radiolucency of bones and collapse of two
vertebral bodies. Examination of her serum is MOST
LIKELY to show:
a) High calcium, low phosphate, raised alkaline
phosphatase
b) Low calcium, high
alkaline phosphatase

phosphate,

normal

c) Low calcium, low phosphate, raised alkaline


phosphatase
d) Low calcium, normal phosphate, normal
alkaline phosphatase
e) Normal calcium, normal phosphate, normal
alkaline phosphatise

Answer: E
It is most likely the patient has steroid-induced
osteoporosis. Plasma calcium, phosphate and alkaline
phosphatase are all normal in osteoporosis. If there are
multiple recent fractures, serum alkaline phosphatase
may be elevated.

Question 13
Question 11
A 42 year old man presents with a recurrent severe
hemicranial nocturnal headache which lasts for 60
minutes and occurs regularly every 3 weeks. The
headache is accompanied by a blocked nose and
watering eye. The MOST LIKELY diagnosis is:
a) Chronic paroxysmal hemicrania
b) Migraine variant headache

Which of the following may be a good reason for


stopping breast feeding:
a) Inverted nipples
b) Cracked nipples
c) Engorged breasts
d) Mastitis
e) Breast abscess

c) Chronic sinusitis
d) Cluster headache

Answer: E

e) Trigeminal neuralgia

Frequent, unrestricted breast feeding day and night is a


treatment for breast engorgement. Improved breast
drainage is important in mastitis and breastfeeding
should not be stopped. Breastfeeding can be continued
in the presence of cracked and inverted nipples.
Temporary stoppage of breastfeeding on the affected
side is necessary while surgical drainage of a breast
abscess is carried out.

Answer: D
Cluster headache has a four-fold higher incidence in
men than women. It is characterised by constant
unilateral orbital pain, with onset usually within 2-3
hours of falling asleep. The pain is intense and steady
with lacrimation, blocked nostril then rhinorrhoea and
sometimes miosis, ptosis, flush and oedema of the
cheek, all lasting approximately an hour or two. It tends
to occur nightly for several weeks or a few months,
followed by complete freedom for months or even years.
The response to inhaled oxygen can be dramatic.
Chronic paroxysmal hemicrania is similar to cluster
headache in presentation except that it is more common
in women, attacks occur at any time day or night lasting
2-25 minutes and it always responds to indomethacin.
Migraine variants include a range of presentations that
are more common in children and often go on to
develop a more typical migraine presentation in
adulthood.

Question 14
A 40 year old physician has had muscle twitching for
one year. The twitching occurs predominantly in the
lower extremities. A neurological and general physical
examination is negative except for these twitchings.
What is the MOST LIKELY diagnosis?
a) Benign fasciculations
b) Hypoparathyroidism
c) Amyotrophic lateral sclerosis
d) Paramyoclonus multiplex
e) Dystrophia myotonica

Answer: A
Diseases of motor neurons or their proximal axons are
often associated with fasciculations, the spontaneous
firing of an entire motor unit. Fasciculations occur at
times in most normal individuals, and unless weakness
is present, are seldom of any significance. Myoclonus is
a descriptive term for very brief, involuntary, random
muscular contractions. Myoclonus may involve a single
motor unit and simulate a fasciculation, or it may
involve groups of muscles that displace the limb. In
Dystrophia Myotonica there is intellectual impairment,
amongst other associated deficits, and the weakness
starts with the face and extremities. Amyotrophic lateral
sclerosis (ALS) is characterised by a progressive loss of
motor neurons, both upper and lower motor neurons.
The first evidence is an insidiously developing
asymmetric
weakness.
Hypoparathyroidism
often
presents with neurologic involvement - manifesting as
tetany - eg. Chvostek's sign.

intercourse should take place prior to the temperature


rise. The purpose of having intercourse on alternate
days is to maximise the sperm count on each
ejaculation.

Question 17
Sudden onset of unilateral orbital pain, photophobia,
lacrimation and blepharospasm suggests a diagnosis of:
a) Open-angle glaucoma
b) Anterior uveitis
c) Temporal arteritis
d) Unilateral blepharitis
e) Vitreous haemorrhage

Answer: B

Question 15
Which of the following is of LEAST value in assessing the
level of active joint inflammation ('disease activity') in
rheumatoid arthritis?
a) Duration of morning stiffness
b) Presence of tiredness, malaise and weight
loss
c) Degree of elevation of the erythrocyte
sedimentation rate (E.S.R.)
d) Presence of rheumatoid factor in the blood
e) Strength of hand grip

Anterior uveitis (acute iritis) presents with pain of acute


onset,
photophobia,
blurred
vision,
lacrimation,
circumcorneal redness (ciliary congestion) and a small
pupil (initially from iris spasm). Talbot's test is positive:
pain increases as the eyes converge (and pupils
constrict). The slit lamp reveals white precipitates on the
back of the cornea and anterior chamber pus
(hypopyon). Open angle glaucoma is painless and
largely asymptomatic until there is visual field loss;
temporal arteritis causes pain in the temporal area, not
in the orbit; blepharitis is inflammation of the eyelids;
vitreous haemorrhage can present with visual field loss
depending on the size of the haemorrhage and it is
generally painless.

Question 18

Answer: D
Rheumatoid factor (RF) is useful in the diagnosis of
rheumatoid arthritis and is of prognostic significance;
however fluctuations in RF titre have poor correlation
with disease activity. Constitutional symptoms such as
tiredness, malaise, weight loss and morning stiffness are
good predictors of disease activity.

A patient presents with acute gout but has a low serum


uric acid. Which of the following drugs can be
responsible for a low serum uric acid?
a) Colchicine
b) Prednisolone
c) Indomethacin

Question 16

d) Naproxyn

In a couple trying to conceive, intercourse should take


place:

e) Salicylates

a) As soon
temperature

as

there

is

rise

in

basal

b) 14 days after the onset of the next period


c) On alternate
temperature rise
d) On alternate
temperature rise

days

prior

days

to

after

the
the

basal
basal

e) Daily after the basal temperature rise

Answer: C
Ovulation occurs on day 14 (plus or minus 2 days)
before the onset of the next menses. Basal body
temperature rises around the time of ovulation and
remains elevated for at least 10 days. By the time the
temperature rise is detected by the patient, they are
probably in the infertile phase of the menstrual cycle, so

Answer: E
Up to 30% of patients with acute gout have a normal
serum uric acid at presentation. This may be due to
precipitation into crystals in the synovium, an effect of a
drug the patient has taken for analgesia. Colchicine,
steroids and NSAIDS reduce inflammation in affected
joints but have no effect on serum uric acid levels.
Salicylates eg aspirin at doses less than 2g/day increase
serum uric acid levels, but at doses above 4g/day
reduce serum uric acid levels.

Question 19
The 'benign' jaundice that occasionally occurs in the
third trimester of pregnancy is most CLOSELY related
to:
a) Decreased circulating amounts of albumin to
bind bilirubin

b) A specific defect in glucuronide conjugation

Question 22

c) An autoimmune process

The serum phosphate is MOST LIKELY to be low in which


of the following disorders:

d) High oestrogen and progesterone levels


e) The use of diuretics in the third trimester of
pregnancy

a) Osteoporosis
b) Osteomalacia
c) Multiple myeloma

Answer: D

d) Polymyalgia rheumatica

In a small number of pregnant women, an intrahepatic


cholestasis may appear. It usually occurs in the third
trimester of pregnancy and presents as jaundice and
pruritus. These patients probably have an increased
susceptibility to the hepatic effects of oestrogenic and
progestational hormones.

e) Paget's disease

Question 20
Accommodation of
accompanied by:

the

eyes

to

NEAR

objects

is

Answer: B
Patients with osteomalacia almost always have
hypophosphataemia. Serum phosphate is normal in
osteoporosis,
Paget's
disease
and
polymyalgia
rheumatica (PMR). Multiple myeloma may have a
normal or slightly raised phosphate level.

a) Loss of binocular vision

Question 23

b) Constriction of the pupils

Which of the following is NOT TRUE regarding


asymptomatic bacteriuria in pregnant women?

c) Divergence of the visual axes


d) Decreased refractive (dioptric) power
e) Relaxation of the ciliary muscles

a) The incidence is 3-7% of all pregnancies


b) Defined arbitrarily as greater than 100,000
bacteria/ml
c) One of the common
Streptococcus Faecalis

Answer: B
Normal pupillary responses consist of prompt,
symmetric constriction (miosis) on exposure to light or
on attempted near convergence. Accommodation
involves the active changing of lens shape to focus near
objects. Light detection by the retina is passed to the
brain via the optic nerve and pupil constriction is
mediated by the third cranial nerve (parasympathetic).
The sympathetic nervous system is responsible for pupil
dilatation via the ciliary nerves.

Question 21
Which finding in this report of a cerebrospinal fluid
(C.S.F.) examination is INCONSISTENT with the other
results?
a) Elevated protein

organisms

is

d) Responds to a short course of oral antibiotics


e) Is a clinically unimportant phenomenon

Answer: E
Approximately 6% of women have asymptomatic
bacteriuria in pregnancy, where there are more than
100,000 bacteria per ml of urine in a mid-stream urine
sample. 30% of patients with asymptomatic bacteriuria
will subsequently develop acute pyelonephritis. There is
an association between pyelonephritis, low birth weight
and prematurity. Treatment is with an appropriate
course of antibiotics. Recurrence can occur in 35% of
cases. The predominant organisms in subclinical or
clinical urinary tract infections in pregnancy are E. coli,
Strep faecalis, Aerobacter, Klebsiella and Proteus.

b) Normal chloride
c) Elevated glucose

Question 24

d) No red cells

With regard to Tietze's syndrome, which of the following


statements is INCORRECT? Tietze's syndrome is:

e)
Gross
leucocytes

excess

of

polymorphonuclear

Answer: C
A lumbar puncture consistent with a diagnosis of
pyogenic meningitis contains excessive polymorphs,
with protein at 1.5g/l (aseptic less than 1.5g/l) and
glucose at less than 2/3 the plasma level. There are no
red cells unless it is a bloody tap (ie. artefact due to
rupture of blood vessel).

a) Painful, tender swelling of one or more


costochondral joints
b) The cause of up to 30% of chest pain
presentations to emergency departments
c) Most commonly diagnosed before the age of
40 years
d) Sometimes associated with the development
of rheumatoid arthritis later in life
e) Often associated with repeated minor chest
trauma such as harsh coughing

Answer: D

Question 27

The predominant feature of Tietze's syndrome (also


called costochondritis) is that of a painful costochondral
junction with a palpable swelling. It often occurs in
patients with persistent cough and is aggravated by
coughing or deep breathing. The 2nd costochondral
junction is most commonly affected. Women are more
often affected than men. The usual onset is before age
40 years. In the assessment of people presenting with
chest pain, 10-30% will have costochondritis, even in
childhood. Although many other conditions such as
rheumatoid arthritis may present with involvement of
the costochondral joints, there is no evidence that
Tietze's syndrome will progress to any other illness. It
usually follows a fluctuating course with eventual
resolution. Management might include analgesics, antiinflammatory drugs, local heat/ice, avoidance of triggers
and local steroid injection.

Marion aged 56 years complains of constipation,


tiredness, lethargy and lack of energy. She looks puffy
in the face, is faintly yellowish in colour and has a pulse
of 56 beats/minute regular. Her full blood count shows
her haemoglobin is 9.7 g/dl, she has a macrocytosis,
lymphocytosis and a raised erythrocyte sedimentation
rate (ESR). The MOST LIKELY diagnosis is:
a) Pernicious anaemia
b) Carcinoma of the bowel
c) Alcoholism with malnutrition
d) Hypothyroidism
e) Lymphocytic leukaemia

Answer: D
Question 25
A mother develops a fever of 38.2 degrees Celsius 3
days after the delivery of her baby. Which of the
following is the MOST LIKELY cause?
a) Endometritis
b) Dehydration
c) Breast engorgement
d) Deep venous thrombosis
e) Urinary tract infection (UTI)

Both pernicious anaemia and primary autoimmune


hypothyroidism can cause lethargy, a yellow tinge to the
skin, lymphocytosis and a macrocytic anaemia with a
raised ESR. They often occur simultaneously and are
more common in females. However constipation and
bradycardia, with puffiness of the face (myxoedema) are
more indicative of hypothyroidism, making this the
MOST likely diagnosis in this case. One would expect a
microcytic anaemia with carcinoma of the bowel due to
chronic occult bleeding and more signs of liver disease
with alcoholism and malnutrition. Lymphocytic leukemia
may cause lethargy, lymphocytosis and jaundice, but
none of the other signs.

Answer: A
Puerperal infection affects 2-8% of pregnant women and
presents with a temperature of greater than 38 degrees
Celsius after the first 24 hours post-partum. In up to
75% of cases the cause is genital tract infection, with
endometritis most common, especially in the context of
a prolonged, complicated labour. Dehydration and
breast engorgement may cause a mild pyrexia up to
37.5 degrees. Thrombosis may also cause a low-grade
temperature initially although a septic pelvic thrombophlebitis would present later with high fever. UTI occurs
in 2-4% of post-partum women and usually presents
with low grade fever unless there is pyelonephritis.

Question 26
In which spinal cord segments are the motor neurones
responsible for the knee-jerk located?
a) L1,L2
b) L2,L3
c) L3,L4
d) L4,L5
e) L5,S1

Answer: C
The four muscles of the anterior thigh join to form a
common tendon which inserts into the patella. These
muscles are supplied by the femoral nerve, spinal roots
L3, 4. The quadriceps tendon continues distal to the
patella as the patellar tendon and it is this tendon which
is stretched in the knee-jerk. Stretching the muscle
spindles causes a reflex quadriceps contraction to limit
the tendon stretch.

Question 28
In which of the following conditions of the colon is
malignant change MOST LIKELY to occur?
a) Adenomatous polyp
b) Melanosis coli
c) Diverticulitis
d) Familial polyposis coli
e) Ulcerative colitis

Answer: D
Familial Intestinal Polyposis occurs in 1/8000 to 1/14000
people in western countries and 50 percent have
hundreds to thousands of polyps by age 16. 90% of
affected people will develop carcinoma of the colon by
age 45. Between 10% and 20% of adenomatous polyps
show histological evidence of malignancy when
removed. The primary genetic defect occurs in the
apoptosis gene p52. The lifetime risk of malignant
change in untreated ulcerative colitis is 12%. Melanosis
coli is due to laxative abuse and is not pre-malignant in
itself. The association between carcinoma of the colon
and melanosis coli is probably due more to chronic
constipation rather than the staining of the bowel wall
from laxatives. Diverticulitis is not associated with
malignant change.

Question 29
A 25 year old epileptic woman who had been in good
health was admitted to hospital in a comatose condition
after the ingestion of 2.0 g of phenobarbitone. On

examination, most reflexes appeared to be intact and


there was no depression of respiration. Which of the
following forms of therapy would be MOST appropriate?
a) Urgent peritoneal dialysis

c) Chlamydia trachomatis
d) Neisseria gonorrhoeae
e) Streptococcus faecalis

b) Alkalinisation of urine

Answer: C

c) Prophylactic antibiotics

The most common aetiology of epididymitis in males


less than 35 years of age is sexually transmitted
diseases, and Chlamydia is much more common than
the gonococcus. E.coli can be a causative factor in
urinary tract infections especially in older men with
urinary
outflow
obstruction
(benign
prostatic
hypertrophy etc). Mumps orchitis and epididymitis are
now rare because of widespread vaccination.

d) Administer analeptics
e) Give IV corticosteroids

Answer: B
Phenobarbitone is a long-acting barbiturate. Peak
plasma levels occur within 2-4 hours. It exerts its effects
through depression of the central nervous system. It is a
weak acid and is 50% protein bound. About 75% of the
dose is metabolised and the other 25% excreted
unchanged by the kidneys. The half-life in over-dose is
80-120 hours. Initial management of barbiturate
overdose
requires
prompt
gastrointestinal
decontamination. Barbiturates are well absorbed by
activated charcoal. For all barbiturates, attention should
be given to hemodynamic and respiratory support,
correction of temperature and electrolyte derangement,
and monitoring for pulmonary complications. Renal
elimination of
phenobarbitone
is
enhanced by
alkalinisation of urine to pH 8 and fluid administration
with or without mannitol to enhance diuresis.

Question 30
A 32 year old man with ankylosing spondylitis presents
to you for management of his disease. Which of the
following clinical features is he LEAST LIKELY to
complain about?
a) Sudden development of dull low back pain
b) Attacks of pain and photophobia in one eye
c) General malaise, fatigue and weight loss
d) Morning stiffness lasting a few hours
e) Bilateral hip and shoulder pain

Answer: A
The key features of ankylosing spondylitis are the
insidious onset of back pain and morning stiffness which
improves with exercise, lasting for more than 3 months.
Back pain is the most common presenting symptom,
and it occurs predominantly in young adults.
Approximately 25-35% have an arthritis in the hips and
shoulders. Some also have an asymmetric arthritis of
other joints. Younger patients often present with a
peripheral enthesitis. Older patients may have more
constitutional symptoms such as fatigue. Up to 30% will
have episodes of acute anterior uveitis and many will
have other extra-articular features.

Question 32
Jane is brought into the surgery after being struck in the
eye with a tennis ball. On examination you note blood in
the anterior chamber of the eye. Which of the following
statements regarding her management is INCORRECT?
a)
Aspirin
inflammatories
contraindicated

and
non-steroidal
drugs
(NSAIDs)

antiare

b) The main treatment goal is prevention of


secondary haemorrhage and glaucoma
c) Topical mydriatics should be used to permit
examination of the posterior chamber
d) Tranexamic acid (Cyklokapron) may be used
to stabilise clot formation and prevent
rebleeding
e) Urgent ophthalmological review is necessary
to exclude other ocular damage

Answer: C
Management of hyphaema is directed at prevention of
secondary haemorrhage within the orbit which carries a
high risk of severe glaucoma. Therefore strict bed rest is
essential with both eyes covered to reduce eye
movements. Aspirin and NSAIDs are contraindicated.
Mydriatics would not be used due to the risk of causing
the iris to rebleed. An urgent ophthalmological
assessment is essential. Ongoing review will check for
secondary bleeding (20% occur within 2-3 days),
glaucoma and corneal staining. Tranexamic acid may be
used to stabilise the clot. Glaucoma may develop
months or years after the hyphaema has resolved.

Question 33
The MOST LIKELY venous source of fatal pulmonary
embolism is:
a) Iliofemoral
b) Subclavian
c) Saphenous

Question 31

d) Pelvic

In men less than 35 years of age the MOST COMMON


causative organism in epididymitis is:

e) Popliteal

a)Mumps virus
b) Escherichia coli

Answer: A
Most pulmonary emboli arise from proximal deep vein
thrombosis (deep veins of lower limb, pelvis and inferior

vena cava). Less frequently, thromboses of the upper


arm are the source. Saphenous vein thrombosis seldom
results in clinically obvious pulmonary embolism. Also,
in order for the thrombus to cause fatality, it has to be
large enough to either cause obstruction in the right
atrium or of the right ventricular outflow tract. It
therefore would have to originate in a large vein.

Question 34
A patient with moderate to severe emphysema would
demonstrate all of the following physical signs EXCEPT:
a) Distant heart sounds
b) A prominent pulmonary second sound
c) A narrow intercostal angle
d) Prolonged expiration
e) Hyperresonance to pulmonary percussion

diabetics will eventually require insulin therapy, the


early introduction of a small dose of intermediate-acting
insulin before bed will often markedly improve control
and ease anxiety about managing injections.

Question 36
In trigeminal neuralgia all of the following statements
are true EXCEPT:
a) It is most common in middle-aged and
elderly persons
b) Multiple sclerosis must be considered in
younger people
c) There is an associated weakness in the
muscles of mastication
d) The severe, lancinating pain may last for up
to two minutes
e) It usually
carbamazepine

responds

well

to

regular

Answer: C
Pulmonary emphysema is one of the obstructive types of
pulmonary diseases which leads to progressive
hyperinflation of the chest. As a result heart sounds
become distant, percussion becomes hyper-resonant
and the intercostal angle enlarges as the chest becomes
more barrel shaped. Pulmonary hypertension leads to a
louder second (P2) heart sound. Prolonged expiration or
a prolonged forced expiratory time is an important sign
in the obstructive airways group of diseases.

Question 35
Mrs Leung, aged 68 years, has Type 2 diabetes for
which she takes metformin 850mg tds and gliclazide
160mg bd. Her blood pressure is well controlled on
candesartan 16mg daily. She has no evidence of heart
failure and is otherwise well. Her HbA1c is now 9.2%
(nondiabetic range <6%). Which of the following
strategies would NOT be an appropriate next step?
a) Commence isophane insulin 10 units before
bed
b) Commence rosiglitazone 4mg daily
c) Refer for dietary and diabetes education to
review lifestyle factors
d) Increase her dose of metformin to 1 gram
tds
e) Increase her dose of gliclazide to 160mg tds

Answer: C
Trigeminal neuralgia is characterised by stabs of intense
pain in the distribution of one or more divisions of the
Trigeminal nerve, lasting 1-2 minutes (up to 15
minutes). It is usually unilateral (96%). Pain may be
precipitated by touching a trigger point on the face or by
eating or talking. It is more common in women than
men and in those over 50 years. The cause is unknown.
In younger patients multiple sclerosis should be
considered. If left untreated it can progress, with shorter
and shorter periods of remission. Carbamazepine is
usually the first line of pharmacological treatment.
Surgical treatment may be necessary if it persists
despite a trial of other medications, but may cause
permanent anaesthesia. The motor division of the
trigeminal nerve supplies the pterygoid and masseter
muscles. It is not usually affected by trigeminal
neuralgia however surgical treatments of this condition
may result in weakness of mastication in 4% of cases.

Question 37
A 40 year old Vietnamese man who arrived in Darwin 6
weeks prior, presents with 5 days of headache, fever
and malaise. What is the MOST LIKELY diagnosis?
a) Malaria
b) Typhoid fever
c) Dengue fever
d) Filariasis

Answer: E
In an older Type 2 diabetic without microvascular
complications, a higher HbA1c may be satisfactory.
However, once the level is greater than 9%, therapy
must be reviewed to improve control. It is always
appropriate to review lifestyle factors. In this case the
gliclazide dose is already at the recommended
maximum. There is room to increase the metformin
dose. However a more effective strategy is to add a
thiazolidinedione.
Only
rosiglitazone
has
a
Pharmaceutical Benefit Scheme Authority listing to be
added to dual therapy with metformin and a
sulphonylurea. It must be monitored closely for possible
elevated liver enzymes and heart failure but would be
expected to lower HbA1c by 1-2%. Since nearly all

e) Meningococcal meningitis

Answer: A
Malaria is endemic in South-east Asia and a clinical
presentation such as this should be regarded as malaria
until proven otherwise. Its symptoms are usually nonspecific, with headache, fevers and malaise being the
most common symptoms. Typhoid fever has an
incubation period of 7 to 21 days, with an average of 14
days. Although headache and fever are common
symptoms, one would expect other symptoms such as
abdominal pain and diarrhoea. Dengue fever has an
incubation period of only 5-8 days, followed by sudden

headache, fevers and severe myalgias ("The Dandy


Walker Syndrome"). A rash typically develops on the
third to fifth day. Filiariasis (due to Wucheria bancrofti)
can present as a lymphangitis, with recurrent fever and
inflammation overlying the affected lymphatic vessel.
Finally meningococcal meningitis has a rapid clinical
course and the patient would be unlikely to present with
such a long clinical history. Photophobia, neck stiffness
and altered conscious state could also be expected.

Question 38
Peter presents with a painful left eye associated with a
sudden loss of vision. Which of the following conditions
is NOT usually associated with pain in or around the
eye?
a) Amaurosis fugax
b) Retrobulbar neuritis
c) Acute angle closure glaucoma
d) Temporal arteritis
e) Anterior uveitis

Answer: D
Facial pain analysis requires a disparate approach.
Neuralgias, particularly Trigeminal neuralgia, are
common causes and are more prevalent in women over
50 years. Neuralgias are characterised by paroxysmal,
fleeting, almost electric shock-like episodes that are
caused by demyelinating lesions of nerves that result in
the activation of a CNS pain-generating mechanism. The
pain of Trigeminal neuralgia, which can be in any branch
of the Trigeminal nerve, usually lasts a minute or two. A
characteristic feature is the initiation of pain by stimuli
applied to certain areas of the face, lips, or tongue, or
by movement of these parts. The adequate stimulus to
precipitate an attack is a tactile one and possibly a
tickle, rather than a noxious or thermal stimulus.
Carbamazepine is the drug of choice for Trigeminal
neuralgia. Dental pain is common, with provocation by
hot, cold or sweet foods being typical. Pain with chewing
itself
may
be
due
to
Trigeminal
neuralgia,
temporomandibular dysfunction or giant cell arteritis
with jaw claudication.

Question 40
The most important muscle used for inspiration is:
a) External intercostals

Answer: A

b) Diaphragm

Amaurosis fugax is the painless temporary loss of vision


in one eye associated with an embolus temporarily
lodging in a retinal arteriole.

c) Scalenes
d) Rectus abdominis
e) Internal intercostals

Question 39
A 62 year old housewife presents because of pain in her
left cheek precipitated by eating or touching her face
just under the left eye. This pain lasts for 30 to 60
seconds. The patient is most likely to benefit from:
a) Dental attention
b) Penicillin
c) Blockade of the infraorbital nerve
d) Carbamazepine
e) Corticosteroids

Answer: B
In resting healthy individuals, contraction of the
diaphragm is responsible for the majority of inspiration.
Clinically, it is important to remember that that the
diaphragm is innervated by the C3/4/5 spinal segments,
(mostly C4/5) via the phrenic nerves. Loss of function of
this segment, either from trauma or metastatic
malignancy, will result in the loss of the function of the
diaphragm and all intercostal muscles. The patient will
die of respiratory exhaustion in three days unless
respiration is supported. Unilateral phrenic nerve
damage (i.e. bronchogenic carcinoma), causes unilateral
hemi diaphragmatic paralysis which can cause a 20%
loss of inspiratory effort. This is, remarkably, quite
asymptomatic.

You might also like